You are on page 1of 223

2016

First edition

[QUICK GUIDE TO MEDICINE


IN PICTURES]
DR.Hamdy M.Ibrahim

2
Prefeace
I wish to express my greatest gratitude and appreciation to all my dear colleagues who
encouraged me to finish this illustrative book ,this book is done as a message of love and
respect to my dear friends on the face book , who gave me the idea, I extend my greatest
thanks to my dear brothers in Embaba Fever Hospital and my dear brothers joined me in the
critical care unit , This work cant be delivered without help,support and encouragement of
my colleagues in all Fever Hospitals .
I hope this diagnostic atlas will be suitable to offer the scientific and practical clinical
approach for ultimate target is to diagnose using photos as a guide , This book is suitable as
a quick reference guide to both undergraduate and post graduate doctors ,
Dr. Hamdy Mohammed Ibrahim
Internal medicine Consultant

Head of ICU in Imbaba Fever Hospital


Cairo, Egypt , 2016

Dedicated to the soul of my parents, who


learned me that the pleasure of life is to be a
renderer , To my Daughters Hanan and
Fatema ,To my son Ahmed hoping for them
better future life, to my wife who is always
standing behind my success.
Dr. Hamdy Mohammed Ibrahim
2016

Erythema marginatum
Macular lesion , Not painful ,Not itchy , Reddish pink color , Snake like ring with pale center ,
Rounded or Irregular shape , long lasting rash , begins on the trunk or arms and spread
outwards , the rash never starts on the face ,made worse with heat . G/E : Check vital signs
for fever , increased pulse rate , signs of heart failure , Search for presence of subcuteanous
nodules over extensor surfaces of joints , Examine the joints for arthritis (migratory
,polyarthritis involving large joints ) . Cardiac examination for valvular involvement , notice
tachycardia not proportional to the fever , continuous with sleep , persist after control of fever
, Check for gallop rhythm (q) What investigations should be done ? (A) Blood examination for
CBC , ESR, ASOT, CRP , Pharyngeal swab for sreptococcal antigen .
Chest x-ray for cardiomegaly , ECG : sinus tachycardia , YOU may see fixed prolonged PR
interval , Echocardiography : to exclude valvular involvement or evidence of pericardial
effusion .
Discussion : Erythema marginatum is one of the major criteria of rheumatic fever (Arthritis,
carditis , subcuteanous nodules , Erythema marginatum , Rheumatic chorea) , It is present in
70% of patients with acute rheumatic fever , it is a highly specific sign of acute rheumatic
fever .

5
Erythema infectiosum
Also known as fifth disease ,usually a benign childhood condition characterized by classic
slapped cheeck appearance ,it results from infection with human parvo-virus B19
erythrovirus , the symptoms begin approximately 1 week after exposure to the virus and last
2-3 days , they include headache,fever ,sore throat ,pruritus ,coryza,abdominal pain
,arthralgia ,these symptoms pass ,then a few days later the rash appears , the bright red
rash most commonly appears in the face particularly the cheecks this is a defining symptom
of infection in children , the rash typically last for 2 days . The patients are no longer
infectious once the rash appeared . Teenagers and adults may present with a self limited
arthritis ,it manifests as painful swelling of the joints (rheumatoid like picture)

6
Erythema nodosum
Look at the legs of this patient and note the rounded nodules up to 5 cm in diameter on the
anterior surface of the legs , Look for similar lesions on the extensor surfaces of the arms ,
Look into the throat for any evidence of sore throat and tonsilitis (since this usually precedes
erythema nodosum by 2-3 weeks ,the findings are often normal ) , Examine the respiratory
system for any evidence of tuberculosisor sarcoidosis commonly causing erythema nodosum
.
(Q) What are the important causes of erythema nodosum ?
(A) (1) Streptococcal infection (2) Sarcoidosis (3) Drug senstivity : penicillin , tetracyclines
,oral contraceptive pills ,sulphonamides ,sulphonureas (4) Tuberculosis
(5) Inflammatory bowel disease (6)Behcet's disease
(Q) What single question would you like to ask the patient ?
(A) What drugs has the patient been taking recently .
(Q) What investigations would you ask for ?
(A) Chest x-ray , Mantoux test (tuberculin skin test)

Erythema induratum
Occurs in the lower legs posterioly , usually in women ,with diffuse indurated dusky red lesions that
may ulcerate , It is more common in patients with poor cuteanous circulation . Epitheloid cell
granulomas may form .
It was originally described in association with tuberculous infection elsewhere in the body . It
represents a vasculitic reaction to the infection ,and when there is no tuberculous infection another
chronic infection may be responsible.

8
Erythema multiforme
Erythema multiforme is a hypersensitivity reaction to a variety of causes as drugs ,infection
or exposure to toxic substances , the disease occurs in 2 forms (minor and major ) , Erythema multiforme minor : is characterized by skin involvement only . The skin lesions are
variable and multiple and may be maculo-papular ,vesicular or urticarial . the lesions appear
in crops for up to 3 weeks and affect mainly the extensor surface of extremities and trunk
with symmetrical distribution ,palms and soles may also be affected . The characteric skin
lesion is ( Iris or target lesion ) is formed of urticarial lesions where dusky centers are
surrounded by darker rings.
Erythema multiforme major (Stevens Johnson): Erythema multiforme major is a severe
serious form characterized by involvement of the skin and mucous membranes , the illness
starts abruptly with fever and erythematous macules involving the face ,body palms and
soles . These macular lesions change into bullae involving the skin,lips ,mouth and
conjunctiva

9
Erythema Migrans (Lyme disease)
Caused by the spirochete Borrelia burgdorferi transmitted by tick bite , A ring like
erythematous lesions (erythema migrans) at the site of the tick bite is accompanied by
influenza -like symptoms ,with neurological or cardiac symptoms appearing weeks to months
later . Usually neurological involvement appears as viral- like meningitis with or without
cranial mononeuropathies . Treatment is with oral doxycline in the initial stages but the
meningitis is usually treated with i.v ceftriaxone .This characteristic rash should raise a
strong clinical suspicion of the diagnosis of Lyme disease , unless this is noted the diagnosis
may often be missed . Note the chronic induration with the characteristic red margin and
central clearing ,sometimes an eschar from the tick bite may be seen near the center of the
lesion

10

Herpes simplex
It is an extremely common problem caused by the herpes simplex virus ,of which there are
two main types . After a primary infection the virus remains in a latent state in nerve cell
ganglia and the patient may be liable to local recurrences ,the virus has the capacity to
invade and replicate in the CNS (Neurovirulence ) causing encephalitis , in
immunocompromized host , infection can cause life threatening complications ,reactivation of
latent HSV can be induced by various stimuli e.g.(Fever , trauma,emotional stress ,sunlight
and menstruation ) Type 1 herpes simplex is usually associated with cold sore and blisters
most commonly on the lips and face it is mainly transmitted by infected saliva . Primary
infection : may be associated with fever and painful vesicles in the mouth which make eating
and drinking difficult ,there may also be local lymphadenopathy
Type 2 herpes simplex : Most commonly associated with genital lesions and is likely to be
spread by genital contact . Ulceration occurs on the penis (men) and on vulva ,vagina ,cervix
this may cause problems of dysuria and difficulties with micturition ,ther may be constitutional
symptoms of fever ,headache and muscle pains .
Management : Primary herpes simplex : spontaneous recovery can be expected in about 2
weeks , plenty of cold fluids should be encouraged as well as mouth washes and
paracetamol to ease the pain .
Recurrent herpes simplex ; healing can be expected within 7-10 days ,there may be some
residual scarring . Advise the patient to avoid any triggering factors as much as possible and
recommend sunscreens when exposed to the sun ,topical acyclovir cream may shorten the
attack ,in severe cases an oral antiviral drug may be recommended , in some cases a
prolonged course of an oral antiviral may be given in an attempt to prevent frequent attacks .

11

Chicken Pox
Chicken pox is an infectious illness most commonly seen in childhood , It is caused by the
varicella zoster virus . Catching the disease should provide life long immunity . The
incubation period of the disease is 10- 21 days , with the patient infectious for 1-2 days prior
to the appearance of the rash and until about 10 days later when all the blisters have
scabbed . The diagnosis is normally obvious on clinical grounds , the patient first develops
crops of itchy red papules , the papules form teardrop -like vesicles and blisters surrounded
by a halo of erythema on the abdomen ,back and face before spreading to other parts of the
body ,the patient may be febrile and generally unwell .
In healthy children aged less than 12 years there is usually spontaneous resolution of
uncomplicated disease , Any serious complicatios (pneumonitis , Rey,s syndrome ,Guillian Barre syndrome, thrombocytopenia ,purpura ,hepatitis , encephalitis ) are more common in
adults ,pregnant women , the immunocompromised ,those who have taken systemic steroids
in the previous 3 months or those with chronic lung disease . Maternal infection in early
pregnancy carries a small risk of infection for the fetus , in contrast maternal infection at term
carries a great risk of infection for the newborn child . Although administration of varicella
zoster immune globulin does not prevent neonatal infection ,it greatly reduces mortality . In
children symptomatic treatment with fluids , paracetamol and topical calamine lotion for the
spots is usually sufficient .
However for those over age of 12 or any one who may be at greater risk of complications ,
oral antivirals may be considered e.g. Acyclovir ,if the patient is seen within 24 hours of the
onset of the rash . Any patient with chicken pox who becomes breathless should be referred
to hospital . The CDC recommend the use of intravenous acyclovir in pregnant patients for the treatment
of life-threatening herpes infections, including encephalitis, pneumonitis and hepatitis. Acyclovir has also
been recommended for the treatment of severe or progressive maternal varicella infection. However, the use
of acyclovir during pregnancy for non-life-threatening infections or for suppressive therapy is not
recommended by the CDC. Primary herpes simplex infection during pregnancy has been treated with
acyclovir, although this practice remains controversial. Some have suggested that acyclovir decreases the
incidence of adverse pregnancy outcomes such as prematurity and growth retardation.

12

Parotid swelling
Ask about the rate of growth of a parotid swelling ,and weather or not it is painful , and the
presence of any associated symptoms such as xerostomia (dry mouth) and xerophthalmia
(dry eye) may suggest sjogren,s syndrome (primary) ,look for any associated autoimmune
disorder like rheumatoid arthritis (secondary sjogren,s) . Look for any facial weakness
together with the rapid rate of growth suggest malignant tumours . Sudden swellingespecially
if painful suggest obstruction ,inflammation , or infection - Intermittent swelling occuring with
eating or drinking implies parotid duct obstruction and if painful could indicate parotid
calculus (rare may occur in hyperparathyroidism) .
Bilateral gland swelling suggest a metabolic aetiology (such as DM, Sarcoidosis ,anorexia ,
bulimia, or sjogren,s syndrome ) or an infective cause including mumps, CMV,and HIV .
Ask for previous parotid surgery for a neoplasm and if so the lump may be a recurrence .
Examine for site and size of the swelling and note overlying skin changes such as erythema
,ulceration,or a scar of previous surgery .
Palpate for size ,consistency ,tenderness and flactuance , infection and calculi cause diffuse
tender enlargement of the gland , palpate the neck to detect metastatic spread, or reactive
cervical lymohadenopathy .the oropharynx should be examined for peritonsillar or soft palate
swelling .
Inflammatory conditions of the parotid e.g acute suppurative parotitis present with diffuse
tender enlargement of the gland .
Granulomatous conditions such as TB, Sarcoidosis , syphilis , Wegner,s granulomatosis can
present with firm non tender swellings of both parotid and submandibular glands .

13
Mumps

Mumps is a contagious disease that is caused by mumps virus, typically starts with a few days of
fever ,headache ,muscle aches ,tirdness and loss of appetite and is followed by swelling and
tenderness of salivary glands under the ear in one or both sides ,Transmission : spread by
droplets of saliva or mucus from the mouth ,nose,or throat of an infected person usually when
coughs ,sneezes or talks ,most mumps transmission likely occur before the salivary gland begin
to swell and up to 5 days after swelling begins , the CDC recommends isolation of mumps
patients for 5 days after gland swelling ,as the swelling worsens ,the angle of the mandible is no
longer visible , swelling usually peaks in 1-3 days and then subsides during the next week
Complications : in 50-60% of clinical mumps CSF pleocytosis occurs , clinical evidence of
meningitis or encephalitis appears in <10% of patients with mumps , the most serious
complication is orchitis (testicular inflammation) rarely leads to fertility problems ,mastitis and
oophoritis in females who have reached puberty .,less common complications include :
pancreatitis ,myocarditis ,arthritis,thyroiditis ,deafness and spontaneous abortion . No specific
treatment for mumps ,supportive care should be given as needed ,bed rest ,a light diet ,increase
fluid intake ,avoid acid containing foods ,analgesics , Prevention : by mumps vaccine .

14

Rubella (German measles )


Rubella is usually a mild disease ,but infection during pregnancy may cause
severe congenital disorders particularly eye defect ,heart defect and deafness
,the virus is spread mainly by the air borne route with an incubation period of 2-3
weeks ,and infectivity from 1 week before symptoms until 1 week after the rash
appears .
A macular rash occurs on the face and trunk with mild fever ,occipital
lymphadenopathy and sometimes transient arthralgia , similar rashes may occur
with enteroviruses and parvo viruse infections . Rare complications are
encephalitis and thrombocytopenia . Treatment is generally symptomatic , if there
is concern about rubella infection in pregnancy aspirate blood for viral antibody
levels and arrange urgent follow up by the obstetrician .

15

Measles
Measles is a highly contagious serious disease ,it is one of the leading cause of death among
young children ,it is caused by a virus in the paramyxovirus family
Transmitted through direct contact and through air . Symptoms and Signs :
The first sign of measles is usually high fever which begins about 10-12 days after exposure to
the virus and lasts 4-7 days ,a runny nose ,cough ,red and watery eyes and small white spots
inside the cheeks (Koplik's spots) can develop in the initial stage ,after several days a rash erupts
usually on the face and upper neck ,over about 3 days the rash spreads eventually reaching the
hands and feet .,the rash lasts for 4-6 days and then fades ,the virus remains active and
contagious in the air or an infected surfaces for up to 2 hours ,it can be transmitted by infected
person for 4 days prior to the onset of the rash to 4 days after the rash erupts . Complications :
more common under the age of 5 or adults over the age of 20 , blindness , encephalitis , severe
diarrhea and dehydration ,ear infections ,severe respiratory infections such as pneumonia .severe
measles is common in poorly nourished young children especially those with insufficient vitamin A
,or immunecompromized patients ,pregnant women are also at risk of severe complications
,people who recover have long life immunity Treatment : no specific antiviral treatment ,
supportive care , ensure good nutrition ,adequate fluid intake and treatment of dehydration , All
children diagnosed as measles should receive two doses of vitamin A supplements , given 24
hours apart. to restore low vitamin A levels and help prevent eye damage and blindness,The
World Health Organization (WHO) recommends administration of an oral dose of 200,000
IU (or 100,000 IU in infants) of vitamin A per day for two days to children with measles in
areas where vitamin A deficiency Vitamin A supplement have been shown to reduce the
number of deaths from measles by 50% , Prevention : by routine measles vaccination .

16
Roseola infantum
Also known as sixth disease , it is a clinical syndrome characterized by 3-4 days of of high
fever may exceed 40 C that resolve abruptly and is followed by development of rash ,the
rash starts on the trunk and spreads rapidly to arms and neck with minimal face involvement
, It fades very rapidly in 24 hours . The rash can be considered as a good sign because
complete recovery will occur over the next 24 hours . the most characteristic feature is the
sudden drop of temperature with appearance of the rash . the condition should be suspected
in presence of high fever without localizing signs in late infancy ,febrile convulsions may
occur with the sudden rise of temperature in susceptible infants .Roseola is usually caused
by human herpes virus-6 .

17

Herpes zoster
Herpes zoster (shingles) is caused by the reactivation of the herpes zoster virus which
following chicken pox ,remains dormant in the dorsal ganglia .
When reactivated it affects the related nerve . The patient first complains of pain , often
severe ,along the area served by the nerve . This is followed within 1-3 days by a blistering
rash in the same area ,New lesions continue appearing for several days ,while the blisters
dry and crust over ,Recovery is usually complete within about a month . The pain tends to
lessen when the rash appears but may persist and be troublesome for months in some
patients ,particularly the elderly , Specific area -related problems include :
Ramsy -Hunt syndrome : Facial palsy due to involvement of the genuclate ganglion
.Vesicles occur on the ear and in the external auditory canal .
Sacral nerve involvement : This affects sacral nerve in the anogenital area and may lead to
urine retention and defecation problems .
Postherpetic paralysis : Temporary paralysis of a limb may occur if motor nerves are
involved in the dermatome .
Ophthalmic branch of the trigeminal nerve : pain ,numbness and tingling around the eye
followed by a blistering rash ,Potentially serious eye complications in 50% of cases e.g.
scleritis ,iritis ,keratitis and glaucoma, in these cases the rash usually extends to the tip of
the nose in the distribution of the nasociliary branch of the trigeminal nerve .
Management : Early antiviral treatment (within the first 3 days ) may reduce the pain and
duration of symptoms and is recommended for any facial shingles ,the
immunocompromised and the elderly ,Severe cases require IV acyclovir and occasionally
systemic steroids , Symptomatic treatment with analgesics ,topical calamin lotion or
povoiodine can be used .Oral antibiotics (e.g flucloxacillin or erythomycin ) will be required
in secondary infection . Analgesics may not be sufficient in postherpetic neuralgia and use
of tricyclic antidepressants , carbamazepine or sodium valproate may be helpful . Maternal
shingles does not adversely affect the fetus or newborn as they are protected by the
passively acquired maternal antibody .

18

Ramsy hunt syndrome


Ramsy Hunt Syndrome results from involvement of the facial and auditory nerves by the
varicella zoster virus . Herpetic inflammation of the geniculate ganglion is felt to be the cause
of this syndrome . Symptoms : painful rash on the ear drum ,ear canal , ear lobe,tongue and
roof of the mouth on the side where there is weakness of the face . Hearing loss on one side
,vertigo, weakness on one side of the face . Treatment : Acyclovir in a dose of 800 mg five
times daily . The newer antivirals ,valcyclovir 1000 mg and famciclovir 500 mg may be given
only three times daily , these agents are as effective as or superior to acyclovir .

19
Infectious mononucleosis :
Infectious mononucleosis also known as ( glandular fever ) is caused by Epstein Bar Virus
(EBV) , which is transmitted by saliva , Incubation period is : 30- 45 days .
Symptoms and Signs : Sore throat ,fever and malaise , Signs of exudative tonsilitis ,
lymphadenopathy and sometimes splenomegaly or a widespread maculopapular rash also
treatment with ampicillin (for presumed streptococcal infection) also causes widespread
maculopapular rash. Check for jaundice , and elevated liver enzymes for associated hepatitis
. Diagnosis :(1) From the clinical picture (fever ,sore throat, rash ,lymhadenopathy, and
splenomegaly. (2) Blood film : lymphocytosis with atypical lymphocytes (3) Positive mono
spot test (Paul -Bunnel ) for heterophile antibodies (4) If associated hepatitis there will be
increased serum bilirubin and transaminases .(5) Specific tests for EBV are also positive (
EBV IgM) for recent infection . (Q) What are the possible complications of infectious
monoculeosis ? (A) - Thrombocytopenia - Hemolytic anemia - Traumatic rupture of an
enlarged spleen - Rarely Guillian Barre syndrome ,cerebellar ataxia ,aseptic meningitis
,encephalitis ,pneumonitis,pericarditis or lymphoma . Comment: Atypical lymphocytes should
be differentiated from abnormal lymphocytes. Abnormal lymphocytes are associated with
lymphoreticular malignancies, whereas atypical lymphocytes are associated with various
viral and noninfectious diseases, as well as drug reactions. Atypical lymphocytes are each
different in their morphology as observed on the peripheral smear, whereas abnormal
lymphocytes are monotonous in their sameness, which readily permits differentiation on the
peripheral smear.

20
Dengue Fever or break bone Fever
It is an acute febrile viral disease (an arbovirus) transmitted by mosquitoes Aedes Aegyptii is
the commonest epidemic vector worldwide ,it has 4 serotypes DEN1,DEN2,DEN3,DEN4,Each
serotype provides specific life time immunity . There are actually 4 dengue clinical syndromes:
(1)Undifferentiated fever (2) Classic dengue fever (3) Dengue hemorrhagic fever (DHF)
(4) Dengue shock syndrome (DSS) Clinical Symptoms and Signs : Fever , headache ,retroorbital
pain ,flushing , muscle and joint pain ,nausea and vomiting , rash ,hemorrhagic rash .Incubation
period :3-14 days most commonly 4-7 days . Dengue fever is not spread by contact with infected
persons. Clinical case definition for dengue hemorrhagic fever : 4 criteria : (1) Fever or recent
history of acute fever (2) hemorrhagic manifestations (3) low platlet count (4) Objective
evidence of leaky capillaries (elevated hematocite-low albumin pleural or other serous
effusions . Clinical case definition for dengue shock syndrome(DSS): The 4 criteria for DHF +
evidence of circulatory failure : rapid and weak pulse narrow pulse pressure or hypotension
,cold clammy skin and altered mental status . Dangerous signs in Dengue Hemorrhagic fever : Abdominal pain (intense and sustained ) persistent vomiting Abrupt change from fever to
hypothermia with sweating and prostration restlesness or somnolence , these are signs of
impending shock . Diagnosis : CBC : Leukopenia and thrombocytopenia Dengue antigen
detection by PCR increased IgM antibodies titres . Management : No specific drug patient
isolation in a mosquito free environment patients infected with classical dengue usually
recovers in 1-2 weeks For serious cases ,supportive treatment provided by hospitals.

21

Bird Flu (Avian flu) H5N1


Human infection is serious with very high mortality , encephalitis as a complication has been
reported in some cases .It is considered as a zoonotic disease , human infection is an exception .
It is caused by strain influenza A virus H5N1. Case definition : Suspected Case : Fever with one
or more of the following : 1- Sore throat 2- Cough and /or sneezing 3- Body aches
and/headache 4- Positive epidemiological history in last 2 weeks in the form of : Contact with
birds (dead ,diseased ,or normal ) , Travelling or coming from an epidemic or epizootic during
the last 2 weeks , Or any complain after sure exposure to sure diseased birds . Confirmed Case :
Suspected case confirmed by laboratory isolation of the virus by PCR for throat or
nasopharyngeal swab . Rapid flaring up of lung infection ,the whole lung can be opacified in less
than 24 hours in non treated cases
High risk groups are : eldrly patients above 65 years and below 5 years , dibetes mellitus,
obesity , pregnancy , immunocompromized patients cadio-pulmonary disease ,chronic kidney
diseases
Treatment: Bed rest good nutrition multivitamins - Hospitalization in high risk and severe
cases , Antibiotics if there is suspicion of secondary bacterial infection. Specific treatment :
Oseltamivir 2-3 mg/kg body weight daily in 2 divided doses or 75 mg twice daily in adults
.Mechanical ventilation when needed.

22
Swine Flu
It is very highly infective virus , transmitted from human to human ,caused by virus influenza
A H1N1 Case Definition : Suspected Case : Fever with one or more of the following (a)
Running nose and sneezing (b) Cough (C) Sore throat (d) Body aches ,chills and headache
(e) vomiting and diarrhea in some cases with positive epidemiological link : a- History of
contact with a confirmed case in the last week (b) History of travelling to epidemic area in the
last week(c) History of contact with a swine . Confirmed Case : Suspected case confirmed by
laboratory identification of H1N1 virus by PCR through nasopharyngeal swab. High risk
patients may develop severe symptoms ,those are extremes of age(below 2 years and
above 65 years ) , diabetes mellitus , obesity , immune-compromised patients , patients with
chronic kidney disease , patients with cardio-pulmonary disease . Treatment : bed rest ,
isolation at home in mild cases , symptomatic treatment , antibiotics for persistent fever or
suspicion of secondary bacterial infection , high risk and severe cases must be hospitalized ,
oseltamivir (tamiflu) 2-3 mg/kg in 2 divided doses ,or 75 mg twice daily for adults ,or
zanamivir 2 nasal puffs (10 mg twice daily.multivitamins ,antibiotics ,ventilation when needed

23

.
Pyoderma gangrenosum
Pyoderma gangrenosum is thought to be an autoimmune disorder that affects patients with
an underlying condition such as rheumatoid arthritis ,ulcerative colitis , Chron,s disease or
chronic active hepatitis ,the patient may present with a painful red nodule that becomes blue
centrally ,often with a superficial blister that quickly ulcerate ,The ulcer rapidly deepens and
has purple undermined edges . Another presentation is of a superficial kind with pustules .
Ulcerated lesions usually occur on the legs , but the more superficial variety may be seen
anywhere . Lesions may sometimes (though not always ) follow trauma ,healing is slow over
a period of months .Diagnosis is made on clinical grounds from the appereance of the
lesion(s) , Biopsy is useful only in the exclusion of other causes , Secondary infection is
always possible and a swab taken from an area for culture will confirm any such problems
and indicate weather an antibiotic is necessary . Necrotic tissue should be removed from the
ulcer ,being careful not to enlarge it , Other treatment may help to arrest the process but
healing is slow . Small ulcers may be helped by topical or intralesional steroids and oral antiinflammatory ,antibiotics such as dapsone or minocycline . ,The area can be dressed with
silver sulfadazine cream or hydrocolloids and compression bandage may be useful in cases
of swollen legs . More severe problems will require immunosuppressive treatment , Choices
range from tacrolimus ointment to systemic steroids ,ciclosporin ,methotrexate
,cyclophosphamide ,mycophenolate or infliximab

24

Bacterial Meningitis
Definition : Inflammation of the meninges covering the brain and spinal cord
Bacterial meningitis most commonly results from hematogenous spread of microorganisms
from a distant site of infection or site of colonization e.g nasopharynx as meningococcal
meningitis
Risk factors for meningitis : otitis ,sinusitis,mastoiditis ,CSF rhinorrhea ,closed head trauma
,sickle cell anemia ,asplena,,neurosurgical procedures, immunosuppression
Diagnostic criteria : Clinical : Fever,headache, lethargy,photphobia , vomiting ,feeding
problems,convulsions,bulging fontanel ,altered level of consciousness(in infants), signs of
meningeal irritation e.g. positive kernig;s and Bruidisinski's signs
Signs of increase intracranial pressure e.g. papilloedema .
Investigations : Lumbar puncture to obtain CSF ,it should be delayed when focal neurological
deficits(e.g. focal fits ,aphasia, monoplegia,hemiplegia) are present and papilloedema until
neuroimaging procedure has been done to exclude other intracranial pathology or
complications and also to avoid brain herniation if lumbar puncture done in such cases . CSF
: TLC:typically >1000 WBC/microL, with a predominance of neutrophils . Traumatic LP:
should be treated presumptively for meningitis pending results of CSF culture.
The presence of a single neutrophil in the CSF is considered abnormal
Glucose: <40 mg/dL in > 50% cases. ratio of the CSF to blood glucose concentration is
usually depressed (<0.66)
Protein:100 to 500 mg/dL . Blood cultures: positive in 50 % of patients. Among children who
were not pretreated with antibiotics.Gram stain , microscopy and culture Radiology: .Brain
CT with contrast or MRI with gadolinium .

25

TB meningitis
Definition : inflammation of the meninges covering the brain and spinal cord due
to invasion of the subarachonid space by TB bacilli .
TB meningitis : 3 stages (1)The prodromal stage of invasion with non specific
symptoms (2) The stage of clear cut meningeal manifestations (3) The terminal
stage of paralysis and coma . Diagnostic criteria : the diagnosis depends on high
index of clinical suspicion : - Previous history of pulmonary or extrapulmonary TB
or - Contact to a tuberculous patient - Long insidious history of non specific
complaint like headache .- Irritability ,loss of weight ,altered sensorium ,,signs of
increase intracranial pressure - signs of cranial nerve affection .
First prodromal stage : diagnosis remains in doubt for 2-3 weeks .
-Non specific symptoms: restless sleep,drowsiness, loss of appetite ,low grade
fever ,headache,vomiting . Second (meningeal stage) : signs of meningeal
irritation ,stiff neck and back ,positive kernig;s and Bruidsinski's signs ,bulging
anterior fontanel in young children ,exaggeration of deep tendon reflexes
sensorium is clouded ,temperature is increased ,vomiting becomes more
frequent .
Fundus examination : papilloedema , peripherally located choroidal tubercles
may be present , cranial nerve palsies ,(this stage lasts 7-10 days )
Third stage : stupor progresses to coma ,pupils become dilated and fixed
,corneal reflex is absent ,deep tendon reflexes disappear ,emaciation and
scaphoid abdomen are notable ,irregular respiration develop .Investigations :(1)
CSF : WBCs : variable(10-1000 cells/ul) <500 cells/ul,predominance of
lymphocytes , initial stage PMN predominance changed later to lymphocytic
predominance , Glucose: <40mg/dl low
Protein : (moderate to marked increase (50-500mg/dl), (2) CSF culture (87%
diagnostic ) (3) PCR : 60% SENSTIVE. (4) Adenosine deaminase : Greater than
6u/l in patients with meningeal syndrome strongly supports the diagnosis of TBM
and permits the commencement of antiTB treatment (5) Tuberculin skin test : in
the absence of symptoms usually the only sign of TB infection in children is a
positive TB skin test .(6) CBC: anemia with high sedimentation rate . (7) imaging
: Chest x-ray (may be associated pulmonary TB) (8) Brain CT with contrast or
MRI with gadolinium may show meningeal enhacement ,areas of infarctionand
hemorrhage may be seen, also single or disseminated tuberculomata may be
seen

26

Cerebral tuberculomas
CNS tuberculomas is a serious form of TB due to hematogenous spread of mycobacterium tuberculosis
,manifesting as meningitis,cerebritis, and tuberculous abscesses or tuberculomas .
Intracranial tuberculomas are the least common presentation of CNS TB .
Tuberculomas often present with symptoms and signs of focal neurological deficit without evidence of
systemic disease ,the radiologic features are also non specific and differenitial diagnosis includes ,malignant
lesions ,CNS sarcoidosis, pyogenic abscesses ,cerebral toxoplasmosis ,and cerebral cysticercosis . The
patient can present with prolonged low grade fever , prolonged headache , convulsions ,Localized
neurological deficits (cranial nerve affection,cerebral and cerebellar affection) and papilloedema are
common . Obstructive hydrocephalus is a common complication

27

Encephalitis
Definition : encephalitis is inflammation of the brain tissue .
Etiology : Viral ,bacterial ,protozoal (p.falciparum),fungal, viral etiology is the most common.
Transmission : (1) Person to person : e.g. Herpes simplex ,varicella zoster ,measles
,mumps, influenza viruses,enteroviruses, ,adeoviruses .
(2) Spread by mosquitoes or ticks: e.g. jabanese B, Dengue fever,West nile virus
encephalitis (3) Spread by warm blooded animals e.g. Rabies encephalitis .
The history can provide useful clues to the etiology ,never accept that the history is not
available for a confused patient also comment from relatives that a patient does not seem
right should not be ignored ,ask about recent rashes,ask weather others in the family or in
the community have been affected .parotid swelling,testicular pain or abdominal pain due to
pancreatitis may suggest mumps encephalitis ,travel history is important Clinical picture :
altered level of consciousness ranging from mild lethargy to coma ,focal or generalized
seizures may occur .
Examination : assess degree of coma ,examine the skin for rash,bite,injection site ,examine
the genitalia for ulcers or vesicles (Herpes simplex virus type 2),examine the inside of the
oral cavity and the sides of the tongue for evidence of previous convulsion ,Look for focal
neurological signs or flaccid paralysis ,look for tremors or invoulntary movements ,examine
the pupils for size,reactivity,and equality on both sides ,fundus examination for papilloedema
, look for signs of cranial nerve affection .
Investigations : CSF: usually lymphocytic pleocytosis (5-500cells/mm3, early in the disease
cells might be polymorphnuclear ,later mononuclear cells predominate ,there may be no cells
in the early or late L.P. Protein content is mildly elevated ,glucose content is normal , PCR :
95% SENSITIVITCTY in HSV ,may be negative early in the disease or 10 days from the
onset. CT scan : in HSV the scan may be normal initially or there may be subtle swelling of
fronto- temporal region , subsequentely there is hypodensity or high signal change of
hemorrhagic transformation .MRI Scan : can look normal if performed very early , more
sensitive than CT .

28

Decorticated patient with encephalitis

Patient with Herpes simplex encephalitis

Decerberated patient with encephalitis

29
Rabies encephalitis
patient with rabies encephalitis (Furious rabies): Look for the clinical features which include : Hydrophobia
,Aerophobia,spasticity,excitement,agitation and confusion on examination you can find automonic changes
in the form of flacutating body tepmperature and blood pressure ,sweating,tachycardia ,hypersalivation
Rabies is a viral zoonotic neuroinvasive disease that causes encephalitis or encephalomyelitis. The
domestic dog is the single most important animal reservoir ,dogs are the source of 99% of human rabies
deaths. The virus is transmitted through infected animal bite wound , incubation period from days to years
,average 4-6 weeks , short incubation period in children or when site of injury is near to the CNS like the
face or the neck , the rare type paralytic rabies begins in the bitten limb and ascends symmetrically or
asymmetrically until it involves the muscles of respiration and deglutition , this type runs a less dramatic and
longer course than furious type and is often misdiagnosed . Diagnosis : from the clinical picture , viral
detection in saliva ,throat swab, or tracheal aspirates by PCR, antibody detection (direct fluorescent
antibody , seum and spinal fluid for rabies antibodies , Brain biopsy of dead rabitic animal showing Negri
body in neuron cell

30

Xanthochromic CSF:
Xanthochromic CSF is a yellow ,orange , or pink discoloration of the CSF ,most often caused by
lysis of RBCs causing hemoglobin breakdown to oxyhemoglobin ,methemoglobin and bilirubin .
Xanthochromia is present in more than 90% of patients within 12 hours of subarachnoid
hemorrhage . CSF protein level of at least 150 mg/dl asseen in many infectious and inflammatory
conditions or as a result of traumatic tap that contains more than 100,000 RBCs per mm3 also will
result in xanthochromia . Newborn CSF is often xanthromic because of the frequent elevation of
bilirubin and protein levels in this age group.

Cerebrospinal Fluid Supernatant Colors and Associated Conditions or Causes


COLOR OF CSF
SUPERNATANT

Yellow

CONDITIONS OR CAUSES

Blood breakdown products

Hyperbilirubinemia

CSF protein 150 mg per dL (1.5 g per L) >100,000 red blood cells
per mm3

Orange

Blood breakdown products

High carotenoid ingestion

Pink

Blood breakdown products

Green

Hyperbilirubinemia

Purulent CSF

Brown

Meningeal melanomatosis

31

Tetanus
Clinical illness characterized by acute onset of hypertonia and /or painful muscular
contractions (usually the muscles of the jaw and neck) and generalized muscle spasms
without other apparent medical cause.
Tetanus is caused by a neurotoxin produced by clostridium tetani ,an anaerobic spore
forming bacillus.Tetanus is not directly transmitted from person to person ,infection occurs
when tetanus spores are introduced into the body Clinical features : Clinical triad of
rigidty,muscle spasms and autonomic dysfunction . Neck stiffness ,sore throat , and difficulty
opening the mouth are often early symptoms . Masseter spasm causes trismus or (Lockjaw)
. Spasm progressively extend to the facial muscles causing the typical facial expression of
fixed smile (risus sardonicus ) and muscles of swallowing causing dysphagia , rigidity of the
neck muscles leads to retraction of the head . Truncal rigidity may lead to opisthotonos which
is the severe arching of the back during a spasm . Respiratory difficulty with decreased chest
wall compliance may also ensue .There are episodic muscular spasms ,these tonic
contractions have a convulsion like appearance affecting agonist and antagonist muscle
groups together .may be spontaneous or triggered by touch ,visual ,auditory or emotional
stimuli .Spasms vary in severity but may be strong enough to cause fractures and tendon
avulsions.,there may be autonomic instability and arrhythmias

eL

32

Botulism
Neuroparalytic disease caused by neurotoxins (BoNTs) produced by bacteria Clostridium
botulinum , seven different BoNTs (A-G) produced by different strains , Strains ABEF cause
outbreaks in humans , BoNT A:most common and most potent
Organism : Clostridium botulinum ,gram positive obligate anaerobic bacillus , spores
resistant to heat,light, drying and radiation , all species cause flaccid paralysis , Human
disease : three forms :(1)Food borne botulism (2)Wound botulism (3)Infant botulism , all
forms are fatal and a medical emergency . Incubation period : 12-36 hours.(1) Food borne
botulism : preformed toxin ingestion from contaminated food ,most common from home
canned foods , beets ,green beans, chile peppers, tomatoes ( type A) , Impropely fermented
fish (type E) (2) Infant botulism : Spore ingestion in infant below 1year , spores germinate
then toxin released and then colonize in large intestine (3)Wound botulism : organism enter
wound .infection develops under anaerobic conditions from ground ,dirt or gravel ,it does not
penetrate intact skin
Adult clinical signs : Nausea ,vomiting ,diarrhea ,Double vision ,Difficulty speaking or
swallowing , Descending weakness or paralysis , shoulders to arms to thighs to caves ,
Symmetrical flaccid paralysis . Respiratory muscle paralysis . Infant clinical signs :
Constipation , lethargy,poor feeding ,weak cry , Bulbar palsy Diagnosis: Clinical signs
Toxin in serum ,stool , gastric aspirate ,suspected food Culture of stool or gastric aspirate
Electromyography .
Management: ICU admission Mechanical ventilator for respiratory failure.
Botulinum antitoxin : Derived from equine source
Botulinum immunoglobulin : infant cases type A and G

33

Opisthotonos
It is a condition in which the body is held in an abnormal position , the person is usually rigid
and arches the back with the head thrown backwards . if the patient lies on his or her back
only the back of the head and the heels touch the supporting surface .Opisthotonos is much
more common in infants than in adults , it is also more exaggerated in infants and children
because of their less mature nervous system .
Causes : Opisthotonos may occur in infants with meningitis , it may also occur as a sign of
reduced brain function or injury of the nervous system
Other causes : Tetanus , subarachnoid hemorrhage , strychnine poisoning , Archnold chiari
syndrome , Brain tumours , seizures , severe electrolyte imbalance . You should ask about
the onset and what other symptoms come before or with this abnormal posturing ( such as
fever , stiff neck and headache ), and if there is any recent history of illness .
Investigations : Blood and urine tests , CSF examination including culture
CTscan of the brain ,MRI, serum electrolytes

Lik

34

Trichinosis
Trichinosis is a parasitic disease caused by round worms trichinella spiralis .Trichinosis is
mainly caused by eating undercooked meat ,Pork products are implicated more often than
other meats . The severity of symptoms usually depend on the number of larvae consumed
in the infected meat .,infection may occur without symptoms (in mild infestation) .,while
intestinal invasion can cause dirrhea ,abdominal pain or vomiting . Larval migrationinto
muscle tissue (one week after being infected ) can cause edema of the face or around the
eyes ,conjunctivitis ,fever muscle pains ,splinter hemorrhages ,rashes and peripheral
eoisinophilia . Life threatening cases can result in myocarditis ,CNS involvement and
pneumonitis . Larval encystement in the muscles causes pain and weakness.
Diagnosis : from the history of eating uncooked pork , from the clinical picture and
investigations : C.B.C.: Eosinophilia - Elevated creatine kinase (CK) level in muscle
involvement - Muscle biopsy .

35

Filariasis
Infection with long and thread like nematodes which normally resides
in the lymphatics of infected people , the mosquito is the main
transmitter .
Clinical features : some patients suffer from acute recurrent filarial
fever ,lymphadenitis,and lymphangitis , Some patients who have low
levels or undetectable microfilaria show chronic signs including
hydrocele, chyluria, elephantiasis of the limbs ,breasts and genitalia
Diagnosis by detection of microfilaria in the peripheral blood - serologic
investigations using immuno-chromatography test for the specific
antigen of the filarial nematode

36

Malaria
Definition : it is a disease caused by the protzoon parasites of the genus plasmodium , 4
species : (1) P. falciparum : the most important species ,it is potentially fatal .(2) P.vivax : the
malaria parasite wiyh the widest distirubtion .
(3) P,ovale : relatively rare (4) P.malariae : Responsible only for 7% of malaria cases .N.B
Mixed infections are common , malaria parasires are transmitted from person to person by
the bite of female anopheles mosquito. Incubation period : 1-3 weeks
It should be emphasized that: fever occuring in a traveller within 3 months of departure from
a malaria endemic area is a medical emergency and the patient should be considered as
malaria until proved otherwise .
Symptoms and signs of simple uncomplicated malaria : -Flu like symptoms .fever ,chills
,muscle aches, and headache some patients develop nusea,vomiting ,cough and diarrhea
,pallor jaundice .tender hepatosplenmegaly ,fever is the most constant symptom of malaria ,it
may occur in paroxysms . Malarial paroxysm :- cold stage:(rigors). - Hot stage : maximum
temperature can reach 40-41C
- Sweating stage : last 8-12 hours start between midnight and midday .
Severe and complicated malaria: (1) Cerebral malaria (2) Severe malaria anaemia
(3) Hypoglycemia (4) Metabolic acidosis (5) Acute renal failure (6) Pulmonary edema (7)
Circulatory collapse , shock .or algid malaria (8) Black water fever
Cerebral malaria : Clinical features : High temperature or (hypothermia ,impaired
consciousness : from prostration to deep coma ,convulsions (bad sign) .other manifestations
may coexist : hypoglycemia, hyponatremia , multiorgan failure ,prolonged post ictal state ,
other infections (may coexist) : meningitis,sepsis
Other severe complications : thrombocytopenia, DIC . Diagnosis : History: Residence or
recent visit in the preceding 3 months to a malaria endemic area , History of fever ,may be
paroxysmal in nature ,non specific clinical features such as vomiting ,diarrhea,headache
,malaise Examination: Identify signs consistent with malaria ,fever ,pallor,
jaundice,splenomegaly , Investigations : Blood film : thick and thin blood films to identify
density and species of parasite ,dipstick test,igM,igG level,,antigen detection by ELISA,,PCR
(very sensitive and specific .

37

Like
Love
Haha
Wow
Sad

38

Cachexia
Cachexia or wasting syndrome is loss of weight ,muscle atrophy
,fatigue,weakness , and significant loss of appetite in someone who is
not actively trying to lose weight . Cachexia may occur in a number of
severe disorders including chronic lung diseases such as pulmonary
fibrosis some auto immune disorders , ,tuberculosis and emphysema
chronic pancreatitis,moto neurone disease ,malignant diseases
including bronchial carcinoma , and infection especially with HIV (Slim
disease ) . Note the obvious signs of weight loss with wide spread
muscle and soft tissue wasting .

39

Sites of infection in critically ill patients


(A) Major sites of infection:
(1) Intravenous lines (particularly central) : Always suspect if the patient develops evidence
of sepsis and lines have not been changed for >4 days they must be replaced .
(2) Lungs : high risk of nosocomial pneumonia in intubated patients ,after being in ICU for
more than 3-4 days particularly if treated with antibiotics ,the nasopharynx become colonised
with gram negative bacteria which migrate to the lower respiratory tract ,prophylaxis with
antibiotics to reduce the incidence of nosocomial pneumonia .
(3)Abdomen : Consider intra abdominal abscess or necrotic gut in patients who have had
abdominal surgery ,pancreatitis ,acute choleycystitis , or perforated peptic ulcer may develop
as a complication of critical illness ,ultrasound ,CT ,aspiration of collections of fluid/pus and
laparotomy may be required .
(4) Urinary tract : always culture a catheter specimen of urine in unexplained sepsis .
(B) Other sites of infection :
(5) Heart valves : transthoracic or transessophageal echocardiogram .
(6) Meninges : Lumbar puncture but check coagulation and platlet count first . (7) Joint and
bones : x-ray , gallium or technatium white cell scan .
(8) Nasal sinuses ,ears,oropharyngeal space : diagnosis clinical examination , plain x-ray ,
CT.
(9) Genito urinary tract (particularly post partum) : PV examination ,ultrasound . (10) Gastro
intestinal tract : PR examination ,stool culture , clostridium difficile toxin assay , sigmoidscopy

40

Pulmonary tuberculosis
Tuberculosis is an infection caused by slow growing bacteria that grow best in
areas of the body that have lots of blood and oxygen that's why it is most often
found in the lungs this is called pulmonary TB. Tuberculosis is either latent or
active .
Latent TB (TB infection) : TB infection is the state of having a small number of
M.tuberculosis bacteria in the body which are unable to grow due to control by
the immune system , The bacteria are inactive ,but remain alive in the body and
can become active later . TB infection doesn't cause a person to feel sick and
there are no symptoms and no signs detected upon medical evaluation .
diagnosed by tuberculin skin test (false negative may be seen in HIV , while false
positive may be seen in persons who are previously vaccinated) ,treatment of TB
infection with isoniazide alone . TB didease : develops when the immune system
can not keep the tubercle bacilli under control and the bacilli begin to multiply
rapidly . TB didease can develop very soon after infection or many years after
infection .common symptoms of TB disease are persistent cough, (more than 2
weeks) ,bloody sputum,weight loss or loss of appetite ,fever or night sweats .
Chest x-ray usually abnormal ,sputum smears and cultures usually positive . The
patient should be considered infectious until the person Has completed at least 2
weeks of standard anti-TB therapy with direct observation by a TB program , Has
had three consecutive negative sputum smears on three different days with at
least one morning specimen and has improvement in symptoms., A TB suspect
should be considered infectious until a diagnostic investigation is completed.
Miliary TB occurs when tubercle bacilli enter the blood stream and are carried to
all parts of the body where they grow and cause disease in multiple sites.

41

Brucellosis
Zoonotic disease of worldwide distribution , Brucella species are the causative
organism which are gram negative intracellular coccobacilli , killed by
pasteurization and not killed by freezing . Brucella meletenisis ,abortus and suis
are the common reservoirs . The disease is transmitted by ingestion of infected
milk and its products , raw meat liver ,spleen and bone marrow , through
inhalation in endemic areas , direct contact of abraded skin with infected meat .
the incubation period is 1-3 weeks . the disease may be acute (1st three months
) ,suacute , (3-12 months ) chronic (more than 12 months ) , Clinical picture : the
patient presents acutely with fever ,rigors , sweating ,headache ,low backache
.Signs : the patient is acutely ill ,febrile ,hepatosplenomegaly , lymphadenopathy
may be present .tender spines , epidydemo-orchitis may be observed , rarely
meningo-encephalitis or endocarditis . In chronic brucellosis the onset is
insidious with headache , low backache sweating and depression ,the
temperature may be normal . Complications: may be the presentation including ;
Reactive ,polyarticular, migratory resembling rheumatic fever ,or septic
(monoarticular) , spondylitis , osteomyelitis ,endocarditis, hepatitis, epididymoorchitis , neurobrucellosis Diagnosis: Standard agglutination test : 1/160 in non
endemic area and 1/320 in endemic areas ,it is false negative in : -prozoon
phenomenon ,patients with immunosuppression and hypoprotenemia , false
positive in cross reaction with other organisms , high titre of old infection
remains high in risky groups like farmers ,butchers and vetrineans
(2)Modified brucella test (Mercapto ethanol test) more specific ,positive in chronic
brucellosis (3 ) a dip stick test : rapid test to detect IgM . (4) Blood culture : takes
about 6 weeks (5) PCR : still under evaluation

42

43

Leprosy
Leprosy is an infectious disease caused by the slow growing intracellular acidfast bacillus mycobacterium leprae .
Transmitted person to person by respiratory droplets from the nasal mucosa .
Incubation period 2-12 years .
Clinical features : The organism grows well at lower temperature and thus affects
the skin , upper respiratory mucosa , superficial nerves ,anterior chamber of the
eye ,lymph nodes and testis . Features are depigmented anaesthetic skin
patches ,or other sometimes nodular skin lesions ,thickened superficial nerves
e.g ulnar nerve ) or signs of involvement of other organs such as blindness .
There is a spectrum of disease from tuberculoid (paucibacillary) to lepromatous
(multibacillary) types .
Diagnosis : From clinical features .
Skin smear from the lesion stained for mycobacteria .
Treatment : WHO recommends combination drug treatment of rifampicin
,dapsone , and clofazimine for 12 months for multibacillary leprosy .
and rifampicin and dapsone for 6 months for paucillary leprosy .

44

Leptospirosos (Weil's disease )


Leptospirosis caused by the spirochaete leptospira interrogans and other
leptospira species ,is spread by contact with infected rat's urine ,often in rivers
,canals or sewers . The leptospires enter the body through small breaks in the
skin or via mucus membranes of the eyes or nose . About 10 days after exposure
(range 2-26days ) the illness starts with fever , severe muscle pains ,headache
,sore throat ,nusea and vomiting ,conjunctival redness is common . A
haemorrhagic rash , jaundice ,renal failure and pulmonary hemorrhage may
occur (Weil's disease) .
Treatment is with penicillin or doxycycline with supportive care and hemodialysis
if necessary . Prophylactic doxycycline is reasonable for people who fall into
waterways likely to be contaminated with leptospires .

45

Like

Erysipelas
Erysipelas is a bacterial skin infection involving the upper dermis that characetrstically
extends into superficial cuteanous lymphatics .
Historically : erysipelas occured on the face but cases today most often involve the legs (7080 %)
Streptococcus causes most of the facial erysipelas ,increasing percentage of lower extremity
infections are now being caused by non group A streptococci .
Examination : check for portal of entery as insect bite ,surgical incision, venous insufficiency ,
stasis ,ulceration ,inflammatory dermatoses .
Notice the intensly erythematous indurated plaque with a sharp demarcated border .
check for local tenderness lymphatic involvement is manifested by overlying skin streaking
and regional lymphadenopathy its more severe infections may exhibit numerous vesicles and
bullae along with petechiae and even necrosis , its well defined margin can help differniate it
from other skin infections e.g. cellulitis
Management : symptomatic treatment for aches and fever , good hydration ,
Bed rest ,elevate the affected limb to reduce local swelling ,inflammation and pain and use
cold compresses for times daily for 48 hours .saline wet dresses to ulcerated and necrotic
lesions and changes every 2-12 hours
Hospitalization for close monitoring and i.v antibiotics in elderly patients ,immunocomprized
,severe and infants , penicillin is the first line therapy ,orally or IM is sufficient in most cases
for 5 days can be extended for 10-14 days . first generation cephalosporins ,macrolids such
as erythromycin or azithromycin if the patient is allergic to penicillin ,for atypical forms of
erysipelas including bullous erysipelas and facial erysipelas (some authors ) should be
treated empirically with penicillinase resistant antibiotics such as dicloxacillin or naficillin to
cover possible staph aureus ,. Two drugs : roxithromycin and pristinamycin have been
reported to be extremely effective in the treatment of erysipelas .
Benzathin penicillin 2.4 million units IM every 3 weeks for up to 2 years .

46

Scarlet Fever
It is an infectious disease caused by streptococcus pyogenes ( group A strept.) ,
usually spread by inhalation ,it is most common in children 5-15 years of age .
Scarlet fever features a bright red rash that covers most of the body this rash is
caused by an erythrogenic toxin released from the bacterium (streptococcus
pyogenes) ,it is almost accompanied by sore throat and high fever . The signs
and symptoms include :
Red rash : The rash looks like a sunburn and feels like a sand paper ,it appears
12-72 hours after the fever start ,on the face it shows as red cheeks with
characteristic pale area around the mouth (circumoral pallor) , it typically begins
on the face or neck and spreads to the trunk ,arms and legs ,if pressure is
applied to the rash it turns pale ,begins to fade 3-4 days after onset and
desquamation begins .the rash may be itchy .
Red lines : The folds of skin around the groin ,axillae ,elbows ,knees and neck
usually became a deeper red than the surrounding rash .
Flushed face : the face may appear flushed with a pale ring around the mouth .
Strawberry tongue : the tongue look red and is often covered with a white coating
early in the disease . Incubation period : 2-4 days .
Rarely scarlet fever can lead to rheumatic fever .
Differnitial diagnosis : kawasaki disease is an important differential especially in
its incomplete form ,scarlet fever appears similar to kawasaki disease in some
aspects but lacks the eye signs or the swollen red fingers and toes .

47

Stasis dermatitis and Stasis eczema


Stasis dermatitis is a common inflammatory skin disease that occurs in the lower
extremities , it is usually the earliest cuteanous sequela of chronic venous
insufficiency accurate diagnosis is critical as many patients admitted for the
treatment of cellulitis actually have stasis dermatitis . Symptoms : swelling
,redness ,scaling ,itching or pain Varicose veins and congestive heart failure are
known to cause leg swelling and stasis dermatitis
Stasis eczema is commonly seen in elderly women in association with venous
insufficiency or frank ulceration , there is also marked pigmentation as a result of
hemosidrin deposition .
Diagnosis : Doppler ultrasound on the lower limbs .

48

Dermatitis herpetiformis
This is a rare condition that usually presents in young adults ,although it is
occasionally seen in children ,the cause is uknown but it is associated with a
gluten senstive enteroathy . Biopsy of the jejunum shows the same changes that
are seen in coeliac disease . Such patients are more liable to develop other
autoimmune conditions such as Graves disease and pernicious anemia . There
is also a small risk of gastrointestinal malignancy particularly lymphoma . The
patient presents with extremely itchy urticarial papules and vesicles that usually
occurs over the elbows ,knees,back and buttocks ,Dermatitis herpitiformis may
resemble scabies,papular urticaria,insect bites ,contact dermatitis ,and papular
eczema .. Management : skin biopsy and direct immunoflurescence are
necessary to confirm the diagnosis ,jejunal biopsy is sometimes recommended
,the patient must be advised to adhere to a strict gluten-free diet which in some
cases may be sufficient to control the problem and may also be protective
against lymphoma , If necessary Dapson 50 mg bd will quickly releive the
irritation within a few hours ,although symptoms return when it is discontinued.

49

Deep venous thrombosis (DVT)


Deep venous thrombosis, (DVT) is the formation of a blood clot (thrombus) within
a deep vein, predominantly in the legs. Patient presents as acutely swollen leg
with diltation of the superficial veins On examination the leg is hot to touch and
palpation along the line of femoral and popliteal veins causes pain. Pulmonary
embolism, a potentially life-threatening complication, is caused by the
detachment (embolization) of a clot that travels to the lungs. Together, DVT and
pulmonary embolism constitute a single disease process known as venous
thromboembolism.. A D-dimer test may be used to assist with excluding the
diagnosis (because of its high sensitivity) or to signal a need for further testing.
Diagnosis is most commonly done with ultrasound of the suspected veins.
Anticoagulation is the standard treatment; typical medications include a lowmolecular-weight heparin and a vitamin K antagonist. Wearing graduated
compression stockings appears to reduce the risk of post-thrombotic syndrome.

50

Like
The meningococcal rash :
The incubation period of meningococcal meningitis is 2-10 days the initial
symptoms are fever ,headache ,malaise ,myalgia and arthralgia witin hours ,
petechial , purpuric or ,morbiliform lesions may develop , fulminant
meningococcemia may occur with rapidly progressive purpura and shock
The appearance of the distinctive rash often one of the final symptoms of deadly
septicemia , means that immediate medical treatment is vital .
(q) .What causes the rash ? (A) This happens when the bacteria multiply in the
blood vessels and release toxins ,these damage the blood vessels so the blood
can leak through into the tissues underneath the skin it can start either as pink
rash or as tiny red or purple blood spots ,like pin pricks any where in the body
which rapidly spread into purple blotches or bruises often the early signs of the
rash can be mistaken for a common ailment, in the final critical stage it spreads
rapidly into purple bruises or hemorrhages which cover the body the patient can
go into shock , The fingers ,toes, and limbs amputation or death may be a result .
Treatment crystalline penicillin 12-20million units/day divided over 6 doses or
ceftriaxon 2gm/12 h for 1 week prophylaxis for contacts within 48 hours rimactan
600mg/12h for 2 days or ciprofloxacin 500 mg single oral dose or ceftriaxon 250
mg im single dose

o
Sad
Like

51

Oral leukoplakia
Leukoplakia is defined as a predominantly white lesion that can not be
characterized by any other definable lesion it can affect the buccal
mucosa (leukoplakia buccalis) or the tongue (leukoplakia lingualis) . .
leukoplakia may be confused with other common causes of white
patches in the mouth such as oral candidiasis or lichen planus . The
lesion of leukoplakia appear white or greyish patches that can not be
rubbed off as would be the case in pseudomembraneous candidiasis
(oral thrush) . Idiopathic leukoplakia more commonly occurs in
smokers , long term alcohol use and other chronic irritants also may
contribute to leukoplakia . Oral hairy leuloplakia : is a type of
leukoplakia primarily affect people whose immune systems has been
weakened by medications or disease especially HIV/AIDS , hairy
leukoplakia causes white patches that resemble folds or ridges on the
sides of the tongue . It may be one of the first signs of HIV infection , its
appearance on patients on antiretroviral therapy may be an indication
that antiretroviral therapy is failing . Hairy leukoplakia is not painful

52

Candida infection or thrush


Infection caused by candida fungus it can affect the mouth and other parts of the
body causing oral thrush, diaper rash in infants , or vaginal yeast infection in
women , Lesions in the mouth usually on the tongue or inner cheecks , throat ,
palate or tonsils in severe cases lesions may spread into the esophagus causing
painful swalling , fever if the infection spreads beyond the esophagus . thrush is
usually diagnosed by clinical examination , a common sign is the presence of
creamy white plaques covering all or part of the tongue, gingiva and oral mucosa
these plaques when removed leave a bright inflammed base . Thrush can be
caused by corticosteroids , antibiotics , birth control pills . It can also be a result
from illnesses or medical situations that disturb the immune system and make
candida infection more likely e.g. uncontrolled diabetes mellitus - cancer - HIV
infection - dry mouth - hormonal changes that occur with pregnancy . Treatment
depends on the severity and the underlying cause and include local or systemic
antifungal medications .

53

Hand Foot and mouth disease


Hand foot and mouth disease is caused by the Coxsackie virus . This relatively
common infection occurs most often in young children , It presents with a
vesicular rash affecting mainly the hands ,feet and mouth ulcers . Typically , the
palms of the hands and soles of the feet are affected . The patient may be febrile
and unwell .
Management :Symptomatic treatment only is required for this condition ,which
resolves spontaneously within about a week . Plenty of fluids and paracetamol
should be sufficient , but a soothing mouth wash may be helpful where mouth
ulcers are troublesome .
N.B. Foot-and-mouth disease (FMD) is a highly communicable viral disease
caused by an Aphthovirus of the family Picornaviridae. There are 7 serotypes , it
is limited to animals , it differs from hand foot and mouth disease,which is caused
by coxsackie virus occurs most commonly in young children

54

Impetigo
Usually caused by staphylococcus aureus or streptoccus pyogenes . It is
common and contagious . The patient presents with pustules and /or
blisters that break down and weep often leaving yellow crusts .
Management : Lesions should be bathed in an anti septic lotion .
Depending on the severity and extent , lesions should be treated with a
topical ( Fucidic acid or mupirocin) and / or systemic (flucloxacillin)
antibiotic for at least a week , It is recommended that school children
should be excluded from school until started on the appropriate
antibiotic therapy and that exposed lesions should be covered with a
water tight dressing ,if there seems to be a carrier site ( e.g. in the
nostrils ) an antibiotic nasal cream (mupirocin) should be applied to the
nostrils three times a day for a week .

55

Wow
Staphylococcal scalded skin syndrome : This syndrome is due to endotoxin
release from staphylococcus aureus , it usually occurs in under -fives who have
not developed protective antibodies against these endotoxins , The child
becomes ill with a fever , within 1-2 days fluid filled blisters develop that rupture
leaving denuded areas resembling a burn
Management : The child should be admitted to hospital for intravenous antibiotics
and skin care , and to maintain fluid and electrolyte balance , if correctly treated
the child should be well within a week .

Sa

56

Love

Angular cheilitis
Inflammation of one or both corners of the mouth , a condition
characterized by dry scaling and fissuring of the lips and angle of the
mouth . Angular cheilitis often represents an opportunistic infection of
fungi and / or bacteria with multiple local and systemic predisposing
factors . such factors include nutritional defeciencies mainly riboflavin
deficiency , dry mouth , lip licking habit , drooling , immunosuppression
and others such as wearing ill fitting dentures . Treatment :Varies based
on the exact cause in each case but often antifungal cream is used
among other measures .

57

Cold sores (Fever blisters)


Herpes labialis (cold sore) : is caused by the herpes simplex virus ,cold
sores commonly appear on the edges of the lip . this virus exists in the
dormant state in the spinal cord nerve cells ,and after certain
environmental triggers like a sun burn or cold the virus induced to
travel along a peripheral nerve to the same skin site over and over again
. The first symptoms of cold sores include pain around the mouth and on
the lips ,fever ,a sore throat or swollen gland . Small children sometimes
drool before cold sores appear .After blisters appear the cold sores
usually break ,open ,leak a clear fluid and then crust over and disappear
after several days . The eruption is self limited to about 7-10 days so
that treatment is unnecessary unless eruption becomes too frequent

58

Aphthous stomatitis or Recurrent mouth ulcers : The disease causes painful


,round ulcers to develop on the linings of the cheecks ,lips ,the tongue or the
base of the gums ,the tendency to develop these ulcers is inherited ,ulcers can
be associated with other diseases particularly connective tissue diseases such
as lupus or Behcet,s syndrome which cause symptoms on the eyes and genitals
as well as the mouth the ulcers can be single or multiple and they are recurrent
,multiple ulcers are scattered along the lining of the mouth ,not clustered , the
cause of recurrent ulcers is not known but most theories involve immune
abnormality ,certain blood diseases ,vitamin and mineral deficiencies ,allergies
,trauma and crohn,s disease cause similar ulcers ..
Recurrent aphthous stomatitis are classified into 3 categories :
(1)Minor ulcers : less than 1 cm and do not leave scar , usually heal within 2
weeks (2) Major ulcers : 1/2 inch or more in diameter take longer time than minor
ulcers to heal and may leave scar . (3) Herptiform ulcers ;
are clusters of dozen of small ulcers (this form is rare ).
people tend to have 2-6 ulcers per episode and have several episodes each year
. If the ulcers become more frequent or severe ,or accompanied by other
symptoms such as rashes ,joint pain ,fever or diarrhea or are larger than about
1/2 inch in diameter ( blood diseases ,drug reactions ,connective tissue disorders
and skin disorders should be considered..
Treatment : Correction of anaemia ,iron ,folate or B12 deficiency the ulcers show
improvement when these deficiencies are treated ..
Treatment focuses on reliving symptoms ,rinsing with warm water solution and
eating bland foods can minimize discomfort ,anesthetic medications or over the
counter agents that are placed directly on the sores to coat them also may help .
People with more severe disease may need steroid medications placed on the
lesion these may shorten the healing time of the ulcers and preventing them from
becoming larger

59

Acute herpetic gingivostomatitis


Also called acute infectious gingivostomatitis ,ulcerative stomatitis ,vincent
stomatitis , This primary infection probably the most common cause of stomatitis
in children 1-3 years of age , can also occur in older children and adults . The
symptoms may appear abruptly with pain in the mouth ,salivation ,fetor oris (bad
mouth odour) ,refusal to eat and fever ,often as high as 40-40.6 C. The onset
may be insidious , fever and irritability preceding the oral lesions by 1-2 days .
The initial lesion is vesicle which is seldom seen because of its early rupture .
The residual lesion is 2-10mm in diameter and is covered with a yellow grey
membrane ,when this membrane sloughs ,a true ulcer remains . Although the
tongue and cheecks are most commonly involved ,no part of the oral lining is
exempt . Acute gingivitis is characteristic of the disease and may precede the
appearance of mucosal vesicles .Submaxillary lymphadenitis is common . The
acute phase lasts 4-9 days and is self limited . pain tends to disappear 2-4 days
before healing of the ulcers is complete . In some instances the tonsillar regions
are involved early and acute tonsilitis of bacterial origin or herpangina may be
suspected .Failure of the lesion to respond to antibiotic therapy differeniates a
bacterial infection , and the spread of the vesiculation to the buccal mucosa rules
out herpangina .Recurrent aphthous ulcers are not caused by herpesvirus

60

Lichen planus
This presents as an itchy ,shiny ,violaceous flat topped ,papular rash with white
lines on the surface known as Wickham's striae . Other affected sites include
mucous membranes ,genitalia ,palms ,soles ,scalp ,and nails . Lichen planus
causes a white net -like pattern and sometimes ulceration on the buccal mucosa.
Causes of white lesions in the oral mucosa :
(1) Lichen planus
(2) Leukoplakia
(3) Chronic candidiasis
(4) Chemical burns

61
Recurrent superficial abscesses
You may be faced in your clinic with a patient presenting with recurrent abscess
formation : Careful clinical history and examination focuses the differential diagnosis and
guides appropriate investigations : (1) Age of onset : Paediatric onset may immediately
suggest phagocytic or other primary immune deficiency (2) Failure to respond as
expected to antimicrobial therapy raises the possiblity of immune deficiency .(3)Positive
family history and consangunity may suggest primary immune deficiency (4)The
infection site may give clues for example perianal and enterocuteanous abscesses
suggest underlying Chron's disease rather than primary immune deficiency
(5)Rash resembling atopic dermatitis associated with recurrent skin infection suggest
hyperimmunoglobulin E syndrome (IgE) (6)Underlying medical conditions with
associated higher rates of staphylococcal carriage like diabetes mellitus , chronic skin
diseases , cuteanous trauma , I.V. drug users , patients on hemodialysis and surgical
patients .secondary immune deficiency especially HIV infection is considered (7) Pattern
of organ involved e.g. skin , liver and bone in chronic granulomatous disease and
apocrine gland bearing skin in hideradenitis suppurativa (HS) affecting predominantly
the axilla , perineum and groin . (8) Organism : Invasive aspergillus infection in chronic
granulomatous disease , pseudomonas otitis externa in DM .
Lab. investigations : Full blood count : White cell differential you may find neutropenia or
lymphopenia , platlet count : you may found thrombocytopenia , Blood film you may find
abnormalities in neutrophil or platlet morphology and platlet size , Serum IgE : elevated
in atopic eczema , very high levels may suggest hyper IgE syndrome .CRP: evidence of
active infection or ongoing inflammation (IBD).- Blood glucose : for diabetes confirmation
.Today's photo (Case 186) : Recurrent superficial abscesses
You may be faced in your clinic with a patient presenting with recurrent abscess
formation : Careful clinical history and examination focuses the differential diagnosis and
guides appropriate investigations : (1) Age of onset : Paediatric onset may immediately
suggest phagocytic or other primary immune deficiency (2) Failure to respond as
expected to antimicrobial therapy raises the possiblity of immune deficiency .(3)Positive
family history and consangunity may suggest primary immune deficiency (4)The
infection site may give clues for example perianal and enterocuteanous abscesses
suggest underlying Chron's disease rather than primary immune deficiency
(5)Rash resembling atopic dermatitis associated with recurrent skin infection suggest
hyperimmunoglobulin E syndrome (IgE) (6)Underlying medical conditions with
associated higher rates of staphylococcal carriage like diabetes mellitus , chronic skin
diseases , cuteanous trauma , I.V. drug users , patients on hemodialysis and surgical
patients .secondary immune deficiency especially HIV infection is considered (7) Pattern
of organ involved e.g. skin , liver and bone in chronic granulomatous disease and
apocrine gland bearing skin in hideradenitis suppurativa (HS) affecting predominantly
the axilla , perineum and groin . (8) Organism : Invasive aspergillus infection in chronic
granulomatous disease , pseudomonas otitis externa in DM .
Lab. investigations : Full blood count : White cell differential you may find neutropenia or
lymphopenia , platlet count : you may found thrombocytopenia , Blood film you may find
abnormalities in neutrophil or platlet morphology and platlet size , Serum IgE : elevated
in atopic eczema , very high levels may suggest hyper IgE syndrome .CRP: evidence of
active infection or ongoing inflammation (IBD).- Blood glucose : for diabetes confirmation

62

Like

63

Intravenous drug misuse


Intravenous drug misuse typically leads to this appearance , which
results from repeated superficial thrombophlebitis of accessible veins in
the arm or elsewhere in the body . The sharing and reuse of syringes
and needles together with the lack of aseptic technique ,puts these
patients at special risk of wide range of infections including bacterial
septicemia (sometimes with unusual organisms ) systemic fungal
infection , hepatitis B , hepatitis C , and HIV infection . Right sided
endocarditis is a common complication

Love
Haha

64

Necrotizing fasciitis
This is an acute necrotizing infection of the skin ,subcuteanous tissues and
superficial fascia ,it may involve muscle . Clinically there is soft erythema or
dusky appearance with severe pain . Risk factors are diabetes , advanced age
,obesity and intercurrent illness . It is often a mixed infection with streptococcal
pyogenes ,anaerobes and coliforms , It requires aggressive surgical debirdement
as well as antibiotics (covering anaerobes , gram negatives , and streptococci
(Clindamycin may have benefit in inhibiting production of sterptococcal toxins )
Fournier's gangrene is a type of necrotizing fasciitis involving the male genitalia .

Like

65

Hydatid disease
Common in areas of sheep and cattle farming ,caused by Echinococcus
granulosus (a dog tapeworm) ,the parasite embryos hatch in the small intestine
and enter the blood stream via the portal venous circulation to the liver as a
result of eating contaminated vegetables ,there may be spread to the lung or
brain - Many cases are asymptomatic ,but right upper quadrant pain is the
commonest symptom ,Jaundice occurs if there is duct obstruction
Peritonitis will result from cyst rupture . Diagnosis : CBC: Eosinophilia
Haemagglutination test confirm the diagnosis . Ultrasound : Cystic lesions on
liver, or lung on Chest x-ray or brain on CT .
Active infection is treated with albendazole followed by surgical resection of the
intact cyst ,chronic calcified cysts can be left untreated .

66

Typhoid Fever (Enteric Fever)


Enteric fever is caused by salmonella typhi and salmonella paratyphi A,B,C.
Incubation period : 1-3 weeks . Transmission : by ingestion of contaminated food or
water . Clinical features : Fever ,myalgia ,cough ,constipation ,abdominal pain and
headache ,Features worsen with time and peak in the third week , deafness and
diarrhea can occur and there is classically described relative bradychardia
,hepatosplenomegaly and rose spots (pale pink blanching spots on the trunk ) may
be found , the tongue may be dry and coated complications include psychosis , or
altered mental state ,hepatitis ,cholecystitis ,pneumonia ,precarditis and meningitis .
Leucopenia is typical , Untreated ,mortality is from septicemia or GI perforation ,
Diagnosis : is made by culture of the organism from blood ,bone marrow , stool or
urine . Serology : Widal test is not useful

67

Rheumatic Fever
Follows a group A sterptococcal infection ,pancarditis usually occurs and valvular
defects are long term sequelae .
Criteria for diagnosis include : evidence of preceding streptococcal infection (raised
antistreptolysin O titre (ASOT) ,positive throat swab or history of scarlet fever )
together with two major or one major and two minor ( Dukette -jones criteria )
Major criteria : (1) Carditis (2) Polyarthritis (3)Chorea (4) Erythema marginatum
(5) Subcuteanous nodules Minor Criteria (1) Fever (2)Arthralgia (3) Previous
rheumatic fever (4)High ESR and Creactive protein CRP (5) Prolonged PR interval
on ECG

68

Arthritis and subcuteanous nodules in rheumatic fever: ( a) Arthritis : Painful,


migratory , short duration ,inflammed joints are warm ,red and swollen . Large
joints are involved , small joints and cervical spine are less commonly affected ,
Arthritis is the most common feature of acute rheumatic fever (80% of patients) .
Polyarthritis and Sydenham,s chorea never occur . Carditis and arthritis frequently
coexist .
Subcuteanous nodules : 0.5-2cm long , firm , painless , freely mobile under the skin .
Site ; over the extensor surfaces of joints (knees, elbows, wrists) ,also on bony
prominences , tendons, dorsi of feet, occiput or cervical spine . Last a few days only ,
occur in 9-20% of cases , often associated with carditis

69
Infective endocarditis Infective endocarditis (IE) is defined as an infection of the
endocardial surface of the heart , which may include one or more heart valves, the mural
endocardium, or a septal defect. Its intracardiac effects include severe valvular insufficiency,
which may lead to intractable congestive heart failure and myocardial abscesses. If left
untreated, IE is generally fatal.Signs and symptoms
Fever, possibly low-grade and intermittent, is present in 90% of patients with IE. Heart
murmurs are heard in approximately 85% of patients.
One or more classic signs of IE are found in as many as 50% of patients. They include the
following:
Petechiae: Common, but nonspecific, finding
Subungual (splinter) hemorrhages: Dark-red, linear lesions in the nail beds
Osler nodes: Tender subcutaneous nodules usually found on the distal pads of the digits,
Janeway lesions: Nontender maculae on the palms and soles
Roth spots: Retinal hemorrhages with small, clear centers; rare
Signs of neurologic disease, which occur in as many as 40% of patients, include the
following
Embolic stroke with focal neurologic deficits: The most common neurologic sign
Intracerebral hemorrhage
Multiple microabscesses
Other signs of IE include the following:
Splenomegaly,Delirium,Stiff neck,Paralysis, hemiparesis, aphasia
Conjunctival, hemorrhage,Pallor,Rales,Cardiac arrhythmia ,Pericardial rub,Pleural friction rub
Diagnosis
The Duke diagnostic criteria, developed by Durack and colleagues, are generally used to
make a definitive diagnosis of IE. The criteria combine the clinical, microbiologic, pathologic,
and echocardiographic characteristics of a specific case
Major blood culture criteria for IE include the following:
Two blood cultures positive for organisms typically found in patients with IE
Blood cultures persistently positive for one of these organisms, from cultures drawn more
than 12 hours apart
Three or more separate blood cultures drawn at least 1 hour apart
Major echocardiographic criteria include the following:
Echocardiogram positive for IE, documented by an oscillating intracardiac mass on a valve
or on supporting structures, in the path of regurgitant jets, or on implanted material, in the
absence of an alternative anatomic explanation
Myocardial abscess
Development of partial dehiscence of a prosthetic valve
New-onset valvular regurgitation
Minor criteria for IE include the following:
Predisposing heart condition or intravenous drug use
Fever of 38C (100.4F) or higher
Vascular phenomenon, including major arterial emboli, septic pulmonary infarcts, mycotic
aneurysm, intracranial hemorrhage, conjunctival hemorrhage, or Janeway lesions
Immunologic phenomenon such as glomerulonephritis, Osler nodes, Roth spots, and
rheumatoid factor
Positive blood culture results not meeting major criteria or serologic evidence of active
infection with an organism consistent with IE
Echocardiogram results consistent with IE but not meeting major echocardiographic criteria
A definitive clinical diagnosis can be made based on the following:
2 major criteria
1 major criterion and 3 minor criteria
5 minor criteria

70

71
Roth's spots
A hemorrhage in the retina with a white center , Originally associated with bacterial endocarditis ,
Roth spots may be seen in leukemia ,diabetes,collagen vascular diseases and other conditions that
predispose to hemorrhage in the retina .
Look for other features of infective endocarditis e.g petechial hemorrhages (in the conjunctiva or oral
mucosa) anemia splenomegaly ,Osler,s nodes ,splinter hemorrhages and Janeway lesions .
Diagnostic features and
investigations in adults with
Roth spots
History
* subacute bacterial endocarditis,
leukaemias: fever, night sweat, weight
loss
* anaemia, fatigue, shortness of breath
on exertion
* hypertension and diabetes
Examination
* subacute bacterial endocarditis: fever,
signs of anaemia, cardiac murmurs,
clubbing, splinter haemorrhages,
Osler's nodes
* leukaemias: hepatosplenomegaly, signs
of thrombocytopenia (mucosal and
skin petechiae and purpura)
* blood pressure
Laboratory investigations
* full blood count (FBC): anaemia,
thrombocytopenia
* peripheral blood film: may be
indicated if FBC is suggestive of
leukaemia
* erythrocyte sedimentation rate,
C-reactive protein: raised in subacute
infective endocarditis
* blood culture: if subacute bacterial endocarditis is suspected , random blood glucose : undiagnosed
diabetes mellitus.

72

Like

73
Splinter hemorrhage
Hemorrhages in the nail beds usually have a linear distribution near the distal end
,Remember these hemorrhages are not considered to be pathognomonic of infective
endocarditis ,they are thought to be caused by dissemination of tiny thrombi do not forget the
differniation of splinter hemorrhages from trichiniciasis , splinter hemorrhages are
longitudinal whereas in trichiniciasis the hemorrhages are always transverse .
Locally : Look very carefully so as not to miss these lesions , Look at the nails of the fingers
of the other hand as well .
Elswhere : Look for other features of infective endocarditis e.g petechial hemorrhages (in
conjunctiva or oral mucosa ) , anemia splenomegaly , Osler,s nodes ,Roth spots , Janeway
lesions etc.
Common associations :
Trauma especially in manual labourers .
Infective endocarditis .
Septicemia .

Lo

ve

HahWow

74

Janeway lesion
Non tender , small erythematous or hemorrhagic macular or nodular lesions on
the palms or soles ,only a few millimeters in diameter that are indicative of
infective endocarditis , Osler,s nodes and Janeway lesions are similar but Osler,s
nodes present with tenderness and are of immunologic origin . Locally Look for
other stigmata of infective endocarditis . Elswhere : Examine the heart for
murmurs , look for hepatosplenomegaly . Discussion : Janeway lesion is
described to be a microabscess of the dermis with marked necrotic and
inflammatory infiltrate not involving the epidermis , they are caused by septic
emboli which deposit bacteria forming microabscesses ,they are distal ,flat
ecchymotic and painless . (Q) What tests should be done ?
(A) (1) Multiple blood cultures (2) Other blood tests . (3) Urine tests
(4) ECG (5) Chest x-ray (6) Echocardiography .

Sad
...

75

Oslers nodes
Locally : Note Small tender slightly raised nodes which occur over the fingers or toes
,thenar and hypothenar eminences ,they are of bluish or pink in color ,These nodes last
for few days and are considered pathognomoic of subacute bacterial endocarditis .
Elswhere : Look for other features of infectuve endocarditis ,splinter femorrhages ,
petechial hemorrhages (in conjunctiva or oral mucosa ) ,anaemia ,splenomegaly ,Roth
spots , Janway lesions etc.
Auscultate the heart for cardiac murmurs .
. Discussion : Osler,s nodes are caused by immune complex deposition , the resulting
inflammatory response leads to swelling ,redness ,and pain that characterize these lesions
. Association : it can also be seen in : Systemic Lupus erythemasous , Endocarditis
,Disseminated gonococcal infection , Distal to infected arterial catheter .

76

Familial Mediterranean Fever (FMF)


FMF is a disorder passed down through families (inhirted) which involves
repeated fevers and inflammation that often affects the lining of the abdomen
,chest ,or joints . Causes : FMF is most often caused by mutation in the MEFV
gene . this gene creates proteins involved in inflammation . Symptoms usually
begin by ages 5-15 years ,repeated episodes of abdominal pain ,chest pain
sharp and gets worse when taking a breath ,fever or alternating chills with fever
that is usually episodic ,joint pain ,skin lesions that are red and swollen and
range from 5-20 cm in diameter .Investigations : no specific test to diagnose the
disease ,certain blood tests may be higher than normal when done during an
attack CBC,CRP,,ESR ,Fibrinogen test WBCs count . Treatment : no known cure
for FMF ,most people continue to have attacks but the number and severity is
different from person to person .
Colchicine is effective in preventing or minimizing attacks of FMF and preventing
the development of amyloidosis ,administer colchicine therapy daily 0.6mg bid in
patients at risk of developing amyloidosis if no response administer 3 or even 4
times a day ., in acute attack NSAIDs successfully control the attack .
In acute attack: the dose of colchicine is : 4 tab. Given 1 tab every hour. Then
2tab given 1tab every 2 hour then 1tab. Every12 hours

77

Heart Failure
Failure the pumping action of the heart to meet the circulatory demands of the
body (in the absence of mechanical obstructions). A broad echocardiographic
definition is of an ejection fraction less than 40% , Causes : Ischemic heart
disease- Valvular heart disease- Hypertension alone or in combination with IHDcardiomyopathy- Heart rhythm disturbance-sometimes anemia ,hyperthyroidism ,
or high pressure in the lung. What are the symptoms of heart failure?
The main symptoms are:
shortness of breath - when you are being active or at rest
swelling - of your feet, ankles, stomach and lower back areas
fatigue - feeling unusually tired or weak
Classification of heart failure (NYHA classification) :
Class i: asymptomatic with ordinary activities , Class 2: Slight limitation of
physical activities , Class 3: Marked limitation of physical activities , Class 4 :
Dyspnoeic symptoms at rest .
When symptoms suggest congestive heart failure, the diagnosis usually is
confirmed by , patient history,physical examination
During physical examination, look for an underlying cause and assesses heart
function. Auscltate to detect abnormal heart sounds (murmurs) that may indicate
a leaky or narrowed (stenotic) valve, and to detect fluid accumulation in the
lungs. The physician also looks for enlarged (distended) veins in the neck and for
swelling (edema) in the legs (particularly the ankles and feet) and/or the
abdomen.
A patient history may include information about the following:
History of hypertension, including treatment , history of diabetes
Prior chest pains or heart attack , history of rheumatic heart disease
Recent viral illness
Recent pregnancy,Alcohol and drug use
Diagnostic tests may include an electrocardiogram (ECG ), an echocardiogram
(cardiac echo), and cardiac catheterization. The purpose of these tests is to
evaluate heart function (e.g., assess ejection fraction), and to detect coronary
artery disease, heart attack, and valve dysfunction.

78

79
Coarctation of aorta
Locally : Like all other cardiovascular cases always start with examination of the pulse Systolic murmur over the precordium with or without cardiac enlargement . Look for visible
l palpable arterial pulsation on the back which is best seen with the patient leaning forward
.Elsewhere : Palpate both radial and femoral pulsations simultaneously , The femoral
pulsations are markedly diminished and delayed in coarctation of aorta when compared
with the radial pulses - Confirm by taking blood pressure both in upper and lower limbs :
the readings will be much lower in the lower limbs . Discussion : The diagnosis of this
tricky case can be easily missed unless you keep a strict practice of palpating the femoral
pulsations along with the radial arteries on both sides . Coarctation is much more common
in men ,and in women with Turner's syndrome . The possibility of coarctation should be
considered in any hypertensive patient under the age of forty-five . The usual causes of
death in untreated cases are bacterial endocarditis and ruptured cerebral aneurysm
Questions : (Q)What single most important investigation would you ask for ?
(A) CXR (Look for rib notching)
(Q)What are other causes of rib notching ?
(A) (1) Neurofibromatosis .(2) Inferior vena cava obstruction .
(Q)What are the four signs commonly seen in the CXR?
(A) (1) Notching of the ribs (2) Dilated ascending aorta (3) Figures of "3" sign due to
indentation of the aorta at the site of coarctation with pre and post stenotic diltation
(4) Dilated left subclavian artery . Examination by continuous wave Doppler
echocardiography is an effective non-invasive method of assessing the severity of
coarctation of the aorta,ejection fraction), and to detect coronary artery disease, heart
attack, and valve dysfunction.

80

Atrial septal defect


Locally : The apex beat is not usually displaced but in some cases there may be
a left parasternal heave due to right ventricular enlargement - Listen carefully for
a midsystolic murmur left of the sternum maximal in the 3rd intercostal space
with wide splitting of the second heart sound which is relatively fixed in relation to
respiration . - Elsewhere : Examine the radial pulse as usual (usually large
volume pulse ) - Look for dyspnea , cyanosis ,raised JVP. liver enlargement and
edema of the feet , signs of heart failure in cases of ASD with reversed shunts . Look for any signs of infective endocarditis (rare in cases of ASD) e.g. clubbing
of the nails , splinter hemorrhages , anemia and splenomegaly ,etc . Question :
What three investigations would to ask for ?
(A) (1) ECG: incomplete right bundle branch block is quite common in ostium
secundum defects ,whereas in primum defects it may be left bundle branch block
.
(2) Chest x-ray : cardiomegaly with pulmonary plethora (hilar dance ) .
(3) Cardiac catheterization .(4) Echocardiography

81

Ventricular septal defect


Locally : The apex beat is usually not displaced - Palpate for a systolic thrill
maximal in the 4th intercostal space to the left of the sternum - On auscultation
you would expect to hear a pansystolic murmur best heared in the 3rd or 4th left
intercostal space radiating over the precordium .- Also listen carefully for the
presence of a third heart sound which is often present and is due to rapid filling of
the left ventricle - Listen for the presence of any pulmonary systolic or mitral
diastolic murmurs which may be seen with larger septal defects where the output
of the right ventricle may be even double that of the left ventricle into the aorta .
Elsewhere : Examine the radial pulse as for other cardiovascular cases - Look for
central cyanosis which is seen in cases of VSD with reversed shunts
(Eisenmenger;s syndrome ) - Look for any evidence of congestive cardiac failure
e.g. raised JVP, liver enlargement or edema of the feet .- Look for any evidence
of infective endocarditis e.g. clubbing , splinter hemorrhages , anemia ,pyrexia
and splenomegaly etc. Questions : What is Maladie de Roger?
(A) When VSD is so small that is asymptomatic , and not accompanied by any
change in the ECG or chest x-ray it is called Maladie de Roger . A small VSD
gives a loud murmur and with larger shunts the murmur may become less
pronunced . (Q) what are the indication and contraindication for surgery in VSD ?
(A) Indications : (1) If right ventricular systolic pressure exceeds 50 mmHg (2)
Pulmonary to systemic flow ratio in excess of 1.5 to 1.7
Contraindication : Eisenmenger's syndrome .

82

Aortic incompetence
Locally : The cardinal sign is the presence of an early diastolic murmur best heared at
the left sternal edge (right sternal edge in Marfan's syndrome and syphilitic aortitis )with
the patient sitting up and holding his breath at the height of expiration - signs of
associated valvular lesions. Elsewhere : as in all cardiovascular cases always start with
the pulse - volume ,regularity and character e.g. collapsing ( water hammer ) pulse . But
do remember that the absence of collapsing pulse does not rule out aortic incompetence
. A pulse with double peak is called pulsus bisferiens , and is due to combined aortic
stenosis and incompetence - Collapsing pulsation in the neck arising from the cartoid
artries ( Corrigan's sign ) - Look for other stigmata of syphilis ,such as Argyll Robertson
pupil and ptosis Look for signs of left ventricular failure Discussion : Many patients with
aortic incompetence are asymptomatic for decades , with progressive regurgitation left
ventricular diltation and evantually left ventricular failure ensues . Some other signs of
aortic incompetence such as pistol shots (sounds auscultated over femoral artery ) and
Duroziez murmurs (to and fro murmur audible on slight compression of femoral artery )
are less important and do not contribute much to the diagnosis
Questions : (Q)What are causes of aortic incompetence ?
(A) (1) Rheumatic heart disease (2)Syphilis (Ankylosing spondylitis (3)Marfan's
syndrome (4) Infective endocarditis
(Q)What are the causes of water hammer pulse ?
(A) (1)Anemia (2) Thyrotoxicosis (3) Paget's disease of bone ( 4) Pregnancy

83

Aortic stenosis
Locally
Ejection mid systolic murmur heared loudest at the base ,conducted to the right
side of the neck , the second aortic sound is absent or diminished - Heaving
displaced apex beat suggestive of left ventricular hypertrophy . Also a systolic
thrill may be present - Any other associated valvular lesion . Elsewhere : Like
other cardiovascular cases ,peripheral signs are very important , Start by
examining the pulse - volume -regularity and character . The pulse is of small
amplitude and rises slowly to a delayed peak with a slow fall . There may be an
extra impulse felt at the peak and then it is called anacrotic pulse , Look for signs
of left ventricular failure Discussion : Aortic sclerosis due to atherosclerotic
hardening of the cusps of the aortic valve without gross narrowing ,is a common
condition and should be differentiated from true aortic stenosis , The pulse
remains of normal volume and character in aortic sclerosis ,whereas is anacrotic
in aortic stenosis .
Questions: (Q) How can the patient present ?
(A) (1) Angina (2) Syncopal attacks (3) Clinical features of left ventricular failure
(Q) What are the common causes of aortic stenosis ?
(A) (1)Rheumatic heart disease
(2) In elderly males it is usually due to calcification of congenital bicuspid aortic
valve .

84

Acanthosis nigricans
It is a skin condition that causes one or more areas of skin to darken and
thickened .
The affected skin often feels velvety . The skin changes typically occur in people
who are obese or have diabetes . Children who develop the condition are at
higher risk of developing type 2 diabetes because it has been associated with
insulin resistance . More rarely acanthosis can be a warning sign of a cancerous
tumour in an internal organ such as the stomach ,colon or liver .It often occur in
people who have hormonal disorders such as ovarian cyst ,hypothyroidism or
disorders of the adrenal gland .Medications such as oral contraceptives and
corticosteroids as well as high doses of niacin may also cause acanthosis
nigrican

85

Xanthelasma
Deposits of lipids predominantly cholestrol in the dermis or subcuteanous tissue
around the eyelids are called xanthelasma ,
Look at the face of the patient , Note the following features :
- Raised ,yellow plaques above the eyelids near the inner canthus .
- Look for xanthomas elsewhere in the body e,g. palms , knuckles
,tendoachilles,etc. - Look for jaundice (obstrutive jaundice ).
- Does the patient look myxoedematous ?
-Does the patient have swelling of the eyelids and legs (nephrotic syndrome ) ?
Causes : Various conditions associated with high serum cholestrol level e.g. (1)
Familial hypercholestrolemia .(2) prolonged obstructive jaundice (3) myxoedema
,(4) nephrotic syndrome (5) Diabetes mellitus .(Q) What are the 3 main types of
xanthomas ?
(1) Eruptive xanthomas : seen as firm raised papules with pale yellow centres
over the buttocks ,elbows ,knees and dorsum of the arms (2) planar or palmar
xanthomas : seen in the palmar and digital creases
(3) Tendon xanthomas : occur in the extensor tendons at the back of the hands
and the achilles or the patellar tendons

86

Necrobiosis lipoidica diabeticorum


It occurs in only 1% of diabetics , the etiology is uknown
Notice irregular discoloured patches in the ankles and shin areas of the legs , the
condition presents as flat reddish brown patches which enlarge and eventually
become yellowish , blood vessels can sometimes be seen through the skin , the
skin becomes shiny and thin , the most common region affected is the pretibial
area , but lesions can present elsewhere , for example the forehead , if
traumatized , the lesion may ulcerate
treatment of necrobiosis lipoidica is unsatisfactory , locl trauma must be avoided
and the legs protected from injury . Elastic stockings and leg rest may be helpful .
Topical or intralesional steroids may help to reduce inflammation in the early
active phases ,but caution should be taken when used on atrophic lesions ,
aspirin and dipyridamole have been tried in an attempt to prolong platlet survival
time , pentoxyphylline has also been tried , these preprations seem to offer some
benefits , as do psoralen with UV light therapy , surgery with exision of the lesion
and skin graft can be considered

87

Granuloma annulare
It is a relatively common benign inflammatory dermatoses that occurs in all age
groups . It is characterized clinically by dermal papules and annular plaques . In
most cases it is not itchy or painful .Note the ring of flesh coloured papules . This
condition may occur in otherwise healthy individuals.This condition may occur in
otherwise healthy individual or in patients with diabetes .

88

Kaposi sarcoma
Kaposi sarcoma is a tumour caused by a human herpes virus 8 , it became widely known as
one of the AIDS defining illnesses .
Presentation : History : months to years , Note, Cuteanous : Purple non pruritic papules any
where on the skin but specially nose ,legs and genitals ,crease line distribution over
trunk,satellite lesions ,bruising ,local lymphadenopathy and edema , typical oral and GI tract
purple raised lesions ,favoured sites palate ,gums . Ask for pulmonary symptoms ,cough
,hemoptysis ,fever ,chest pain , Look for breathlesness .Examine the abdomen for
hepatosplenomegay . Suspect if : Raised purple spots on skin ,in mouth, or on endoscopy
/bronchoscopy. (Q) What are the complications of kaposi sarcoma ? (A)(1) Ulceration and
chronic lymphoedema from skin lesions .
(2) Anaemia and bleeding from the GI tract.
(Q) What investigations would you ask for?
(A) (1) Laboratory studies : CD4 lymphocyte count and plasma HIV viral load studies for
suspected HIV infection .
(2)Chest x-ray : typically disease affects middle and lower zones with patchy course reticulo
nodular shadowing and mediastinal lymphadenopathy ,pleural effusion in 25% . (3)
CT,Bronchoscopy ,and endoscopy if indicated by symptoms .(4) Skin biopsy for single or
atypical lesions . Management: Cuteanous/oral : First line : commence HAART (Highly
Active Anti Retroviral Therapy) (75% response )
Radiotherapy : valuable for localized where lymphoedema is prominent or there is mass
effect .
Visceral ,widespread or HAART unresponsive mucocuteanous : First line :cyclical liposomal
doxorubicin ,Paclitaxel for refractory or relapsed disease

89
Hodgkin's disease or Hodgkin's lymphoma
It is a cancer of lymphatic system ,the cause is not known , it is most common among people
ages 15-35 years and 50-70 years , Past infection with EBV contribute to some cases .
Diagnosis : Ask about history of fever and fatigue, presence of swollen glands in the neck
and groins(lymph nodes), ask about soaking night sweats and loss of wieght and appetite
,presence of cough ,chest pain or breathing problems ,ask about pain or feeling fullness
below the ribs due to enlarged liver or spleen . Examination : Look for the skin for itching
marks , bruising or purpuric spots . - Examine for swollen lymph nodes including the neck
and groin (painless, firm, rubbery and movable) ,examine the chest for any breathing
problems if there are swollen lymph nodes in the chest , Examine the abdomen for
hepatomegaly ,splenomegaly or both. .(Q) What investigations should be done ?
(A) (1) Biopsy of lymph nodes usually diagnose Hodgkin's ( Reed-Sternberg cells which are
large cells with large pale nuclei containing large purple nucleoli are indicative of Hodgkin's
disease .
(2) CBC: ,ESR (3) Biochemistry including protein levels , Liver function tests, kidney function
tests and uric acid level . (4) Bone marrow biopsy (in certain cases) . (5).X-Ray chest , CT
scan of the chest ,abdomen and pelvis. (6) Position Emission tomograhy (PET) : A type of
scan that measures the activity of cells in different parts of the body and check the spread of
the cancer and the imact of treatment .

90

Osteoporosis
Definition : Osteoporosis means bones that are full of holes ,it can develop
without symptoms until bone fractures occur . Risk factors : - Family history of
osteoporosis - Personal history of fracture after age 45 . -Prolonged use of
steroids .Post menopausal and not on estrogen replacement therapy . Early
warning signs : history of broken bones - unexplained back pain - loss of more
than one inch in height - posture has become stooped .,there is no relation
between osteoporosis and arthritis ,using steroids for treatment of arthritis may
put the patient at risk of osteoporosis .
What is DXA (Dual energy x-ray absorptiometry) ?
Is the gold standard for diagnosis of osteoprosis , it measures bone density in the
hip ,spine and forearm ,it is painless and ,non invasive ,
WHO T-Score is comparing the average bone density to that of healthy young
woman . 0-1 (normal ) -1 - -2.5 (osteopenic ,low bone mass
-2.5 or lower (osteoporotic)

91
Rickets
Definition : Softening and weakening of bones in children usually because of an extreme and
prolonged vitamin D deficiency which makes it difficult to maintain normal calcium and
phosphorus levels in bones which can cause rickets . Note : Impaired growth ,Short stature (
adults les than 5 feet) . ,Large skull , Bow legs ,Bumps in the rib cage (rachitic rosary) ,
pigeon chest , spine deformities (spine curve abnormalities including scoliosis or kyphosis) ,
Dental deformities , delayed formation of teeth or defects in the structure of teeth ,dental
caries . Locally you may find tenderness in the bones rather than the joints or muscles
,muscle cramps (Q) What investigations would ask for ? ( A) Serum calcium,
phosphorus,alkaline phosphatase , serum parathyroid hormone ( PTH), serum 25(OH) Vit D
level . Bone x-rays . (Q) What is the prognosis of rickets ? (A) If rickets is not corrected while
the child is still growing ,skeletal deformities and short stature may be permenant , while if it
is corrected while the child is young ,skeletal deformities often improve and disappear with
time

92

Vitamin A deficiency
Deficiency of vitamin A is found among malnourished ,elderly and chronically
population . Abnormal visual adaptation to darkness (night blindness) ,dry skin ,
pruritus,dry hair , xerophthalmia (dry eyes) broken finger nails and decreased
resistance to infections are among the first signs of vitamin deficiency . Bitot
spots these may be single or multiple and they represent areas of desquamated
kertanized conjunctival cells together with lipid material . Blindness due to retinal
injury . The WHO recommend vitamin A administration to all children
( especially those younger than 2 years ) who are diagnosed with measles as
coexistent vitamin A deficiency in young children increases the risk of death .
200,000 IU of vitamin A for at least 2 days reduces mortality rates .

93

Osteogenesis imperfecta
It is an autosomal dominant disease characterized by excessive bone fragility
,multiple fractures (healing with angulated deformities ) and blue sclera

...

94

Turner and Pseudo Turner (Noonan syndrome) : Turner syndrome is the most
common cause of primary ovarian failure , The chromosomal anomaly is the
presence of only one x chrosome in all cells , Look at the patient and notice : - Short
stature of the patient , Look for webbing of the neck -Look for shield chest -broad
chest with widely apart nipples and poorly developed breasts , Look for signs of
coarctation of aorta , Look for short metacarpals(Q) What single investigation would
you ask for ? Cytogenetic analysis of buccal mucosa ( absent chromatin bodies in
most cases
Noonan Syndrome :Autosomal dominant condition : Notice ,Low set and posterior
rotated ears , webbed short neck, pectus excavatum , wide spaced nipples cubitus
valgus ,short stature , small genitalia ,and undescended testicles, Cardiac : : cardiac
valve stenosis ,hypertrophic cardiomypotha y septal defects.

95

Mongolism
Note the small stature of the patient , Note that the skull is small ,and the face is
expressionless ,and the patient appears to be mentally retarded .
Note that the eyes are slanted and epicanthal folds are usually present .
There may be a convergent squint and in the first year of life , ther may be
Brushfield,s spots seen around the periphery of the iris , Note that the bridge of
the nose is flattened and the tongue is large and protruding .
Note that the hands are short and broad with single palmar crease (simian
crease ) and the fingers are small , the little finger is incurved , with an ulnar
convexity and with an abscent second phalanax .
Now examine the heart for any congenital heart disease e.g. atrioventricular
septal defect which are present in up to 30% of mongols .
(Q) What is the genetic abnormality in these patients ?
(A) In most cases , it is a trisomy of 21 i,e these patients have 47 chromosomes
instead of the normal 46 , In some cases , it is due to mosaicism or translocation
of the chromosomes

96

Achondroplasia
Note the small stature of the patient with gross shortening of all four limbs but
with a normal trunk length - Note the skull appears to be enlarged but this is in
fact due to the abnormally short stature of the person and not an actual
enlargement of the skull - Note that the forehead is rather prominent and bulging
with depressed bridge of the nose - Ask the patient to stand and note some
degree of dorsal kyphosis with compensatory posterior rotation of the sacrum
and pelvis - Note that the hands are small with all fingers being almost equal
length . Questions : (Q) What is the inheritance of this disorder
(A) Autosomal dominant
(Q)What is the differential diagnosis? (A) (1)Hydrocephalus (2) Paget's disease
of bone ?(3) Rickets .
(Q)Does the condition affect the intelligence of the patient ?
(A) No - they have normal intelligence ?

97

Marfan syndrome
It is a connective tissue disorder inherited as autosomal dominant trait resulting in
skeletal , ocular, and cardiovascular abnormalities
Locally : Look for the long spider like fingers (arachnodactyly) , patient is taller than
average with an arm span greater than height
Elswhere : check for hyperextensibility of joints , flat feet and kyphoscoliosis , chest wall
deformities e.g. pectus excavatum , high arched palate which leads to crowded front
teeth ,look for umbilical hernia and stretch marks of skin . Ocular ; high myopia is
common , lens dislocation and premature cataract .Cardiovascular system : diltation of
the ascending aorta with or without aortic regurgitation , mitral valve prolapse with or
without mitral regurgitation. Aortic dissection and aortic insufficiency are responsible
for 90% of morbidity and mortality ,average age of death is 30-40 years , Differnitial
diagnosis : Homocystinuria because of similar skeletal, ocular and cardiovascular
abnormalities but can be differniated by downward disclocation of the lens which is
upward in Marfan syndrome and also by mental retardation and finding homocysteine
in urine .

98

99
Rheumatoid arthritis
What does the examination of rheumatoid hand find ? (a) Early ; synovitis Late : ulnar
deviation , swan neck and boutonniere deformities and nodules on bony prominences and
extensor surfaces . Local examination ; Note the swelling with or without ulnar deviation of
the metacarpophalyngeal (MCP) joints , Note that only the proximal interphalyngeal joints
(not the distal ones ) are involved in rheumatoid arthritis , Look for swan neck deformity and
boutonniere deformity
Elswhere : Look for rheumatoid nodules , if present, aroundthe elbow joints on the extensor
, check for evidence of arthropathy in other joints like knees and elbows , check for vasculitis
lesions in the nail folds
(q) What investigations would you ask for ?
(a) (1) X-ray of both hands . (2) latex rheumatoid factor
(3) Anti-CCP which stands for anti-cyclic citrullinated peptide antibody is a very useful test to
order during the diagnostic evaluation of a person who may have rheumatoid arthritis. If
present in such a patient at a moderate to high level, it not only confirms the diagnosis but
also may indicate that the patient is at increased risk for damage to the joints. Low levels of
this antibody are less significant. In the past, doctors relied on another antibody, the
rheumatoid factor (RF) to help confirm a diagnosis.

100

Septic arthritis
septic arthritis refer to sudden onset of acute arthritis ,monoarticular ,most often in large
weight bearing joints and wrists , previous joint damage or injection drug abuse are common
risk factors , Infection with causative organisms commonly found elsewhere in the body .
Joint effusions are usually large with white blood count >50,000 /mcl . Imaging : Radiographs
are usually n0rmal early in the disease but evidence of demeneralization may be present
within days of onset - Bony erosions and narrowing of the joint space followed by
osteomyelitis and periostitis may be seen within 2weeks (Q)What are the three differential
diagnosis ?
(A) (1) Gout (2) L ocal cellulitis (3) Simple trauma

101

Reiter's disease and reactive arthritis


Reiter's disease : Classically this is the triad of non specific uretheritis
,conjunctivitis and arthritis( monoarthritis of the knee or asymmetrical
inflammatory arthritis of some interphalyngeal joints) that follows bacterial
dysentery or exposure to sexually transmitted infection , Incomplete forms are
frequent and include the commonest variety of inflammatory arthritis seen in
young men . The term reactive arthritis is frequently used When arthritis alone
follows sexual exposure or enteric infection .
The bacteria implicated in reactive arthritis are : salmonella ,shigella
,campylobacter ,yarsenia and chlamydia . Investigations : ESR often greatly
raised during the acute phase , polymoprhonuclear leucocytosis and anemia of
chronic disease are indication of active systemic disturbance

102

Psoraitic arthritis
Look for pitting of the nails ,Remember that psoraitic arthropathy is seronegative
and characterstically involves the terminal interphalyngeal joints with absence of
subcutaneous nodules ,psoraisis cause a unique type of swelling (sausage like
swelling ) ,Ask about pain in the bottom of the foot (plantar faciitis and achilis
tendinitis (pain in the heel),ask about lower back pain (due to spondylitis ),
morning stiffness may be present in both rheumatoid arthritis and psoraitic
arthritis . Look for signs of anemia which may be present .
Look for nail problems including pitting or separation from the nail bed .
Look for skin lesions : red scaly rash , Ask for fatigue and sense of tirdness due
to pain and inflammation . Check for the range of movement of the fingers , you
may find reduced range of motion as a result of pain ,stiffness and swelling of the
joints . Examine the eyes for signs of inflammation , conjunctivitis ,dry eyes and
lid swelling .
(Q) What are the spondyloarthropathies ?
(A) These are group disorders characterized by seronegative (i.e rheumatoid
factor negative ) inflammatory arthritis and /or spondylitis ,the peripheral arthritis
is typically asymmetric involving large joints especially the knees and ankles
,characteristic musculoskeletal features include enthesitis (inflammation at sites
of tendon insertion ),sacroiliitis ,these arthropathies should not be confused with
seronegative RA , which is a symmetric small joint arthritis

103

Swelling of the phalanges


Locally : Note the swelling is in the phalanges and not in the joints .
Look at the other hand as well for similar involvement of the phalanges .
Elswhere : Look at the face for any evidence of lupus pernio ,a bluish
discoloration of the nose seen in cases of sarcoidosis .
Common causes : Sarcoidosis - Sickle cell disease - Chronic infection of the
bone . (Q) What investigations could you ask for ?
(A) x-ray of the chest and hands .

104

Osteoarthritis
The most common joint disease characterized by softening and degradation of
articular cartilage with secondary changes in adjacent bone . Notice that the
patient is commonly obese and elderly . Look for the distal interphalangeal joints
for the presence of Heberden's nodes ., Look for the proximal interphalangeal
joints for the presence of (Bouchard's nodes ). Examine the other joints that can
be affected : base of the thumb , hips,knees and spine ,unlike rheumatoid
arthritis the joints do not typically become hot or red , look for pain and
tenderness in the joints affected , the pain is usually described as sharp or
burning sensation in the associated muscles and tendons , occasionally there is
joint effusion .
(Q) What are the x-ray findings in OA?
(A) The typical changes are narrowing of the joint space , subchondral sclerosis
(increased bone formation around the joint) and osteophytes.

105

Heberdens nodes
Heberden,s nodes are osteophytes at the distal interphalyngeal joints
Locally : Look for similar bony swellings at the proximal interphalyngeal joints
(Bouchard,s nodes ) , and metacarpophalyngeal joints especially the first( thumb
) metacarpophalyngeal joint . - Note that the patient is usually obese and elderly .
Elswhere : Look for evidence of osteoarthritis in other joints ,especially the knees
and the hips . (Q) What single investigation would ask for ?
(A) X-ray hands.

106

Systemic lupus erythematosus


Systemic lupus erythematosus is an inflammatory connective tissue disorder of
uknown aetiology occuring predominantly in young women . Various systems of
the body are affected , the clinical features include ,facial erythema , purpura ,
alopecia ,photosenstivity , arthritis , pleuritis , pericarditis , renal involvement and
psychosis .
Note the following features : An erythematous lesion over the bridge of the nose
extending to both cheeks which looks like a butterfly .
Look for area of scarring ,telangiectasia ,keratotic plugging ,hypopigmentation( in
the center ) and hyperpigmentation (at the edges) in the butterfly lesion , Look for
alopecia over the scalp .
In the hands look for evidence of arthropathy usually involving the
metacarpophalyngeal (MCP) and proximal interphalyngeal joints , Ulnar deviation
of the fingers and subluxation of the proximal interphalyngeal joints may be seen
without x-ray evidence of erosion .
Look for evidence of pleurisy with or without pleural effusion
Look for pericardial rub and heart murmurs .
(Q) What investigations would ask for ?
(A) (1) Full blood count including platlet count and ESR.
(2) Anti nuclear factor , Anti double stranded DNA.
(3) LE cells , (4) Urine for protein and casts
(5) Low C3 and C4 suggest lupus nephritis .
(6) Chest x-ray .
(Q) What are the American Association Criteria of Rheumatology For
diagnosis of SLE?
(A) Eleven criteria presence of 4 or more criteria diagnose SLE:
1- Malar rash 2- Arthritis 3- Discoid rash 4- photosensetivity 5-Serositis
6- Hematologic disorders 7- Neurologic disorders 8-Renal disorders
9- Immunologic disorders 10- Oral ulcers 11- Positive ANA .

107

108

Malar rash
Definition: Malar rash[: an erythema over the cheeks and nasal bridge (but
sparing the nasolabial folds, which is in contrast to the rash of dermatomyositis) .
It lasts from days to weeks and is occasionally painful or pruritic.it is one of the
cuteanous manifestations of Systemic lupus erythematosus ,note also the patient
is female and young .(Q) What are the 3 cuteanous manifestations of SLE?
(A) (1) Discoid lupus (2) Malar rash (3) Photosenstivity.
(Q) What is meant by photosenstivity?
(A) It is unusual rash or symptom exacerbation after sun exposure .

109

Discoid lupus
Discoid lupus is a chronic lupus rash , discoid lesions develop in sun exposed
areas it is erythematous lesion ,plaque like in character with follicular plugging
and scarring , there is excessive scaling , atrophy , hypo -and hyperpigmentation,
lesions subside and leave deep scars behind the face, neck and arms . The
scalp also may be involved ,they may be part of systemic lupus ( one of the 3
cuteanous manifestations of SLE ) or may represent discoid lupus without organ
involvement , Discoid lesions can develop in up to 25% of patients with SLE
,while there is 5 % risk of discoid lupus disease developing into systemic
condition
(Q) Name 3 drugs which cause SLE -like syndrome ?
1- Hydralazine 2- phenytoin 3- procainamide
(Q) Name 3 drugs which exacerbate SLE ?
1- Penicillin 2- sulphonamides 3- Oral contraceptives

110

Sjogren,s syndrome
A connective tissue disorder characterized by lymphocytic infiltration of exocrine
glands especially lacrimal and salivary glands . The reduced secretions lead to
dry eyes and dry mouth of the sicca syndrome and either rheumatoid arthritis or
a connective tissue disorder . About 30% of rheumatoid patients have secondary
sjogren,s yndrome .
Clinical features : (1) Dryness from atrophy of exocrine glands :
Eyes (xerophthalmia ) which may lead to corneal ulceration .
Mouth (xerostomia) with increased dental caries . -Respiratory ; with hoarsness
,dysphagia ,respiratory infections. -Vaginal ,causing dyspareunia .(2) Arthralgia
or arthritis : can be erosive .
(3) Raynaud,s phenomenon . (4) Lymphadenopathy . (5) Gland swelling :
In the early stages (e.g. parotid ) (6) Vasculitic purpura . (7) Neuropathies
(8) Renal tubular acidosis (30%) (9) Pancreatitis .
(Q) What investigations would you ask for ?
(A) (1) CBC : Anaemia and leucopenia are common.
(2) ANA : frequently present .
(3) Anti-Ro or anti-La present in primary Sjogren,s syndrome .
(4) ESR and CRP reflect disease activity
(5) Rheumatoid factor positive in most cases .
(6) Polyclonal hypergammaglobulinaemia .
(Q) What is the treatment of Sjogren,s Syndrome?
(A) - Artificial tears , Moistening sprays for the mouth ., NSAIDS and sometimes
hydroxychloroquine for arthritis

111

112

Behcet's syndrome
Major Criteria are : (1) Recurrent aphthous stomatitis (2) skin lesions (3) iritis (4)
genital ulceration . Minor Criteria : (1) Inflammatory arthritis of large joints (2)
Intestinal ulcerations ,( 3) Meningoencephalitis (4) Epididymitis (5)
Thromophlebitis
in the presence of all 4 major criteria ,the syndrome is said to be complete ,in the
presence of three ( incomplete ), The arthritis is monoarticular or oligoarticular
and non erosive ,it most frequently involves the knees ,ankles and elbows
,occasionally the sacro iliac joints are involved .

113

Still's disease: Still's disease is a form of arthritis characterized by high spiking


fevers, salmon-coloured rashes and inflammation of the joints. The disease is
most common among children, for whom it is commonly referred to as systemic
juvenile idiopathic arthritis.
Still's disease can also occur among adults, although much less commonly than
for children. In this case it is referred to as adult-onset Still's disease or AOSD.
(Q) What are the warning signs of Still's disease? Still's disease usually
begins with symptoms that affect the whole body (systemic symptoms). People
with the disease often feel very tired and experience daily fevers of 40C or
higher. These fever "spikes" usually occur at about the same time every
day (late afternoons and evenings) and always go away after a while. A faint
salmon-coloured skin rash also appears from time to time, usually at the height
of the fevers. The rash does not itch.Poor appetite, nausea, and weight loss are
common for people with Still's disease. Often there is swelling of the lymph
glands, enlargement of the spleen and liver, and sore throat. Swelling of the
joints usually starts after rash and fevers have been present for some time. This
usually involves many joints (polyarticular arthritis). Everyone with Still's disease
eventually develops joint pain and swelling.
History : Ask about daily fever,rash ,joint pain,myalgia,sore throat and enlarged
lymh nodes ,ask about symptoms of differential diagnosis including : infection:
diarrhea,uretheritis , Antecedent drug exosures,muscle weakness,easy
bleeding,or bruising ,swelling of feet or legs . Examination: careful skin
examination preferably during the patient febrile period ,evaluation for
splenomegaly,hepatomegaly,and thorough examination for lymphadenopathy
,joint examination of the upper and lower extremities and spine . Investigations :
CBC : Leucocytosis and high ESR, Rheumatoid factor and ANA are usually
negative , serum ferritin is markedly elevated,abnormal liver function tests .
Treatment according to severity of the disease ,in mild disease (fever ,rash as
well as arthritis) some atients may resond to NSAIDs alone but most of them
require at least low dose of glucocorticoids . Moderate disease Patients with
moderate disease may experience high fevers, debilitating joint symptoms, or
evidence of internal organ involvement that is not life-threatening or severe. Initial
treatment of such patients generally requires glucocorticoids to control the
inflammatory response and disease manifestations. Nonbiologic or biologic
disease-modifying antirheumatic drugs (DMARDs) may be needed for long-term
management, prevention of joint and other organ injury, and treatment of
refractory inflammatory manifestationsSevere disease Severe disease is
defined by the presence of life-threatening organ involvement and/or conditions,
such as severe hepatic involvement, cardiac tamponade, and/or disseminated
intravascular coagulation. Such patients require high-dose or pulse glucocorticoid
therapy and should receive early intervention with a biologic agent, such as an
interleukin (IL)-1 or IL-6 inhibitor.

114

Salmon like rash in Stills disease

115

Dermatomyositis
The main complaint of this patient is weakness of the proximal muscles
Look carefully for the presence of any skin rash over face , shoulders or the arms
(Heliotrope) rash with suffusion of the eyelids may be diagnostic of this disorder ,
Look for any wasting of the proximal muscles ,present only in the late stages ,
Test for the strength of these muscles . Weakness may be striking at the hips or
shoulders , Look for any areas of muscle tenderness by palpating the various
muscles . Note that the reflexes are depressed in the affected muscle groups ,
but there is no sensory deficit .
Examine the lungs for any evidence of interstitial lung fibrosis or carcinoma
Examine the breasts and palpate the abdomen for any evidence of carcinoma
(breast, stomach, uterus or ovary ) which is seen in more than 50% of cases
. (Q) What five investigations would ask for ?
(A) (1) Raised ESR and full blood count (Leucocytosis )
(2) Estimation of muscle enzymes (CPK and transaminases are raised )
(3) Electromyography (fibrillations and polyphasic discharges )
(4) Muscle biopsy (inflammation and necrosis ).
(5) Barium swallow ( atonic dilated oesophagus )

116

Scleroderma
Scleroderma or progressive systemic sclerosis is a collagen disorder of unknown
cause characterized by diffuse fibrosis and vascular abnormalities in the skin ,
joints , GIT , Lung , heart and kidneys .
Note the following features : Locally : thickened , taught and waxy skin with thin
hair on the face ,specially around the lips should give a high index of suspicion Thickened skin over the fingers with disappearance of the normal skin folds over
the knuckles , The joints of the fingers may be swollen as a result of
polyarthralgia . Tiny areas of ulceration at the ends of the digits as a result of
vasculitis . - Areas of telangiectases on the face ,lips , tongue and fingers ,
Subcuteanous calcification in the fingers is common ,
(Q) What investigations would ask for ?
(A) (1) ESR ,antinuclear factor .(2) Chest x-ray (pulmonary fibrosis)
(3) Barium meal (Dilated atonic oesophagus .
(Q) What is CREST syndrome (a) Calcinosis of the skin , Raynaud,s
phenomenon ,Sclerodactyli and telangictasia . Patients with crest syndrome are
thought to have an association with primary biliary cirrhosis

117

Gout
Look at the metatarsophalygeal joint of the big toe which will be red ,swollen ,
and tender , the big toe is the first joint to be affected in over 85 % of gout cases .
Examine for the swelling and deformity of other joints e,g, ankles , knees, ,wrists
, elbows , shoulders,hips and hands as in chronic form the condition become
polyarticular .
Look carefully for the presence of tophi which consist of deposits of urates in the
periarticular tissues and the cartilages of the ears .
Common questions (Q) What are the three investigations you would like to
ask for ? (1) Full blood count and ESR .
(2) serum uric acid .(3) X-ray of the affected joint .
(Q) What are three common precepitating factors for an acute gouty
attack?
(1) Trauma (2) Thiazide diuretics (3) Alcohol ,(Q) What are the
most useful drugs for an acute attack ? (A) Colchine (2) Indomethacin (Q)
What are the three differntial diagnosis ? (A) (1) Local cellulitis (2) Simple trauma
to the joint (3)Pyogenic arthritis

118

Hands with joint swelling


Remember the important differentiating features of common cases of joints with
hand swelling : Rheumatoid arthritis , systemic lupus erythematosus , psoraisis
,osteoarthritis and gout :
Locally : Note the swelling with or without ulnar deviation of the
metacarpophalyngeal (MCP) joints .
Note that only the proximal interphalyngeal joints( not the distal ones ) are
involved in rheumatoid arthritis .
Since the patient could have both rheumatoid and osteoarthritis ,look for the
presence of Heberden,s nodes at the terminal interphalyngeal joints .
Look for swan neck deformity or trigger finger (tendon sheath involvement
and Boutonniere (button hole ) deformity ..
Look for any associated palmar erythema
Look for pitting of the nails to exclude psoraisis .Remember that psoraitic
arthropathy is seronegative and characterstically involves the distal
interphalyngeal joints .with absence of subcuteanous nodules .
Look for lumpy swelling (tophi) of the joints ,sometimes seen with superficial
ulceration and chalky material through the shinny skin ( as seen in the photo
below ).
In systemic lupus you may find evidence of arthropathy usually involving the
metacarpophalyngeal (MCP) and the proximal interphalyngeal joints (IP) may be
seen without x-ray evidence of erosion .
Elswhere : Look for rheumatoid nodules ,if present ,around the elbow joints on
the extensor surface . Look for tophi in the cartilage of the ears .
Also check for evidence of arthropathy in other joints like knees and elbows
.Check for vasculitis lesions in the nail folds (rheumatoid arthritis ) or pitting of the
nails (psoraisis ) . Look for psoraitic lesions on the skin , most commonly on the
extensor aspects .
(Q) What two investigations would ask for ?
(A)(1) x-RAY of both hands .
(2) Latex rheumatoid factor ?

119

120

Carpal tunnel syndrome


Carpal tunnel syndrome occurs when the median nerve which runs from the forearm into the
palm of the hand becomes pressed or squeezed at the wrist , the patient main complaint
may be the painful tingling of the hands , Locally : Look for any scar of injury or bony
deformity which may suggest previous trauma to the wrist joint . Look at both hands carefully
for any evidence of rheumatoid arthritis or wasting of small muscles of the hands if present
then test the muscles of the thumb and note their weakness . Examine for any sensory loss
over the palmar aspects of the thumb ,index finger ,middle and radial half of the ring fingers
(the rest of the palm being normal ) - Ask for any paraesthesiae experienced by the patient
on percussion over the carpal ligament (Tinel's sign)
Elsewhere : Look carefully for any features of acromegaly ,hypothyroidism and amyloidosis
.(Q) What are the common causes of this syndrome ?
(A) (1) Local trauma (2) Rheumatoid arthritis (3) Acromegaly (4) Hyothyroidism (5)Pregnancy
(6) Premenstrual edema

121

Absent radial pulse (either side)


Locally :it is important to check to check both radial pulses simultaneously ,to
assess weather there is any gross difference between the two sides .
If one side the radial is feeble or completely missing ,feel for brachial ,axillary ,
and subclavian arteries respectively .
Listen for any bruit on either side of the neck .
Look for any mass compressing the arteries in the neck .
Causes : Thrombosis/embolism in a peripheral artery .
Rarely ,subclavian artery stenosis .
Brachial artery catheter .
(Q) What is pulsless (Takayasu,s disease) ?
It is a rare condition occuring particularly in young women ,All three major trunks
arising from the aortic arch are affected (brachio cephalic artery ,left common
cartoid artery , left subclavian artery.) resulting in coronary insufficiency ,absent
pulses in the arms and symptoms of cerebral ischemia . (Q) What is subclavian
steal syndrome ?
It is a rare condition in which symptoms of vertebro basilar insufficiency are
experienced by the patient on exercising his arm since the subclavian artery
distal to the stenosis ,steals, blood retrogradely from the vertebral artery , The
condition is seen in subclavian artery stenosis at or near its origin .

122

kyphoscoliosis
Note weather kyphoscoliosis is mainly in the thoracic region or thoracolumbar
region . Look for any hump or gippus formation due to rotation of the spine with
prominence of the posterior angles of the ribs .
Note weather the curvature of the scoliosis is to the left or to the right .
Look for fused vertebrae ,spina bifida and absent ribs.
Look for other features of certain conditions commonly associated with
kyphoscoliosis e.g. Marfan,s syndrome ,neurofibromatosis .
Look for any evidence of cardiopulmonary disability which may be caused or
result from gross kyphoscoliosis .
(Q) What two investigations would you ask for ?
(A) (1) Chest x-ray
(2) Lung function tests .

123

Acute pulmonary edema


Pulmonary edema is fluid accumulation in the air spaces and parenchyma of the
lungs ,it leads to impaired gas exchange and may cause respiratory failure ,it is
due to either failure of the left ventricle of the heart to adequately remove blood
from the pulmonary circulation (cardiogenic pulmonary edema ) or an injury to
the lung parenchyma or vasculature of the lung (non cardiogenic pulmonary
edema) .
Presentation : the patient is usually severly breathless ,sweaty nuseated and
anxious .they may have a dry or productive cough also may develop paroxysmal
nocturnal dyspnea or orthopnea .
Diagnosis : History : Check for past history of relevant conditions e.g ischemic
heart disease ,valvular heart disease , diabetes .and hypertension .
Review current medications ,ask about smoking and alcohol use
Signs : The patient is in respiratory distress ,pale sweaty ,tachypneic
and,tachycardic ,he may be cyanosed , have evidence of congested neck veins
and raised jugular venous pressure ,Basal wide spread of fine crepitations are
usually heared when listening to the chest .
Oxygen saturation is usually <90% (on room air) , Assess for gallop rhythm (3rd
heart sound) and murmurs suggestive of valve stenosis or regurgitation .
Hypotension - the triad of hypotension , oliguria,and low cardiac output is known
as cardiogenic shock , Pulmonary edema occurs in association with right heart
failure hepatomegaly and peripheral edema are usual (Q) What investigations
would ask for ? (A) (1) Chest x-ray : you may find cardiomegaly ,pulmonary
congestion with or without pleural effusion, ECG: Look for evidence of arrythmia
myocardial infarction, and other cardiac diseases e.g left ventricular hypertrophy
.,Echocardiography : transthoracic is usually adequate ,Blood tests ; Renal
function,electrolytes, glucose ,cardiac enzymes ,LFTs,clotting tests (PT,aPTT,
INR) ,ABGs,,

124

125

Superior vena cava syndrome


Note the conjunctival suffusion and oedema of the face ,neck , upper arms and
thorax . Note the prominent veins over the upper trunk with blood flowing
downwards (due to development of superficial collateral vessels . - Note the
prominent and distended jugular veins - Note any marking of the area of
radiotherapy , Observe for hoarsness as another sign for metatastatic
manifestation . Look for finger clubbing of the nails .
Examine the chest for signs of carcinoma of the lung e.g collapse ,consolidation ,
pleural effusion , etc
(Q) How does carcinoma of the lung cause this syndrome ?
(A) Metastasis in mediastinal lymph nodes cause compression or invasion of the
superior vena cava and thus interferes with the venous return .

126

Lung collapse
Examination of a patient with collapsed lung :
Locally : Inspection : Diminshed chest movement with flattening of the chest wall
on the side of the collapse .
Palpation : Trachea will be shifted to the side of the lesion .
Percussion : percussion note is impaired over the site of collapse .
Auscultation : Impaired breath sounds ,Bronchial breathing and crepitations may
be present if associated with consolidation .
Discussion : The shift of the trachea is seen if the upper lobe of the lung is
collapsed whereas if the lower lobe is collapsed ,it is the apex beat that is shifted
towards the side of the collapse , in the chest x-ray the area of collapse of the
lung is seen as a homogeneous dense opacity ,A lateral film may help to locate
the lobes affected by the collapse .
(Q) What are the causes of collapse of the lung ?
(A) (1) Obstruction of a major bronchus by foreign body , mucus or pus,
malignancy or enlarged lymph nodes .(2) Compression of the lung by pleural
effusion or pneumothorax .
(Q) What three investigations do you want to do ?
(A) (1) Chest x-ray. (2) Sputum for AFP ,malignant cells , culture and sensitivity
(3) Bronchoscopy .

127

Fibrosing alveolitis
Idiopathic diffuse interstitial fibrosis or fibrosing alveolitis is a condition
characterized by dyspnea ,dry cough ,clubbing of fingers and crepitations mainly
over the lower portions of lungs ,some patients with rheumatoid arthritis develop
similar fibrosis over the base of the lungs , In ankylosing spondylitis the fibrosis is
seen in only the upper portions of the lungs . Other collagen disorders causing
interstitial pulmonary fibrosis include scleroderma and dermatomyositis .
Locally : Listen to the back of the chest ,you will find fine basal crepitations
usually in both sides. -Look for other features e.g dyspnea .clubbing and
cyanosis . Look for features of right sided heart failure (cor pulmonale ) e.g.
raised JVP ,hepatomegaly and pitting oedema in the legs .
(Q) What investigations would you ask for?
(A) (1) Lung function tests
(2) Serum autoantibodies -antinuclear factors and rheumatoid factor .
(3) Blood gases .

128

Empyema: Empyema is collection of pus in the pleural space ,it must be


differniated from an abscess which is collection of pus in a newly formed cavity ,it
is a potentially life threatening medical emergency , it is a complication of other
medical conditions such as pneumonia and lung abscess .and is more common
when there is a history of excess alcohol consumption ,poor dentition ,aspiration
or general anaesthesia
Diagnosis : is suspected if a patient is slow to improve ,has persistent fever or
elevation of white cell count or CRP and has a radiological evidence of pleural
fluid collection ,PH of the pleural fluid is less than 7.2.
If untreated ,extensive fibrosis occurs in the pleural cavity ,weight loss and
clubbing develop .,pleural tap should be done immediately once empyema is a
consideration if the fluid is grossly purulent diagnosis is established .
Locally : Note decrease chest movement on the affected side
Auscultation : Decrease breath sound on the affected side ,you may hear
abnormal sound (friction rub). Elswhere : Note clubbing of fingers , and loss of
weight . (Q) What three investigations would you like to do?
(A) (1) Chest x-ray (PA and lateral) (2) CT Scan of the chest
(3) Thoracentesis (aspiration of pleural fluid for microscopic examination testing .

129

Pleural effusion
Locally : Inspection : Note the movement of the chest wall is reduced on the
affected side . Palpation : Trachea and apex beat of the heart are not shifted until
the effusion is massive , vocal fremitus is impaired or absent .
Percussion : Stony dullness over the effusion Auscultation : Absent or reduced
intensity of breathing ,vocal resonance is usually absent bronchial breathing may
be present over the upper limit of the effusion .
Elswhere : Look for possible causes of effusion e.g. rheumatoid arthritis ,
systemic lupus erythematosus .
Discussion : With the patient sitting up ,the rising dullness of the pleural fluid can
be demonestrated in patients with pleural effusion ,in the chest x-ray the opacity
seems to rise laterally , in the lateral view the opacity is not segmental or lobar in
distribution .
(Q) What are the common causes of pleural effusion ?
(A) (1) Heart failure (2) Infection e.g. pneumonia ,T.B .
(3) Infarction (4) Malignancy
(Q) What investigations would you ask for ? (A) (1) Chest x-ray
(2) Aspiration of pleural fluid for cytology ,AFP ,and biochemistry for protein
content . (4) pleural biopsy .
(Q) What is the difference between transudate and exudate ?
(A) The transudate has low protein content (less than 2 gm/litre )and is usually
seen with heart ,renal and hepatic failure , while the exudate has a high protein
content (more than 2 gm/litre )and is usually seen in cases of infection ,infarction,
and malignancy of the lung.

130

Consolidation of the lung


Consolidation simply means an area of pneumonia , On a chest x-ray there is a
homogeneous opacity with ill defined margins , a lateral film usually helps to
locate the lobes affected by consolidation. In pure consolidation a tracheal or
mediastinal shift is not expected , Clubbing of the fingers may be seen if the
consolidation is due to bronchial carcinoma
Locally : Inspection ; Note the movement of the chest wall is reduced on the
affected side , No shift of the trachea or apex beat .
Palpation : Vocal fremitus is increased , Look for lymphadenopathy in the axillae
and the neck . Percussion : Percussion note is impaired .
Auscultation : Bronchial breathing , Fine or coarse crepitations ,pleural rub may
be present , Vocal resonance is increased , Listen carefully for the presence of
whispering pectoriloquy .
Elswhere : Look for cyanosis , Examine the pulse : usually tachycardia .
Look for clubbing of the fingers
(Q) What are the causes of consolidation ?
(A) (1) Infection (2) Infarction (3) Malignancy
(Q) What three investigations do you want to do ?
(A) (1) Chest x-ray (2) Sputum for AFB ,malignant cells ,culture and sensitivity .
(3) Bronchoscopy

131

Cyanosis
Cyanosis : is a bluish discoloration of the skin and mucous membranes
,Cyanosis can be seen when the deoxygenated hemoglobin in the blood reaches
between 4-6 gm/dl ,normally 80-87% oxygen saturation could give rise to
clinically apparent cyanosis .in cases of anaemia cyanosis may be seen when
oxygen saturation may be as low as 60%.
Ask about the onst of cyanosis did it happen suddenly or has it been appearing
over a long period ? a sudden onset may be due to pulmonary emboli or severe
asthma , Ask about any associated symptoms as dyspnea chest pain ,and cough
, Ask about any past medical history of respiratory or cardiac disease .Ask about
any medication that may cause abnormal hemoglobin ,Don,t forget occupational
and smoking history.
Examination : Check the pulse and blood pressure if week and blood pressure
is low ,there may be a circulatory shock .fever in case of pulmonary infection
,look for clubbing of fingers and growth retardation in children The combination of
clubbing and cyanosis is frequent in congenital heart disease and may also occur
in pulmonary disease ( lung abscess, bronchiectasis ,cystic fibrosis and
pulmonary arteriovenous shunts ).Look for the tongue for central cyanosis Look
for the neck for raised JVP in heart failure and working accessory muscles in
severe asthma , Examine the chest : Note for barrel chest
,kyphosis,scoliosis,scars . Poor chest expansion in COPD and asthma ,unilateral
reduced chest expansion with lobar pneumonia , dullness to percussion over an
area of consolidation ,localised crepitations may be heared in lobar pneumonia
wide spread crepitations in bronchopneumonia and pulmonary edema .bornchial
breathing may be auscultated over an area of consolidation .
(Q) What are the types of cyanosis ?
(A) (1) Central cyanosis : (a) cardiac causes : cyanotic or congenital heart
diseases e,g fallot tetralogy ,transposition of great artries , and trancus arteriosus
, (b) Respiratory causes : pulmonary edema , pulmonary embolism , high altitude
sickness ,severe pneumonia ,severe asthma , COPD, ARDS (C) Causes in the
blood : methemoglobinemia ,sulfhemoglobinemia and polycythemia .
in central cyanosis limbs appear warm to touch and cyanosis does not disappear
on warming of the hands .
(2) Peripheral cyanosis : occurs with sluggish or slowing of circulation , may be
seen in heart failure and shock ,exposure to cold temperature ,arterial vascular
occulsion of one limb ,limbs appear bluish and cold to touch ,warming may
resolve the cyanosis .
(Q) What is the differential cyanosis ?
(A) Cyanosis and clubbing of the lower extremities with normal upper extremity
nail beds it is diagnostic of patent ductus arteriosus with pulmonary hypertension.

132

133

Clubbing of fingers
Clubbing is thickened tissue at the base of the nail with obliteration of the angle
between the base of the nail and the adjacent skin of the finger .
What are the causes of clubbing ? (1) Respiratory system (remmber
ABCDE ) : (1) Abscess of the lung (2) Bronchiectasis (3) Cancer of the lung (4)
Diffuse fibrosing alveolitis (5) Empyema of the lung
(2) Cardiovascular system : e.g. Cyanotic heart disease , infective
endocarditis , atrial myxoma . (3Liver and GI tract e,g, Chron,s disease
,ulcerative colitis , coeliac disease , primary biliary cirrhosis of the liver
(4) Familial / idiopathic clubbing . (5) Thyrotoxicosis (thyroid acropachy)
Locally : Note the convexity of the nail from above down as well as from side to
side - Look for the drum stick appereance of the fingers - Check for evidence of
hypertrophic pulmnory osteoarthropathy and painful swelling of the ends of
radius and ulna due to periosteal thickening .
Elsewhere : Look for dyspnea and cyanosis .- Examine the chest for conditions
like bronchiectasis ,lung abscess , carcinoma and fibrosing alveolitis - Examine
the cardiovascular system for any evidence of congenital heart disease .

134

Lupus pernio
Lupus pernio is a chronic raised indurated(hardened) lesion of the skin , it is
pathognomonic of sarcoidosis , lupus pernio must be considered in any patient
with characteristic findings of lymphadenopathy and respiratory complaints
.Locally : look at the face and notice ; - A bluish discoloration on the nose . Look for any plaques,scars, or keloids , Look for any evidence of uveitis , Look
for any evidence of facial nerve palsy .
Elsewhere : Look for the presence of erythema nodosum, Check for
lymphadenopathy , Look for any swelling of the phalanges (bone cysts )
For the emergency physician the chest x-ray is the one test that should be
obtained routinely in th ED ? (Q) what other investigations should be done? (a)
Calcium level , renal and hepatic profile , tuberculin skin testing ,ECG,
Ophthalmologic evaluation and pulmonary function testing

135

Digital ulcers
Trophic changes in the skin may be seen in: - Raynaud,s disease ; a disorder of
the peripheral circulation ,severe vasospasm referred to as Raynaud,s
phenomenon occurs frequently in women after exposure to cold ,- Digital ulcers
can be caused by all types of systemic scleroderma ,either limited or diffuse
systemic sclerosis ,digital ulcers affect approximately half of patients with
systemic sclerosis ,the ulcers can occur on fingers or toes and are often but not
always painful and difficult to heal
Trophic ulcers at the tips of the fingers are seen in leprosy and syringomyelia ( a
disease of the spinal cord in which cavitation of the central grey matter occurs ) ,
Vasculitis can cause ulceration of the fingers and even gangrene , In diabetic
patients ulcers may occur in the lower limbs due to neuropathy particularly in the
feet , treating ulcers promptly and effectively is critical to preventing development
of gangrene which can lead to amputition .

136

Raynaud's syndrome
Raynaud;s syndrome is an episodic vasospastic disorder characterized by digital color
change (white-blue-red ) with exposure to cold envirnoment or emotional stress (a) if
idiopathic it is called Raynaud;s disease . (b) If associated with a possible precipitating
systemic disorder (autoimmune disorders ,multiple myeloma ,cryoglobulinemia
,myeloproliferative disorders , myxedema , macroglobulinemia , or arterial occlusive
disease ) it is called Raynaud's phenomenon . Other vasospastic disorders such as variant
angina and migrain headache are common in patients with Raynaud's syndrome The
distinction between Raynaud's disease and phenomenon is important because of the
difference in prognosis .. Diagnosis is based on : history - physical examination .
Raynaud's disease appears first between 15-45 years almost always in women , a patient
with suggestive symptoms that persist over 3 years without evidence of an associated
disease is given the diagnosis of Raynaud's disease . Clinically both Raynaud's disease
and phenomenon are characterized intermittent attacks of pallor of the hand and fingers
brought on by cold or emotional stress progressing to cyanosis and then rubor (redness)
on rewarming ,mild discomfort ,parathesias , numbness and trace edema often
accompany the color changes . In Raynaud\s disease the disease is symmetric while in
Raynaud's phenomenon the changes may be noticed in one hand or even in one or two
fingers only .Between the attacks the affected individuals may be entirely normal .
Raunaud's disease is benign and often controllable while Raynaud;s phenomenon may
progress to atrophy and finger gangrene ..Serologic testing may be helpful in excluding
the collagen vascular disorders , scleroderma ,SLE , dermatomyositis and rheumatoid
arthritis . Cryoglobulinemia can be excluded by appropriate (serum cryoglobulins , serum
protein electrophoresis , ESR , HCV antibody testing )

137

Peripheral vascular disease Look for signs of peripheral vascular disease :


Typical changes in the skin include atrophy , pallor , loss of hair and in some
patients trophic nail changes . It is important to examine the peripheral pulses (
Femoral , popliteal, posterior tibial and dorsalis pedis pulsations ) . Patients with
chronic limb ischemi may feel coldness and loss of sensation of the toes and
dorsum of the foot together with intermittent claudication . Chronic limb ischemia
may cause gangrene of the toes and foot . Amputation is still necessary in
patients with severe peripheral vascular disease

138

Longitudinal melanotic band of the nail


Longitudinal pigmented bands are normal findings in the nails of dark skinned
persons older than 20 years , these findings present a diagnostic problem
because they must be differeniated from subungual melanomas which also occur
in older groups , when these bands are accompanied by pigmentation of
underlying skin and disruption of nail growth ,affected patients should undergo
biopsy to rule out melanoma . Factors increasing the likelihood of melanoma in a
patient who has a nail longitudinal bands : (1) New longitudinal band in a light
skinned person .(2)Sudden change in appearance of band (3) Single nail
involvement especially thumb, index or great toe .(4) pigmentation of the skin of
the nail fold or proximal nail bed (5) new pigmentation in older persons (60-79
years ) (6) Band width more than 3 mm . (7) Family history of melanoma or
dysplastic naevi .(8) Abnormal nail structure (i.e. destruction or disruption of the
nail plate .

139

Nail changes associated with systemic diseases


Beau's lines ( transverse furrows ) may follow any serious systemic illness.
Atrophy of the nails may be related to trauma or to vascular or neurologic
disease . Clubbed fingers may be due to prolonged hypoxemia associated with
cardiopulmonary disorders - Spoon nails may be seen in anemic patients Stippling or pitting of the nails is seen in psoraises ,alopecia areata and hand
eczema .

140

Leg ulcers
The ulcers develop due to problems with the circulation in the legs either venous
or arterial . Acute ulcers show healing in less than 4 weeks and include traumatic
and postoperative wounds , chronic ulcers are those that persist for longer than 4
weeks . LOCALLY : Look for the size ,surface and edges of the ulcer , palpate
the peripheral arteries for any evidence of peripheral vascular disease , look for
the presence of varicose eczema with or without varicose veins . Elsewhere :
Look at the hands for any evidence of arthropathy , look at the face for the
presence of xanthelasma , look for any evidence of splenomegaly , The following
are the commonly seen ulcers on the legs : (A) Varicose ulcer ; venous
incompetency causing edema, ecezma and secondary bacterial infection
commonly results in varicose ulcers around the ankles most commonly around
medial malleolus ,lower and medial aspect of the leg , the ecezma is due to
brownish hemosiderin pigmentation (b) Ischemic ulcer : seen in extreme degree
of peripheral vascular disease , the limb is usually cold to touch with poor or
abscent peripheral pulses .(c) Rheumatoid arthritis : the ulcers are usually over
the lower legs , there may be areas of vasculitis in the finger tips , rheumatoid
hands with or without splenomegaly (d) Necrosis lipoidica diabeticorum :
yellowish red sclerotic plaques most commonly seen on the shins with ulceration
.(e) Pyoderma gangrenosum : the ulcers have elevated purulent border with a
zone of erythema beyond the edge , ulcerative colitis ,multiple myeloma and
leukemia are associated conditions , Sickle cell anemia ,tertiary syphilis ,
systemic sclerosis , and vasculitis are other causes of leg ulcers .

141
Steven Johnson syndrome ( SJS ) and Toxic Epidrmal Necrolysis (TEN) : Steven
Johnson syndrome and Toxic epidrmal necrolysis are 2 forms of life threatening skin
condition in which cell death causes the epidermis to seprate from the dermis ,it is
hypersenstivity complex that affects the skin and mucous membranes .
Causes : The most well known causes are certain medications ,but it can also be due to
infections and rarely cancers . Classification ; there 3 forms : (1) Steven Johnson
syndrome ( SJS ) (2) Toxic epidermal necrolysis (TEN) (3) SJS/TEN overlap . the
distinction between the 3 is based on the type of the lesion and the amount of body surface
area covered with blisters and erosions .
in SJS 3-10 % of body surface area covered with blisters and erosions.
In SJS/TEN overlap 11-30 % of body surface area covered with blisters and erosions , In
Toxic epidermal necrolysis (TEN) over 30% of body surface area is covered with blisters and
erosions .
Signs and Symptoms Typical prodromal symptoms of Steven Johnson Syndrome include :
cough productive , of a thick purulent sputum ,headache , malaise and arthralgia , the patient
may complain of burning rash that begins symmetrically on the face , the rash can begin as
macules,that develop into papules, veasicles, bullae,urtricial plaques or confluent erythema
The typical lesion has the appereance of a target , this is considered pathognomonic . In
contrast to the typical lesion of erythema multiforme ,these lesions have only 2 zones of color
,the lesion,s core may be vesicular purpuric or necrotic that zone is surrounded by macular
erythema ,lesions may become bullous and later rupture leaving denuded skin which
become susceptible to 2ry infection which is responsible for scarring associated with
morbidity ,urtrical lesions typically are not pruritic , the palms soles , dorsum of the hands
and extensor surfaces are most commonly affected , signs of mucosal involvement include
Erythema,Edema , Sloughing ,Blistering ,Ulceration ,Necrosis.
Management : the symptomatic treatment of patients with SJS does not differ from the
therapy applied to patients with extensive burns ,withdrawal of any suspected offending
agent ,tetanus prophylaxis must be given
IV fluids and nasogastric or parentral feeding and symptomatic e.g analgesic mouth rinse for
mouth ulcer . Treatment with corticosteroids is controversial , IV immunoglobulin in treatment
has shown some promise in reducing the length of reaction and improving symptoms , use of
topical pain anesthetics and antseptics, consult ophthalmologist

142

Cuteanous vasculitis
This condition may be due to a number of different causes , such as adverse
drug reactions , reduced circulation associated with peripheral vascular disease
or varicose veins , malignancy or autoimmune disorders ,symptoms are the result
of inflammation of any of blood vessels or lymphatics in the skin . Presenting
symptoms usually appear in the lower limbs with a rash that comprises petichiae
, palpable purpura ,ecchymosis ,blisters and or ulceration .
Management : The diagnosis can usually be made on clinical grounds , although
skin biopsy will sometimes help to confirm it . However screening tests will be
required to try to identify the underlying cause , full blood count and ESR ,
antinuclear antibodies to check on autoimmune disease ,antistreptococcal
antibodies in case of recent streptococcal infection , hepatitis screen , protein
and immunoglobulin electrophoresis in case of multiple myeloma and
cryoglobulins to exclude cold antibodies , If a cause is found,the underlying
condition should be treated and if a drug is implicated it should be withdrawn ,
Rest ,elevation of the limb and analegesia may be sufficient to allow for
resolution , while more severe problems may sometimes require medication such
as prednisolone , a non- steroidal antiinflammatory drug or hydrochloroquine,
Ultimately cuteanous vasculitis has a good prognosis , although recurrences
sometimes occur .

143

Henoch-Schonlein purpura
It is a systemic vasculitis (inflammation of blood vessels) ,usually occurs in
children and young adults ,and generally has a good prognosis ,it is
characterized by immune complex deposition in small vessels with associated
vasculitis ,The typical presentation is with purpura over the buttocks and lower
legs , abdominal symptoms (pain and bleeding ) and arthritis (knee or ankle)
following an upper respiratory tract infection .Nephritis is found in 40% of patients
and may occur up to 4 weeks after the onset of other symptoms . The diagnosis
can be confirmed by tissue biopsy which demonestrates deposition of igA within
and aroud blood vessel walls ,the prognosis is determined by the severity of
renal involvement . Adverse prognostic features at presentation in adults include
hypertension ,abnormal renal function ,and proteinuria > 1.5 g/day but only 1% of
patients develop end stage renal failure .
Management of HSP includes adequate hydration; immediate discontinuance of
any exposure to antigenic stimulants (eg, drugs); and follow-up each week for the
first month, every other week for the second month, and monthly thereafter until
abnormal urinary findings subside.
Patients with HSP are often admitted to the hospital and monitored for abdominal
and renal complications, which may be severe and may occur precipitously (eg,
acute abdomen, acute scrotum, and renal failure). Hospitalization should be
strongly considered for HSP patients with severe abdominal pain, significant
gastrointestinal (GI) bleeding, or marked renal insufficiency. All pregnant women
with even mild renal symptoms at the onset of HSP should be carefully observed
during and after pregnancy.
Nephropathy is treated supportively. Patients fluid and electrolyte balance
should be monitored, their salt intake should be restricted, and antihypertensives
should be prescribed when needed. Various drugs (eg, corticosteroids,
azathioprine, and cyclophosphamide) and plasmapheresis have been used to
prevent renal disease from progressing. ,conservative management with ACE
inhibitors ,ARBs or fish oil (omega 3 fatty acids ) may prevent deterioration of
renal function ,progressive disease may benefit from a course of prednisolone 1
mg/kg /d with or without cytotoxic agents , other investigations CBC , you will find
the platlet count is normal ,urine examination for hematuria and proteinuria , skin
biopsy to detect arteiolar and capillary vasculitis(,Antinuclear Factor to exclude
SLE , VDRL to exclude secondary syphilis ),

144

145

Tongue abnormalities
Recognition and diagnosis of tongue abnormalities require a thorough history
including onset and duration ,tobacco and alcohol use , Examination of tongue
morphology and a careful assessment for lymphadenopathy are also important
(a) Pallor of the tongue : this indicates anaemia ,and may be associated with
atrophy of the filiform papillae resulting in a smooth clean -looking tongue which
may be caused by iron or vitamin B deficiency , (b) Enlarged tongue : May be
seen in cases of amyloidosis ,acromegaly ,and myxedema and if so look for the
features of such conditions (c) .Black hairy tongue :This is commonly due to
fungal infection . This patient may be a heavy smoker on steroids or antibiotics in
some cases . In candidiasis (thrush) creamy white patches appear on the tongue
which can be easily scraped off.
(d) Geographical tongue : Disappearance of patches of papillae leaving smooth
areas on the tongue . This type of tongue has no clinical significance , (e)
Leukoplakia : This type of tongue is characterized by a painless ,whitish patch
which can not be wiped off ,This is a precancerous condition ,it can be a marker
for underlying immunodeficiency ,is caused by EBV and is treated with oral
antivirals .(f) Tongue growths : usually require biopsy to differniate benign
lesions from premalignant leukoplakia or squamous cell carcinoma

146

Jaundice
Ask about anorexia,malaise, emesis myalgias and weight loss prior to the onset
of jaundice (suggest viral hepatitis) , Ask about infectious exposure ,injection
drug use or prior transfusion of blood , A history of recent biliary surgery and
fever especially when accompanied by rigors and right upper quadrant pain is
suggestive of biliary obstruction with colangitis , sudden onst of jaundice with
right upper quadrant pain in otherwise healthy individual suggest gall stones ,
slow onset of painless jaundice with central abdominal ache ,loss of appetite and
weight loss suggest malignancy .
Pruritus occurs before the patient becomes overtly jaundiced in a middle aged
females and the cause is uknown suggest primary biliary cirrhosis .
occupational history may be important e.g in sewage workers ,exposure to
slaughter houses or stagnant water ( may suggest leptospirosis) or people
exposed to hepatotoxic chemicals .
Travel history to any country where hepatitis or any other infective cause is
endemic (e.g malaria ) , Drug history weather prescribed or non prescribed ,Ask
about alcohol abuse , ask about pregnancy in females ,
Ask about history of contact with jaundiced patient ,ask about family history of
jaundice ,anemia especially when requiring splenectomy for inherited
hemoglobinopathies such as thallassemia or sickle cell disease .
Examination : Look for the general appearance of the patient , mental state
,obvious weight loss , muscle wasting is almost universal in patients with chronic
liver disease , the presence of parotid gland enlargement ,gynaecomastia and
Dupuytren,s contracture is highly suggestive of chronic alcohol abuse . skin
bruising from disturbing blood coagulation ,itching marks ,Look for signs of
anemia ,examine the hands for palmar erythema , Duputren,s contracture ,look
for the depth of jaundice,Kayser-Flaisher ring , check for distended jugular
venous pressure , look for other stigmata of the chronic liver disease
(gynaecomastia, spider naevi, caput medusae ,ascites ) Check for sacral and
pitting edema Abdomen : Examine for hepatomegaly , a rapidly shrunken liver
combined with jaundice and rapid clinical deterioration indicate acute liver failure
, a hard or nodular liver is probably fibrotic or infiltrated by tumour , splenomegaly
is suggestive of cirrhosis ,hematological disorders or reticulosis . Elicit Murphey,s
sign , percuss for ascites
Check for lymphadenopathy , Perform PR examination
Jaundice must be distinguished from yellow or green skin resulting from
carotenemia or quinacrine ingestion , by the absence of yellow color in mucous
membranes and sclera in these two conditions and also the normal urine color
and the accentuation of yellow brown caretonid pigment in palms ,soles and
nasolabial folds (quinacrine is commonly used for treatment of giardiasis.

147

148

Carotenemia
Carotenemia is a clinical condition characterized by yellow
pigmentation of the skin (xanthoderma) and increased beta carotene
levels in the blood . In most cases the condition follows prolonged
and excessive consumption of carotene rich foods such as carrots
and sweat potatos it i. it is a common finding in children and
vegetarians . The condition is harmless but it can lead to mistaken
diagnosis of jaundice . Clinical features : Carotenaemia is
characterized by yellow discoloration of the skin particularly in the
soles and palms , The sclera and mucous membranes are unaffected
by carotenaemia . The presence of yellow sclera means increased
circulating bilirubin (jaundice) . Treatment : The skin color can return
to normal when the patient is advised not to overeat foods containing
carotene , N.B. the yellow color of the skin may persist for a few
months due to accumulated carotene in tissues .

149

Pyogenic liver abscess


This patient present with malaise ,low grade fever and dull abdominal pain
increased by movement . The onset may be insidious and the duration at least
one month before diagnosis .Multiple abscesses are associated with more acute
systemic features ,the single abscess is more insidious and often cryptogenic , If
there is subdiaphragmatic irritation or pleuro-pulmonary spread of infection the
patient may complain of right shoulder pain and an irritating cough ,the liver is
enlarged and tender and pain may be accentuated by percussion over the lower
rib cage . The spleen is palpable in chronic cases . Jaundice is late unless ther is
suppurative cholangitis .Recovery may be followed by portal hypertension .
Investigations : Jaundice is usually mild except in cholangitic types.
It is more common than with amoebic abscess .. Serum alkaline phosphatase is
usually raised , The ESR is very high , Polymorphnuclear leucocytosis is usual ,
Blood cultures show the causative organism or organisms in 50 % .
Ultrasound distinguishes a solid from -fluid filled lesion , CT scanning is
particularly valuable , ERCP may be used to diagnose cholangitic abscesses

150

Amoebic liver abscess


An amoebic abscess is caused by the protozoon Entamoeba
histolytica ,which invades the colonic mucosa and is carried to the
liver via the portal circulation .confirmation that the abscess is
amoebic is usually made serologically . examination of faeces : cysts
and vegetative forms are rare .
In a patient with hepatic abscess the symptoms of sepsis (spiking
fevers ,night sweats ,poor appetite and loss of weight ) tend to
overshadow the right upper quadrant pain ,jaundice is unusual and if
present ,mild ,bile duct
compression with large and multiple abscesses may lead to jaundice
as a presentation ,hepatic tenderness is constant ,it may be elicited
over a palpable lower edge or by percussion over the lower right
chest wall , the spleen is not enlarged , the lungs may show
consolidation of the right lower zone ,pleurisy or an effusion , pleural
fluid may be blood stained , Ultrasound is most useful in diagnosis ,
CT is more sensetive in small abscesses

151

Wilson's disease
Autosomal recessive disorder ,in which copper absorption is normal but
intrahepatic formation of ceruloplasmin is defective which is the principal
transport protein for copper and necessary for biliary excretion , Clinically the
disease is characterized by abnormal copper deposition in the basal ganglia as
well as elsewhere in the brain ,the eye and in the liver . This disorder should be
considered in any young person presenting with an extrapyramidal syndrome
,psychiatric symptoms are particularly common in adults . Hepatic dysfunction :
Acute hepatitis,chronic hepatitis , cirrhosis, massive hepatic necrosis Hypoparathyroidism - Hemolysis .
CNS involvement : (Often presents later than hepatic dysfunction ): Behavioral
problems /psychosis - mental retardation ,Tremor/chorea ,seizures -Kyser
Fleisher corneal rings . Diagnosis : (1) Decrease serum ceruloplasmin - Increase
urinary excretion of copper
N.B. Serum copper levels are of no diagnostic value .

152
Primary biliary cirrhosis
The cause is uknown although factors point to an autoimmune etiology especially the strong
association with other autoimmune disease such as rheumatoid arthritis ,Sjogren syndrome
and CREST syndrome ,Ninety percent of patients are female often in middle age
What are the symptoms?
Many people who have primary biliary cirrhosis do not have symptoms. When symptoms do
occur, they may include:
Itching (pruritus).
Fatigue.
Yellowing of the skin and the white part of the eyes (jaundice).
Discomfort in the upper right part of the abdomen.
Dry eyes and mouth.
Vaginal dryness. Signs : Jaundice , Xanthelasmata due to hypercholestrolemia - Skin
pigmentation -Clubbing -Hepatosplenomegaly -Signs of Portal hypertension -Osteoporosis
and osteomalacia
Diagnosis : Antimitochondrial antibody present in 95% - Raised alkaline phosphatase in
advance of symptoms and signs - Raised IgM- Liver biopsy:shows bile duct obstruction proliferation -fibrosis or Cirrhosis
Treatment : symptomatic ,liver transplantation remains the only hope of cure

153

Palmar erythema: Note the redness of the palms , especially over the thenar , hypothenar
eminences ,and the pulps of the fingers .Elsewhere : since the commonest association is
with liver cell disease look for signs such as Dupuytren,s contracture , spider naevi
,hepatosplenomegaly ,ascites, oedema of the legs , gynaecomastia ,absent secondary
sexual hair ,etc Common associations : (1) cirrhosis of the liver (2) pregnancy (3)Females on
contraceptive pills (4) Thyrotoxicosis (5) Rheumatoid arthritis (6) some normal people
Discussion : palmar erythema is a persistent extensive redness of the palms due to local
vasodilatation with increased blood flow , the cardiac output is usually increased in patients
with prominent palmar erythema .
Skin over the thenar , hypothenar , and finger pulp prominences is markedly red as
compared to the rest of the palm .

154
Gynaecomastia
Gynaecomastia is enlargement of the male breast tissue
What are the causes of gynaecomastia ? (1) Can be seen during infancy and adolescence (
physological) (2) Pathologic hyperestrogenism as in liver disease (reduced estrogen
metabolism ) ,it is one of the stigmata of liver cirrhosis( palmar erythema,deupytren,s
contracture , leuconychia ,spider naevi , gynaecomastia ) as mentioned in the previous
cases (3) testicular tumours (4) HCG producing tumours , (5) Adrenal tumours (6)
Medications : e.g. cimetidine , digoxin ,tricyclic antidepressants Marijuana, spironolactone
and alpa methyl dopa (7) Breast carcinoma (8) Hypogonadism with reduced androgens
(relative increase in estrogen )
Make sure that this is definite breast tissue and not excessive fat deposition in an obese
person , if the person is very tall look for features of Kleinfelter syndrome (disproportionately
long lower extremities , gynaecomastia in 50 % of patients , firm and small testis ) , look for
stigmata of liver cirrhosis , look for testicular tumours , look for rare conditions which may
account for gynaecomastia e.g. acromegaly ,thyrotoxicosis and addison,s disease

155

Spider naevi
It is a type of telangiectasis (swollen blood vessels) foun slightly beneath the skin surface
often containing central red spot and reddish extensions which radiate outwards like a
spider;s web ,more than 3 spider naevi is likely to be abnormal
Distribution : they are found only in the distribution of the superior vena cava (face
,neck,upper part of the trunk and arms ) , it is one of the stigmitata of liver cirrhosis ,but can
also occur in healthy individuals ,firm pressure with a glass slide or with a pen will cause it to
blanch and when the pressure is released it will rapidly refill .
Look for other stigmata of liver disease (eg palmar erythema ,ascites,
gynaecomastia,leuconychia )
(Q)What investigations you should ask for ?
(A) For most healthy and young people or just a few and especially children no investigation
is required , otherwise assess liver functions including hepatitis markers ,thyroid function
tests may be indicated as they can occur with thyrotoxicosis .
(Q) What are the associated conditions ?
(A) Spider naevi may be associatedwith any condition that result in increased circulating
levels of estorgens including :
pregnancy , Alcoholic cirrhosis , Hepatitic cirrhosis , Hepatosplenomegaly

156
Dupuytren,s contracture
It is flexion deformity of the metacarpophalyngeal joints especially of the ring and middle
fingers and loss of function of the fingers , the fibrosis and thickening of the palmar fascia
and of the flexor tendons is the underlying pathology . The common conditions associated
with Dupuytren,s contracture are : Portal cirrhosis ,Rheumatoid arthritis , , Familial ,
Epileptics , Trauma (gardners , vibrating machines ) .
-Ask the patient to spread the fingers so as not to miss this condition , also
make sure that you examine both hands so as not to miss the condition in the other hand ,
Early Dupuytren,s contracture may be missed unless you carfully not to forget to palpate the
palmar fascia ,look for palmar erythema , clubbing , and leuconychia .Since the commonest
association is with cirrhosis of the liver , look for other stigmata of cirrhosis e.g. spider naevi
,hepatosplenomegaly , ascites , oedema of the legs ,etc.
It is important to palpate the palm so as not to miss the thickened fascia in cases of early
Dupuytren,s contracture , In most cases the condition becomes bilateral Differnitial diagnosis
: The condition should be differentiated from ulnar nerve palsy where the dorsal and palmar
interossei muscles are paralysed with resultant loss of flexion of proximal phalanx ,extension
of the dorsal phalanx, adduction and abduction of fingers , Also ther will be loss of sensation
over the ulnar border of the hand .

157

Leuconychia :
Leuconychia means white nail , this whiteness may affect the whole nail or bands
or flecks of whiteness , it is one of the stigmata of liver cell disease
(cirrhosis) , It is the result of hypoalbuminaemia due to any cause
Causes :(1) Liver cirrhosis (2) Nephrotic syndrome (3) Protein - losing
enteropathy (4) Chronic organic arsenic poisoning
N.B. Small isolated white patches may sometimes be seen in the nails of normal
persons (Q) What two investigations would you ask for ?
(A) (1) liver function tests . (2) Urine examination for proteinuria

158

Lower limb swelling


Swelling of lower limbs may be bilateral or unilateral
(a) Bilateral lower limb swelling : causes (1) Congestive cardiac failure (2) Nephrotic
syndrome (3) Liver cirrhosis (4) Primary lymphoedema
(5)Filariasis (6) Neoplastic infiltration of lymph nodes (7) Surgical removal of lymph nodes .
Local examination : Determine weather the oedema is pitting (venous ) or non pitting
(lymphatic ), determine weather the local temperature of the overlying skin is raised or not ,
look for varicose veins ,look for lymphadenopathy in the groins , most patients are obese
middle aged females if the swelling is due to primary lymphoedema
Elsewhere : look for raised JVP , hepatomegaly , heart murmurs and congestion of the
lungs (basal crepitations ) , Hypoalbuminemia either due to nephrotic syndrome or liver
cirrhosis cause bilatera pitting edema of the legs so look for the relevant signs e.g. renal
enlargement ,puffiness of the eye lids (nephrotic syndrome ) or stigmata of cirrhosis as
mentioned in the previous cases palmar erythema , ascites , Dupuytren,s contracture ,
leuconychia , spider naevi and hepatosplenomegaly
(b) Swelling of one leg Common causes : (1) Deep vein thrombosis of the leg (2) Cellulitis of
the leg (3) Lymphedema (4) Ruptured Baker,s cyst
Locally : Determine weather the oedema is pitting or non pitting , compare skin colour with
the other leg , determine weather local temperature of the skin is raised or not ,look for
positive Homan,s sign , measure the girth of both legs Elswhere : check for any
lymphadenopathy in the groins , look for any abdominal mass causing obstruction to the
venous flow to the leg . examine knee joint .N,B. DVT is a common condition and carries the
risk of pulmonary embolism .

159

Hereditary hemorrhagic telangiectasia


Hereditary hemorrhagic telangiectasia or Osler -Rendu -Weber syndrome is inherited as an
autosomal dominant condition , Bleeding from the superficial lesions may be profuse .
Visceral telangiectasia or AV aneurysms may occur in the lungs ,liver and spleen . Epistaxis
and GIT bleeding is more common and much more serious .Blood transfusions are usually
required for acute hemorrhages . Continuous iron therapy to correct the iron deficiency
anaemia is usually necessary for many patients .It is important to remember that small
telangiectatic lesions may also be seen in scleroderma ,but the character of the skin remains
unchanged in hereditary hemorrhagic telangiectasia , whereas in scleroderma the skin
becomes thick , taught and difficult to pinch .
(Q) What is the role of hormones in the treatment ?
(A) Oestrogens are considered to be of value in the therapy and in many cases reduce
epistaxis ,testosterone is also required to avoid undesirable femenizing effects .

160
Beta Thalassemia Major
Anemia ( pallor) and monogloid features :
Anemia ,large head ,prominent maxilla and mongloid features . The clinical triad of chronic
anemia ,mild jaundice and splenomegaly (or hepatosplenomegaly) is characteristic of
chronic hemolytic anemia , positive family history and history of repeated transfusions may
be obtained . Growth failure (short stature ) and skeletal changes (large head , prominent
maxillae ,monogloid features ) may be prominent especially in severe cases of Cooley's
anemia .

161
Idiopathic thrombocytopenic purpura (ITP)
It is the most common cause of purpura in children . The illness is mostly immune in origin
and is often preceded by a viral infection 1-4 weeks before the onset of illness . The onset is
abrupt with purpura and may be bleeding . Laboratory diagnosis depends on the presence of
severe thrombocytopenia (platlet count is usually below 20,000),prognosis is excellent in
most cases . Purpura and bleeding usually subside over 1-2 weeks . Persistence of purpura
and thrombocytopenia for more than 2-3 weeks is an indication for bone marrow examination
and leukemia or aplastic anemia should be excluded . .Essentials of diagnosis : Isolated

thrombocytopenia ,Other hematopoietic cell lines are normal , no associated


systemic illness , spleen not palpable ,normal bone marrow with normal or
increased megakaryocytes .Differential diagnosis : disorders such as
disseminated intravascular coagulation (DIC) , thrombocytopenic purpura (TTP) ,
hemolytic uremic syndrome (HUS) , hypersplenism and sepsis are easily
excluded by absence of systemic illness Conclusion :: Patients with isolated
thrombocytopenia with no other abnormal findings almost certainely have ITP

162

Disseminated Intravascular Coagulation (DIC)


Disseminated intravascular coagulation (DIC) is characterized by systemic
activation of blood coagulation, which results in generation and deposition of
fibrin, leading to microvascular thrombi in various organs and contributing to
multiple organ dysfunction syndrome (MODS). Consumption and subsequent
exhaustion of coagulation proteins and platelets (from ongoing activation of
coagulation) may induce severe bleeding,It occurs as a complication of another
severe systemic illness , the main precepitating factors are septicemia ,shock
and acidosis , In children gastroenteritis with dehydration ,shock and acidosis is
an ideal situation for DIC to develop . Clinically the patient is critically sick and
the features of the precepitating disease are well evident (as septicemia ,shock
,acidosis) The hematological manifestations of the disease include bleeding from
puncture sites and surgical incisions ,purpura(petechiae and ecchymosis ) and
necrotic skin patches (characteristic of DIC) .Internal hemorrhage including
(intracranial hemorrhage) may occur due to thrombocytopenia and severe
coagulation defect
Laboratory diagnosis depends on the presence of thrombocytopenia ,severe
coagulation defect (prolonged thrombin ,prothrombin and partial thromboplastin
times) and Fibrin degedration products (FDPs) in the peripheral blood .
Prognosis is generally bad and it depends on the proper control of the
precepitating factors and the extension of internal hemorrhage .

163

Polythythemia
Polythythemia refers to increase in red cell count ,hematocrite and usually
hemoglobin , True polythythemia may be either primary or secondary
Primary true polythythemia : uncontrolled production of red cells by the bone
marrow , Clinical features :Look for the plethoric features of the patient , Look for
the peripheral arterial pulsations , note for limb swelling (venous thrombosis) or
digital ulcerations (arterial thrombosis) Look for itching marks due to pruritus
,Check the blood pressure for hypertension , Look for swelling erythematous big
toe (gout), examine the abdomen for splenomegaly (Q) What are the causes of
secondary polythythemia ?
(A) Increased renal erythopoietein production either due to hyopoxia or renal
causes e.g. renal causes or ectopic tumours . Investigations : High red cell count
, hemoglobin ,hematocrite , whole blood viscosity and uric acid

164

Hemophilia
Hemophilias are a group of inherited disorders that result from a defect in phase
1 coagulation
Hemophilia A (Classic hemophilia ) : It is the most common type (80% of cases )
. It is an x-linked disease caused by deficiency of factor 8 (antihemophilic factor) .
The clinical severity depends on the level of factor 8 activity in plasma . The most
characteristic features of hemophilia A are spontaneous or traumatic
hemorrhages which can be subcuteanous , intramuscular or within joints
(hemarthrosis ) . In infants excessive bleeding may follow circumcision . but
bleeding is usually not evident in the first year of life , with time the child begins to
walk ,easy bruising and hemarthrosis becomes evident , Laboratory diagnosis
depends first on the presence of prolonged partial thromboplastin time (phase 1
defect)
Hemophilia B (Christmas disease ) : is the second most common type of
hemophilia (about 10% of cases ) It is an x-linked disease caused by factor 9
deficiency , clinically it cannot be differeniated from hemophilia A .

165

Grey -Turner's sign


Grey Turner;s sign refers to bruising of the flanks , the part of the
body between the last rib and the top of the hip . The bruising
appears as blue discoloration and is a sign of retroperitoneal
hemorrhage , or bleeding behind the peritoneum . It takes 24 -48
hours to develop and can predict a severe atteck of acute pancreatitis
. Causes : Acute pancreatitis- Pancreatic hemorrhage - Blunt
abdominal trauma - Ruptured / hemorrhagic ectopic pregnancy Spontaneous bleeding secondary to coagulopathy .- Ruptured aortic
aneurysm

166

Virchow's node ( Left supraclavicular lymphadenopathy ) :


Virchow;s node is a lymph node in the left supraclavicular fossa (the
area above the left clavicle ) , it may result from lymphatic spread of
gastric or pancreatic neoplasm via the thoracic duct . This may be the
first sign of malignancy . Biopsy of the node is indicated and search
for the primary tumour . N.B. Lymph nodes harboring malignant
disease tend to be firm . non tender , matted (stuck to each other )
fixed (not freely mobile but rather stuck down to underlying tissue and
increase in size over time .
Infected lymph nodes , tend to be firm , tender ,enlarged , and warm ,
inflammation can spread to the overlying skin , causing it to appear
reddned

167

Fixed drug eruption


So called because the lesion recurs at the same site after each
administration of a specific drug . The lesion is intensly itchy . Acute
fixed drug eruption usually presents with a single or a small number
of dusky red or violaceous plaques that resolve leaving post
inflammatory hyperpigmentation . A common cause is
phenolphthalein found in various laxative preprations ,other drugs
most frequently associated with fixed drug eruption include :
trimethoprim-sulfamethexozole,tetracycline ,penicillins quinolones ,
dapsone ,clotrimazole ,phenytoin, cetrizine,and pseudoephedrine,

168

Lupus vulgaris
This is a persistent and progressive painful cuteanous tuberculosis .
The slowly extending lesion is hyperpigmented at its margin and
depigmented in the healing central zone , ulceration may occur if left
untreated . Skin lesions with nodular appearance most often on the
face around the nose , the lesions . The lesions persist for years
leading to disfugrment .it may develop in the extremities e.g. dorsum
of the hands and at the site of BCG vaccination .

169

Ichthyosis vulgaris Ichthyosis vulgaris is a skin condition that causes dry, dead
skin cells to accumulate in patches on the surface of the skin
, the scales of ichthyosis vulgaris sometimes called fish skin disease .There are
two types : Hereditary ichthyosis vulgaris and acquired ichthyosis vulgaris . The
hereditary type is first evident in early childhood ,it is the most common form of
ichthyosis ,visible scales are retained for longer periods and sloughed off in
clumps .
Acquired ichthyosis usually appearing in adulthood is a non hereditary condition
associated with internal disease ,it must be viewed as a marker of systemic
disease including malignancy , Cases have been attributed to use of certain
medications .Symptoms of ichthyosis vulgaris are typically worse in winter, when
the air is colder and dryer.
Symptoms of ichthyosis vulgaris include:
a flaky scalp
itchy skin
polygon-shaped scales on the skin
scales that are brown, gray, or white
severely dry skin
thickened skin
The patches of dry skin typically appear on the elbows and lower legs, most often
affecting the shins in thick, dark segments. In severe cases, ichthyosis vulgaris
may also cause deep, painful cracks to develop on the soles of the feet or palms
of the hands.

170

Acne vulgaris
Acne vulgaris ( or simply acne ) is a long term skin condition
characterized by non inflammatory open or closed comedones (black
heads) and by inflammatory papules , pustules and nodules . Acne
vulgaris typically affect the areas of skin with the densest population
of sebaceaous follicles e.g. face , upper chest and back , . The older
lesions are healing and scarring .
Local symptoms may include pain , tenderness , or erythema ,
systemic symptoms are most often absent.

171

Rhinophyma
Rhinophyma consists of hypertrophic ,hypermic ,large nodular
masses centered on the distal half of the nose. Rhinophyma is seen
almost exclusively in men over age 40 .The tip and wings of the nose
are usually involved by large lobulated masses , which may even be
pendulous . The hugely dilated follicles contain plugs of sebum and
keratin . The cause of rhinophyma is unknown . It is usually
associated with long history of rosacea
Rhinophyma is best treated by surgical ablation ,electrosurgery
(surgical cutting current) ,laser
surgery or wire brush surgery.

172

Rosacea
Rosacea is a chronic inflammatory condition of the face that is
characterized by redness(a red face due to prominent,dilated blood
vessels (telangiectasia which is the first stage of rosacea ) . Red
papules and ,pustules on the nose ,forehead , cheecks and chin often
follow occasionally(papulo -pustular rosacea). By the overgrowth of
nasal connective tissue ,enlarged nose with prominent pores and
fibrous thickening (rhinophyma ). Rosacea superficially resembles
teenaged acne but there are no black heads , white heads or nodules
(sometimes called acne rosacea ) ,but it occurs in adults

173

Angioedema
Rapid swelling of the dermis , subcutanous tissues ,mucosal and submucosal
tissues , very similar to urticaria but urticaria occurs in the upper dermis .
Cases where angioedema progresses rapidly should be treated as medical
emergency as airway obstruction and suffocation can occur , the cause may be
hereditary or allergic as a side effect of a number of drugs , it is particularly
known to be associated with ACE inhibitors , the effect is not seen with
angiotensin receptor antagonists , Epinephrine may be lifesaving when the cause
of angioedema is allergic .

Sad

174

Eczema
Pompholyx eczema (vesciular eczema ): This common type of eczema affects
hands and feet , the patient develops itchy vesciles on the soles of the feet
,palms of the hands and sides of the fingers . These vesciles coalesce to form
larger blisters that break down ,weep and crust ,and the skin becomes cracked ,
Secondaty infection ,usually staphylococcal ,may occur . This condition is most
commonly found in young adults and the middle aged ,and while the cause is
uknown it sometimes seems to be associated with excessive sweating or stress .
Aggrevating factors may be irritants and some patients are found to be senstive
to nickel .

175

Pemphigus
Pemphigus vulgaris : It is a serious autoimmune disease in which IgG antibodies
target desmosomal proteins to produce intraepithelial and mucocuteanous
blistering . The condition is most commonly found in the 50-60 age group , and
the diagnosis should be suspected in anyone with mucocuteanous erosions or
blisters as signs may first appear in the mouth ,pharynx,oesphagus ,genitals and
conjunctiva . Skin lesions may develop several months later . Unlike pemphigoid
,the flaccid blisters quickly rupture and the superficial blisters containing clear
fluid ,rapidly break down leaving erosions that are painful and slow to heal , Any
site may be affected ,including the axilla ,umbilicus ,trunk ,scalp and face .

176

Pemphigoid
Bullous pemphigoid : This is a problem that affects the elderly and most
commonly presents at around the age of 70 ,it is thought to be an autoimmune
disease in which the IgG immunoglobulins and activated T lymphocytes attack
components of the basement membranes ,The patient develops cropsof tense
blisters that contain clear or blood stained fluid which do not easily break down
as the roof of the blister is made up of entire epidermis . The condition usually
causes much irritation . The blistering may be localized to one area ,often starting
on the inner thighs or may be more widespread and involve the mouth . Before
the onset of the blisters ,the patient may present with an itchy ,urticarial rash that
is often misdiagnosed as dermatitis or urticaria .

177

Vitiligo
Vitiligo is a condition that causes depigmentation of parts of the skin ,it occurs
when melanocytes (the cells responsible for pigmentation ) die or unable to
function . Vitiligo is strongly associated with a number of autoimmune disorders
,autoimmune thyroiditis 21%,IDDM (1-7%) , Addison,s disease (approximately
2%) .
Pernicious anemia ,SLE ,and possibly IBD are linked with vitiligo .
Vitiligo is commonly seen as a component of a number of autoimmune
polyendocrinopathies , Collagen vascular disorders may be associated with
hypopigmentation -Diffuse scleroderma may result in areas of hyperhypopigmentation

178
Ankylosing spondylitis
Look at the patient or at the back as the patient is standing .
Note the loss of normal lumbar concave curve (lordosis) which is due to the muscle spasm
and involvement of adjacent spinal joints .
Note the flexion deformity of the hips and neck , and see if the patient is walking in a stooped
posture with stiff ankylosed spine and has difficulty in seeing ahead . Ask the patient to bend
forward and notice the limitation ofjoint movements , similarly the lateral movements of the
spine are limited .
Measure the expansion of the chest ,this may be affected due to costovertebral joint
involvement .
Check for movements of the neck and sacroiliac joints ,these may be limited too .
Listen to the heart for the murmur of aortic incomptence .
Look for any evidence of uveitis .
(Q) What three investigations would you like to do ?
(A) (1) ESR ,Hb
(2) Spine x-ray including sacroiliac joints .
(3) Chest x-ray ( bilateral apical fibrosis )

179
Livedo reticularis: This can arise from a variety of reasons :
(1) Cutis mamorata : causes temporary livedo in about 50% of normal infants and many
adults when exposed to cold ,it is more intense and persistent in conditions associated with
debility and other factors that cause stasis within blood vessels ,the mottling is diffuse
,temporary ,mild and usually symptomless. (2) Idiopathic livedo reticularis (i.e. cause
uknown ) occur most commonly in young and middle aged females particularly during winter
Mottling occurs first only on exposure to cold but can become permenent , tingling and
numbness on cold exposure are common .(2) Sneddon's syndrome : is a form of idiopathic
livedo reticularis with systemic involvement i.e. internal blood vessels are affected most
commonly in brain ,eye,and heart .
(3) Secondary livedo reticularis : it may be a sign of (A) vasculitis : ,livedoid vasculitis,poly
arteritis nodosa,SLE, dermatomyositis ,rheumatoid arthritis,lymphoma,pancreatitis,T.B.(B)
Obstruction: - Cryoglobulinemia
-Antiphospholipid syndrome or lupus anticoagulant syndrome . - Hypercalcemia Polycythemia rubra vera,Thrombocythemia .
Infections(T.B. and syphilis - Arteriosclerosis and homocystinuria ,
Intra arterial injection (drug addicts).
- Adverse drug reaction (Amantadine)

Like

180

Cluster headache
Attacks occuring in clusters lasting days or weeks and remissions lasting months males are
more often affected than females and onset of attacks is typically between 25 and 50 years .
Typical features of cluster headache : - unilateral severe headache lasting up to an hour Lacrimation - partial Horner's may occur - pain may be retroorbital - Redness of ipsilateral
eye - Nasal stuffiness Management of acute attack includes : inhaled oxygen (face mask) ergotamine and sumatriptan -steroids may be helpful

Hah

181
Migraine
Classically preceded by a visual aura followed by a unilateral throbbing headache with
photophobia and nausea . Features of migraine : EEG and neurovascular abnormalities
associated with the headache - Rarely may result in stroke - May have a unilateral
lacrimation - Can be associated with (Reversible ) neurological signs e.g. (hemiplegic
migraine), therapy is aimed at stopping an attack (abortive) or if the frequency of attacks is
high enough ,regular medication is given as a prophylactic agent Migraine therapy : Abortive
- Paracetamol - Codeine +antiemetic - Ergotamine - Sumatriptan (5HT1agonist)
Prophylactic : Propranolol -Pizotifen - Amitriptyline - Methsergide

182

Pes cavus
Locally : Note the high arched foot , Test for any motor or sensory deficit in both lower limbs .
Elsewhere : Check for features of syringomyelia e.g. wasting and trophic lesions of the
hands with dissociated sensory loss and upper motor neurone signs in the lower limbs - Look
for features of Friedreich's ataxia e.g. ataxia ,intention tremors ;dysarthria and nystagmus
(spinocerebellar tract) impaired joint and vibration sensation (posterior column ) ,extensor
plantar reflexes (corticospinal tracts) and optic atrophy . Absent ankle jerk with extensor
plantar reflexes may be the sole manifestation . In peroneal muscle atrophy the legs look like
inverted champagene bottles. Pes cavus may be a feature of Charcot -Marie -Tooth disease

183

Syringomyelia/Syringobulbia
Look at the hands and note the wasting of the small muscles of the hands with or without
trophic lesions - Check for segmental areas of loss of pain and temperature sensation and
see if there is any scar of painless burn or cut on the fingers - Look for signs of lower motor
neurone lesions in the upper limbs (due to involvement of the anterior horn cells) and upper
motor neurone lesions in the lower limbs due to involvement of the pyramidal tracts- Look for
signs of Horner's syndrome on either side - Look for the presence of nystagmus (probably
due to involvement of fibres of the vestibulospinal tracts )- Look for fasciculations

184

Dystrophia myotonica
Both muscular dystrohy and myotonia are seen in this autosomally
dominant condition which usually begins in adult life , mental
retardation may also be seen . Look at the wasting of of face and
neck muscles particularly the sternal head of sternomastoids . Look
at the eyes for bilateral ptosis which is quite commonly seen . Look
for cataracts .
Frontal balding is also commonly associated , the distal parts of the
limbs may also show some muscle wasting ,especially in the
forearms .
Check the grip of the hands on both sides and look for slow relaxation
of the muscles . Testicular atrophy must be looked for as well .

Sad

185
.
Facioscapulohumeral muscular dystrophy
Look at the bilateral symmetrical weakness of facial and sternoclavicular muscles .
Note bilateral ptosis without any over action of frontalis muscle .
Examine and note the weakness of sternal head of the pectoral muscles .
Ask the patient to push against a wall with the arms extended and note the winging of the
scapulae . Also examine the muscles of the pelvic girdle and lower limbs as in later years
they may also be involved .
Demonestrate the loss or poverty of reflexes in the affected muscles with no sensory deficit .
(Q) What is the inheritance of this disorder of muscles ?
(A) Autosomal dominant .
(Q) What are the 2 other important possible diagnosis ?
(A) (1) Dystrophia myotonica .
(2) Mythenia gravis .

186
Pseudobulbar palsy
Definition : bilateral pyramidal tract lesion above pons
Look at the patient and examine the cranial nerves , Note the emotional lability of the patient
,the patient may have a peculiar facial expression and laugh or cry for no apparent reason .
Ask the patient to protrude the tongue and note that this only be achieved with difficulty
,because of the spasticity and stiffness of the tongue . Test for the movement of the soft
palate and note that the palate is paralysed (note in the photo below accumulation of saliva
in the hypopharynx due to palatal paralysis ) .
Demonestrate briskness of the jaw reflex .
Ask the patient about his address and notice the dytharthria .
Look for the presence of fasciculations in the limb muscle (motor neurone disease) . Look for
upper motor neurone signs in the limbs usually more marked in lower limbs with spasticity ,
brisk reflexes and upgoing planters .
(Q) What are the causes of pseudo bulbar palsy ?
(A) (1) Inflammation : Encephalitis (2) Bilateral cerebrovascular accidents
(Double stroke) . (3) Multiple sclerosis (4) Motor neurone disease (5) Midline brain tumour .
(Q) What is bulbar palsy ?
(A) This condition is less common than pseudobulbar palsy , it is caused by bilateral nuclear
lesion (lower motor neurone lesion ) ,characterized by bilateral wasting of the tongue with
fasciculation (last photo) ,paralysis of the palate ,dytharthria , and rarely extraocular muscle
palsy

187

Horners syndrome
The syndrome consists of ptosis ,miosis ,enophathalmos and anhydrosis .
Look at the face of the patient and notice the eyes :
Note the partial ptosis on either side .
Note the small size of the pupil on the same side which fails to respond to the light reflex .
Look carefully for the small and rather sunken eyeball (enophathalmos) .Lack of sweating on
that side of the face is rather difficult to demonstrate and not considered a very important
part of the syndrome . Elswhere : Look for any signs of brain stem lesion e.g. nystagmus .
Examine the neck for cervical ribs ,lymphadenopathy , or a scar of injury or operation .
Check for patchy loss of sensations in the upper limbs (syringomyelia ) .
(Q) What are the causes of Horner,s syndrome ?
(A) (1) Lesion in the brain stem paralysing the autonomic pathways .
(2) Syringomyelia . (3) Lesion in the superior mediastinum e.g. aneurysm, glandular
enlargement ,bronchial carcinoma .
(4) Lesions in the neck e.g trauma ,lymphadenopathy

188
Neurofibromatosis
Locally : Look for the cuteanous fibromas commonly seen on the trunk which are discrete
movable lumps arranged along lines of nerves ,sometimes may be painful or tender on
pressure . Look for the cafe -au-lait spots ,which are colored patches of skin pigmentation .
Pigmentation is most prominent over the trunk . Axillary freckling is commonly seen .
Look for kyphoscoliosis .
Elswhere : Look for deafness or cerebellar signs (ataxia ,nystagmus, and scanning
speech)for acoustic neuroma which may be seen in a patient with neurofibromatosis . Check
the blood pressure as phaeochromocytoma (causing hypertension ) is rarely associated with
neurofibromatosis . Ocular manifestations of neurofibromatosis are good clue for diagnosis
and may be the first criteria for diagnosis. All ocular tissues may be involved. Lisch nodules
of the iris is the early and pathognomonic (Q) What is the treatment of this condition ?
(A) No treatment is required unless a localized tumour causes cerebral or spinal
compression ,if the patient is hypertensive phaeochromocytoma should be strongly
suspected .Phaeochromocytomas ,intracranial and spinal tumours require surgery . Recent
advances in laser technology have permitted non surgical removal of small cuteanous
neurofibromas

189
Papilloedema
Papilloedema is optic disc swelling that is caused by increased intracranial pressure the
swelling is usually bilateral and can occur over a period of hours to weeks , Further
evaluation with CT ,MRI of the brain and /or spine is usually performed , unilateral
papilloedema can suggest an orbital pathology , The different stages of papilloedema are :
Enorgegment of the retinal veins , Blurring of the disc margins , A redderdisc with loss of the
physiological cupping of the disc .
Take a full history ,specifically ask about any headaches and their characterstics ( the
headache and nusea /vomiting worse on waking ,coughing , bending ). N.B Patients with
papilloedema should be considered to have an intracranial mass until proved otherwise .
(Q) What are the causes of papilloedema ?
(A) (1) Raised intracranial pressure . (2) Malignant hypertension
(3) Central retinal vein thrombosis . (4) Optic neuritis/papillitis
(5) Rarely hypercarpia ,hypoparathyroidism ,
(Q) What is the difference between papillitis and papilloedema ?
(A) In papillitis the condition is usually unilateral with loss of visual acuity ,whereas in
papilloedema the condition is usually bilateral and the visual acuity remains normal but there
is an enlargement of the blind spot .
(Q) What is Foster Kennedy syndrome ?
(A) Papilloedema on one side and optic atrophy on the other is called Foster Kennedy
syndrome . This may be caused by a tumour of the frontal lobe ,which may cause optic
atrophy because of the pressure on the optic nerve ,with increase in size of the tumour ,there
is a rise in the intracranial pressure ,and this causes papilloedema on the other side

190
Parkinsonism
Parkinsonism is characterized by slowness and poverty of emotional and voluntary
movement , rigidity and tremors , parkinson,s disease is due to decrease dopamin content in
axonal terminals of cells projecting from the substantia nigra to caudate nucleus and
putamen .
In parkinson,s disease the mental state remains normal .Tremor or rigidity may occur alone
or co-exist .
Note the following features : The patient is invariably elderly with an expressionless(mask)
face . Note the pill rolling movements of the thumbs and fingers on one or both sides . The
tremors are coarse ,slow ,present at rest and disappear on voluntary movement .
Check for the presence of other extrapyramidal signs such as cog wheel rigidity of the limbs
.Ask the patient to walk and note the short shuffling character of his gait with absence of
normal swinging of the arms .
The patient is hurrying with small steps in a stooped posture .
(Q) What are the causes of parkinsonism ?
(A) (1) Idiopathic (2) Drug induced (e.g phenothiazines) (3) Post encephalitic (4) Toxic
(copper in Wilson,s disease ,mangnese or carbon monoxide poisoning ) (5) Arteriosclerotic

191
Subarachnoid hemorrhage
The term subarachnoid hemorrhage refers to extravasation of blood into the subarachnoid
space between the pial and arachnoid membranes ,the most common cause is head trauma
,non traumatic SAH occurs in the setting of ruptured cerebral aneurysm or arteriovenous
malformation , first clue in subarachnoid hemorrhage is clinical history ,sudden onset of
severe headache (the worst headache you have ever seen in your life ) with or without
sudden loss of consciousness ,accompanying nusea and vomiting ,signs of meningeal
irritation ,photophobia and visual changes ,focal neurologic deficits may occur like aphasia
,monoplegia ,hemiplegia , focal fits , seizures may occur during the acute phase .
On Examination :
You may find mild to moderate elevation of blood pressure .Fever ,tachycardia, papilloedema
,global or focal neurologic abnormality .
(Q) What are the complications of SAH?
(A) -Hydrocephalus ,-Rebleeding ,seizures ,cardiac dysfunction

192

Cerebral edema
Cerebral edema is simply defined as increase brain content . it is frequently encountered
in clinical practice in critically ill patients with acute brain injury . Causes : (a) Traumatic
brain injuries ,subarachnoid hemorrhage , and intra cerebral heamorrhage .
(b) Non traumatic : Ischemic stroke , neoplasms , inflammatory diseases
(meningitis,encephalitis, ventriculritis,cerebral abscesses ). and severe toxic metabolic
derangements (Hyponatremia and fulminant hepatic encephalopathy ) ,Cerebral edema is
a major cause of morbidity and death in these patients . Cerebral edema causes the gyri to
swell and the spaces between them (sulci) to decrease or even oblitrated so you can not
see the CSF spaces between the gyri . Types : (1) Cytotoxic edema , affects both grey and
white matter the character of cytotoxic edema (a) effacement of the cortical sulci (b)
compression of the ventricles (c) shift of the midline structures . (2) Vasogenic edema :
commonly encountered in traumatic brain injuries , neoplasms and inflammatory
conditions it appears as finger like projections around the SOL , (C) Interstitial edema : a
consequence of impaired absorption of CSF resulting in acute hydrocephalus . Symptoms
include : nusea , vomiting ,blurred vision ,faintness and in severe cases seizures and coma
if herniation occurs respiratory symptoms or even arrest can occur due to compression of
the respiratory center . Management : general measures : optimal head and neck
positioning (3o degrees) ,avoidance of dehydration and systemic hypotension,
maintainance of normothermia acetaminophen 325 or 650 mg orally or rectally every 4-6
hours , Specific theraputic interventions interventions : Controlled hyperventilation ,
Administration of corticosteroids , diuretics ,osmotherapy , Hypoxia and hypercapnea
should be avoided (potent cerebral vasodilators) , also systemic dehydration and the use
of hypotonic fluids should be avoided . Maintain euvolaemic or slightly hypervolaemia
with the use of isotonic fluid (0.9% saline) ,Vigorous attention to daily fluid balance
,body weight ,and serum electrolyte monitoring .Sharp rises in systemic blood pressure
should be avoided , Potent vasodilators (nitroglycerin and nitroprusside are to be avoided
as they may exacerbate cerebral edema .Anticonvulsants (phenytoin) are widely used
emperically in clinical practice . Hyperglycemia can also exacerbate cerebral edema
,Mannitol remains the major osmotic agent of choice in clinical practice Hypertonic
saline has been recommended in patients with acute brain injury .Mannitol : dose 0.25 -1
gm /kg body weight every 4-6 hours or 20% mannitol 5 ml/kg over 5-10 min.followed by
3ml/kg 6 hourly till 48 hours .Oral glycerol 1ml/kg /dose 8 hourly may be used if ICP
elevated beyond 48 hours . Acetazolamide in a dose of 20-25 mg/kg in 3-4 doses .
Furosemide 1-2 mg/kg /day i.v . Dexamethazone use is controversial .

193

194

Facial (7th nerve palsy )


Note the loss of nasolabial fold and the assymmetrical face on either side .
Ascertain weather this involves the lower half of the face (UMNL) or the complete half of the
face (LMN lesion) . This can be done by asking the patient to produce furrows over the
forehead ,blowing out the cheeks , closing the eyes tightly ,whistling and smiling .
UMNL type of facial palsy :
Look carefully for the presence of associated hemiparesis or hemiplegia , as a patient with
UMN facial palsy would have very commonly a stroke on the same side of the facial
paralysis ,as you notice this patient with UMN FACIAL paralysis on the left side (notice the
massive cerebral infarction on the right fronto-temporal area ) there is also left hemiplegia ,
this may be associated with speech difficulties .
LMN type of facial involvement :
Look for any parotid swelling (tumour) .Look behind the ear for any scar of operation on the
mastoid , Look for any evidence of sarcoidosis on the face such as lupus pernio plaques etc.
Remmber sarcoidosis may cause bilateral facial LMN type of palsy .
(Q) What are the other causes of facial (LMN type ) palsy ?
(1) Bell,s palsy . (2) Multiple sclerosis . (3) Neoplasm (acoustic neuroma )

195

6th nerve palsy (Abducent )


Note the deviation of the eyeball medially (convergent squint) of the affected eye
. This is due to the unopposed action of the medial rectus muscle ,. Now test the
various movements of each eye seprately by asking the patient to follow your
finger in all directions ,In cases of 6th nerve palsy the eyeball on the affected side
fails to move laterally ,due to the paralysis of the external rectus .Also the patient
will experience diplopia when trying to move the eye in the direction of the
paralysed muscle
(Q) What are the common causes of 6th nerve palsy ?
(A) (1) Raised intracranial pressure (due to the stretching of the nerve ) and thus
a false localizing sign .(2) Vascular lesion e,g, aneurysm of the internal cartoid
artery .(3) Neoplasm (4)Encephalitis (5) Multiple sclerosis

Haha

196

3rd nerve palsy


Note : The eyeball is rotated outwards (by the unopposed action of lateral
rectus ) and downwards (by the unopposed action of superior oblique ).
Note the ptosis which is due to drooping of the levator palpabra superioris .Note
that the pupil is dilated (due to unopposed action of the sympathetic impulses )
with loss of light and accommodation reflex .Test the functions of the muscles
supplied by the third nerve (oculomotor) and you will find that the eyeball fails to
move in other directions .
(Q) What is ptosis ? (A) Drooping of the eyelid is called ptosis , normally only the
upper one sixth of the cornea is covered by the eyelid ,more than that is ptosis .
(Q) What are the important causes of third nerve palsy ?
(1) Vascular lesion e.g. thrombosis, aneurysm or hemorrhage .(2) Neoplasm (3)
Diabetes mellitus (4) Multiple Sclerosis (5) inflammatory : meningitis commonly
TB meningitis

197
Ptosis
Definition : Ptosis is drooping of the upper eyelids ,
Determine weather the ptosis is unilateral or bilateral ,partial or complete ,with or without
over action of the frontalis muscle , Look for size of the pupil ,light and accommodation
reflexes and presence of strabismus (the two eyes do not line up in the same direction and
therefore do not look at the object at the same time as occurs in 3rd nerve palsy and
Horner,s syndrome) .
Look for any evidence of myopathy and other features of dystrophia myotonica e.g. cataract
,gonadal atrophy and frontal baldness .
(Q) What are the causes of ptosis ?
(A) (1) 3rd nerve palsy : ptosis is usually complete ,unilateral and with over action of frontalis
,Paralytic squint with a dilated pupil are additional features . (2) Sympathetic paralysis
(Horner,s syndrome) . The ptosis is always partial with small pupil .(3) Tabes dorsalis : ptosis
is usually bilateral with over action of frontalis (4) Myopathies : like fascioscapulohumeral
myopathy ,dystrophia myotonica or myathenia gravis .( 5) Congenital : a common cause of
unilateral or bilateral partial ptosis with over action of frontalis .(6) Hysterical ptosis : very rare
and is always unilateral

198
Hydrocephalus
Note that the skull is globular ,smooth ,and symmetrically enlarged and the facial bones are
normal . - Both eyeballs are prominent (mild to moderate exophthalmos ) and are rather
pushed downwards so that the upper part of the sclera is visible . - Look for signs of raised
intracranial pressure e.g. fundi for papilloedema or optic atrophy .
(Q) What is the differnitial diagnosis of hydrocephalus patient who has survived into adult life
? (A) (1) Paget,s disease of the skull ,in these patients the skull is irregularly enlarged and
there may be bowing of the tibia as well .(2) Achondroplasia , in this autosomal dominant
disorder , the person is carrying a normal size skull with a short stature and thus the head
appears to be enlarged in relation to the height .
(Q) What is the neurosurgical management of hydrocephalus ?
(A) Shunt operation for diversion of CSF flow , either ventriculo-peritoneal or ventriculo-atrial

Li

ke

Love
HahaWow

199

Multiple sclerosis or Disseminated sclerosis


Definition : Diseases characterized by loss of myelin sheath (a substance that
surrounds the axon of the nerve in a layer .The disease is characterized by
affection of the white matter of CNS (brain and spinal cord ) resulting in
demyelination with reactive gliosis and impaired function , Distirubation is patchy
in brain and spinal cord and lesion can be easily detected by MRI .Age of onset
between 20-40 years The disease has acute onset with remission and
exacerbation . The mode swings between euphoria ,depression ,emotional
lability (uncontrolled burst of crying or laughing) , Look for speech of the patient
(slurred or staccato speech ) , Examine the cranial nerves (commonly 3rd
,4th,6th ,7th are affected ,trigeminal neuralgia is an early symptom , vertigo is
also a common symptom (due to affection of cochleo-vestibular nerve ) Examine
the field of vision you may find visual field defect also color vision may be
affected . Examine the motor system : you may find monoparesis
,hemiparesis,,paraparesis Check the tone and reflexes : you will find hypertonia
and hyperreflexia with positive Babiniski sign .Examination of the sensory system
: you may find deep sensory loss , Flex the head of the patient ,the patient may
feel electric radiation in the back (Positive L'hermite sign),Check for cerebellar
tests : you may find cerebellar ataxia Ask about bladder disturbance (precipitency
or retention ) and sexual dysfunction (impotence) .Lastly do not forget to
examine the fundus :you may find optic neuritis or primary optic atrophy
(temporal pallor of optic disc (characteristic ) . (Q) What is the most valuable
diagnostic investigation ?
(A) MRI : Shows multiple white matter lesions .

200

Cushings syndrome
Hypersecretion of glucocorticoids produces Cushing.s syndrome , this is usually caused
by bilateral adrenocortical hyperplasia due to an excess of pituitary adrenocorticotrophic
hormone (ACTH) . Less commonly it is caused by an adrenal adenoma or rarely by
carcinoma .
Look at the patient and note the following features :
Moon shaped rounded face with plethoric appearance .
Buffalo hump - prominent supraclavicular and dorsal cervical fat pads .
Truncal obesity , Purple striae over the abdomen . Purpura and spontaneous bruising over
the extremities . Acne and hirsutism .
Note the blood pressure as patients are often hypertensive .
(Q) What investigations would ask for ?
(A) (1) Tests to confirm hypercortisolism (Cushing syndrome):
(a) Loss of diurnal variation (midnight cortisol not lower than morning cortisol ) . (b)
Dexamethasone suppression test (1mg at midnight then 9 a.m cortisol. (c) Low dose
dexamethasone suppression test (0.5mg qds for 48 hours .(d) Urinary free cortisol (24
hour collection) .
If one of these are positive then one can proceed to localization .
Tests to localize the cause of Cushing syndrome :
Possible causes of cushing syndrome are :
Adrenal tumour .-Pituitary tumour (Cushing disease )
Ectopic production of ACTH - either from cancer (e,g. small cell cancer of the lung ) or
from bronchial adenoma (often very small) .
Ectopic production of corticophin -releasing hormone (CRH) (very rare).
MRI scanning of the adrenal or pituitary alone can not be relied on to localize the cause ,
the tests used to identify causes of cushing syndrome are : ACTH test , high dose
dexamethasone suppression test ,CRH stimulation test ,Metyrapone test (these tests to
differniate adrenal ,from pituitary and ectopic causes of cushing syndrome.

201

Alopecia
Loss of hair is called alopecia and can be scarring or non scarring , Scarring
alopecia is loss of hair with destruction of the hair follicles while in non scarring
alopecia the hair follicles are preserved
Causes of scarring alopecia : Hereditary : Ichthyosis- Bacterial : Tuberculosis, Syphilis . Physical injury : Burns , Radiotherapy Fungal: Kerion Others : Lichen
planus , Lupus erythematosis , Sarcoidosis ,Cicatricial pemphigoid . Causes of
non scarring alopecia : Male pattern /androgenic alopecia : the most common im
men and women - Alopecia areata - Endocrine : Hypopituitary state ,
Hypothyroidism , Hyperthyroidism , Hypoparathyroidism , Pregenancy - Drugs :
Retinoids -Anticoagulants -Antimitotic drugs -Oral contraceptive pills Carbimazole -Thiouracil -Lithium - Iron deficiency - Chronic illness . Alopecia
areata is associated with nail dystrophy , cataracts ,vitiligo ,autuimmune thyroid
disease ,pernicious anemia and Addison's disease

202

Hirsutism
Hirsutism refers to the growth of coarse ,dark hair in areas where women
typically grow fine hair or no hair at all . Sites : above the lips and/ or chin ,chest
,abdomen and back , the excess hair growth is caused by an increased level of
male hormones (androgens) Causes : (1) Ovarian: - Polycystic ovary syndrome Ovarian tumours (2)Androgen therapy (3) Adrenal : (1) Congenital adrenal
hyperplasia (2) Caushing syndrome (3) Prolactinoma . the two most common
causes are polycystic ovary syndrome and idiopathic hirsutism . Symptoms :
deepening of voice -Balding -Acne -Decrease in breast size - Enlargement of
clitoris . History : An accurate history of patient's onset of hirsutism and
developmental milestones can be hepful in etiologic diagnosis , Age of onset :
idiopathic and other less serious causes usually begin at puberty - Hirsutism that
occur in middle aged or older women should suggest adrenal or ovarian tumour .,
Family history : A patient with family history of hirsutism is consistent with
congenital adrenal hyperplasia and polycystic ovary syndrome .
Ask about rate of progression : Pubertal onset with slow progression in benign
form and polycystic ovary syndrome While rapid severe hirsutism with other
signs of virilization is suspected in androgen secreting tumours .- Early
development of pubic hair point to congenital adrenal hyperlasia While normal
development of pubic hair and delayed menarche or irregular mensses is
associated with ovarian hypergonadism

203

Acromegaly
Acromegaly results from the excessive secretion of human growth hormone
,caused by acidophilic adenoma of the pituitary ,Acromegaly is characterized by
an increase in the size of the viscera ,bones and soft tissues of the hands ,feet
,supraorbital ridges ,sinuses and lower jaw ( proganthism ) .Glycosuria occurs in
about 30% of untreated cases and hypertension is not uncommon .
Locally : Look carefully at the face -enlarged lips ,accentuated skin folds with
characteristic coarsning of the facial features . Look for proganthism
(protrusion of the lower jaw ) . Look for any scar of operation on the skull .
Always check for visual field defect e.g bitemporal hemianopia
Elswhere : Look at the hands which are unusually thick and fleshy .
Shake hands with the patient and you may get the feeling of loosing your hand in
a mass of dough .(Q) What three investigations will you ask for ?
(A) (1) X-ray Skull (enlargement and erosion of sella turcica ) , hands
(tufting of terminal phalanges ) ,feet (heel pad thickness ).
(2) Serum growth hormone level (fasting level in excess of 10ng/ml) .
(3) Blood sugar , (Q) How can the condition present ?
(Q) (1) Headaches and visual problems .
(2) Paraesthesia of the hands and feet .(3) Change in size of the hat,ring or
shoes (4) Arthralgia and excessive sweating .(5) Diabetes mellitus .
(6) Hypertension .

204

Goitre
Any enlargement of thyroid gland is described as goitre
Look at the neck and note the swelling of the thyroid gland and check weather the
swelling moves with deglutition .
Palpate both from the front and behind the the patient to determine the size ,shape ,
consistency , and extent of the swelling including any possible retrosternal extension ( the
photo below with chest radiograph ,and computed tomography showing an enlarged right
upper mediastinal mass one of the differnitial diagnosis is intrathoracic goitre ) . Check
for any signs of compression of the trachea ,recurrent laryngeal nerve (ask for any
difficulty in swallowing and speech) , and superior vena cava . Listen carefully for any
bruit over the gland ,if present this suggests hyperactivity of the gland .
Look for any enlarged lymph nodes in the neck which may mean metastasis from
carcinoma of the thyroid . Look for other signs of thyrotoxicosis staring look of the
patient with exophthalmos ,lid lag ,tremors , and pretibial myxoedema ) , or of
hypothyroidism ( coarse features with dry skin , slow pulse and slow relaxation of the
ankle jerk

205

Addisons disease
Note the brown or grey discoloration of the mucous surface of the lips and buccal mucosa
,especially the inner aspect of the cheeks ,also there may be pigmentation of the palate
,gums and sides of the tongue .
Note the hyperpigmentation of the exposed parts of the body ,e.g. palmar creases ,hands
,arms , face and neck , Nipples and genitalia are also excessively pigmented . Note that
the patient looks asthenic and if female there may be loss of hair both axillary and pubic .
Note that the blood pressure is low , and check for postural hypotension .
(Q) What are the other causes of pigmentation of the mouth ?
(A) (1) Familial intestinal polyposis (Peutz -Jegher syndrome )
(2) Racial (3) Arsenic , bismuth or silver intake .(4) Metastatic malignant melanoma .
(Q) What is peutz -Jegher syndrome ? ( A) It is an autosomal dominant condition
,characterized by the presence of mucocuteanous pigmentation of the lips ,buccal mucosa
,palms ,toes and umbilical area along with the presence of multiple polyps in the small
intestine . .(Q) What are the causes of pigmentation in Addison,s disease ? (A)
Pigmentation is caused by excess of both melanocyte stimulating hormone (MSH) and
(ACTH) ,the lower the cortisol secretion the higher the (ACTH) and (MSH ) level

206

Exophthalmos or proptosis
Locally : Look for the prominence of the eyeballs , if in doubt examine the
patient in profile ( from the side ) and from above .Note the lid retraction with
wide palpebral fissure . Test for the presence of lid lag . ( Lid lag means delay in
moving the eyelid as the eye moves downwards.)
Check the movements of the eyeballs to rule out exophthalmic ophthalmoplegia .
Listen for any bruit over the eyeball.
Listen for any bruit over the eyeball .
Elswhere : In the neck , look for any thyroid enlargement .
Ask the patient to spread the fingers and look for the presence of fine tremors
.Look for pretibial myxoedema .Examine the cardiovascular system for the
presence of tachycardia, atrial fibrillation.
(Q) What are the causes of unilateral exophathalmos ?
(A) (1) Thyrotoxicosis (early stage) (2) Retro-orbital tumours or cellulitis
(3) Cavernous sinus thrombosis - usually seen with marked chemosis and
ophthalmoplegia

207

Renal enlargement
Common associations : (1) Polycystic disease of the kidney .(2)
Hydronephrosis (3) Renal tumours (4) Nephrotic syndrome .
Locally : Make sure that this is renal enlargement and not splenomegaly .
- See if the other kidney is enlarged as well .
Elswhere : Look for anaemia , Any puffiness around the eyelids , Does the
patient look uremic ? , Look for oedema of the legs .
(Q) How can you differentiate the kidney enlargement from splenomegaly ?
(A) In kidney enlargement it is possible to pass fingers between the upper end of
the kidney and ribs , in splenomegaly you can not , The kidney is bimanually
palpable while the spleen is not so ,,The kidney has rounded border while the
spleen has sharp edge ,there is a band of colonic resonance over the kidney ,in
spenomegaly it is not so , The kidney moves only slightly with respiration while
the spleen moves freely with respiration .
(Q) What investigations would ask for?
(A ) (1) Urine examination for protein ,cells and casts ,etc.
(2) Renal function tests .(3) IVP (4) Renal ultrasound (5) Abdominal CT Scan .

208

Renal mass
On examining the back of your patient , you should look at both loins ,
you may find fullness at one or both renal angles (also called
costovertebral angle which is located in the back between the 12th rib
and the spine ) . The kidney lies directly below this area . This mass
characterizes the kidney mass if pallotable on bimanual examination
and moves freely with respiration. Note this young boy has a mass in
the left loin with fullness at the left angle.

209

Uraemic facies

Note the pale -yellow brown appearance of the skin and anemic pallor of the
sclera Also the,uremic frost present on the forehead and scalp of a young
man who presented with complaints of anorexia and fatigue . The uremic
syndrome can be defined as the terminal clinical manifestations that result
from inadequate excretory ,regulatory ,and endocrine function of the kidney
Signs and symptoms: Classical signs of uremia are: progressive weakness
and easy fatigue, loss of appetite due to nausea and vomiting, muscle
atrophy, tremors, abnormal mental function, frequent shallow respiration and
metabolic acidosis. Without intervention via dialysis or kidney transplant,
uremia due to renal failure will progress and cause stupor, coma and death.
Spontaneous bleeding can occur with severe uremia and may include GIT
bleeding and spontaneous subdural hematoma .

210

Nephrotic syndrome
It is a clinico-laboratory syndrome of 4 components :
(1) Generalized edema : Swollen eyelids,puffy face and edematous limbs ,scrotal
edema in males ,abdominal wall edema and ascites occur in advanced cases .
(2)Massive protennuria : above 3 g proteinuria / day in adults and above 2 g /24
hours in children (levels above 5-10 gm /24 hours may occur .
(3) Hypoproteinemia : serum albumin below 2.5 gm/dl
(4) Hypercholestrolemia : Serum cholestrol level above 300 mg/dl.
Causes : (A) Common :
-Primary glomerulonephritis - Diabetes mellitus - Basement membrane
nephropathy (e.g. Alport syndrome ) - Infections e.g. leprosy , malaria ,hepatitis B
associated with secondary glomerulonephritis - Pre-eclampsia - Accelerated
hypertension - Myeloma - Amyloidosis - Drugs (e.g. gold, penicillamine, captopril
, NSAID, mercury - associated with secondary GN ) - Connective tissue disease(
e.g. SLE)
Rare : Vesico - ureteric reflux - Constrictive pericarditis - Sickle cell disease Allergies (e.g. bee sting ,penicillin ) - Hereditary glomerulonephritis

211

Wegener's granulomatosis
A type of systemic vasculitis in which arteries ,veins and capillaries are affected
by granulomatous formation , Ninety percent of cases present with upper or
lower respiratory tract symptoms ,Upper airway involvement includes crusting
and granulation tissue on the nasal turbinates producing nasal obstruction and
bloody discharge ,collapse of the nasal bridge produces a saddle shaped nose ,
cANCA is present in 90% of cases , Respiratory symptoms : cough ,hemoptysis
,breathlessness and pleurisy , stenosis of a main airway may cause severe
breathlessness and stridor.
Chest x-ray : large rounded shadows may be visible on the chest x-ray and these
often cavitate ,pleural effusion and infiltrates may develop , 75% develop
glomerulonephritis and eye and joint involvement are common .
Patients may have typical vasculitic skin rash and mononeuritis multiplex may
also develop .

212

Chaurg Strauss syndrome


A rare systemic vasculitis with associated asthma and eosinophilia , any organ
can be affected but the lung and skin are the most commonly affected
Clinical manifestations : -Rapidly progressive glomerulonephritis - Pulmonary
hemorrhage - hemoptysis - Constitutional symptoms (Fever -chills- weight loss
,arthralgia ,myalgia , -Cuteanous (Purpura ,urticaria ,subcuteanous nodules CNervous system (mononeuritis)- Cardiac (pericarditis ,HF, coronary vasculitis )GI (diarrhea ,GI bleeding - Renal : proteinuria, hematuria . pANCA is positive in
50% of cases - Chest x-ray shows nodular or confluent shadows without
cavitation
According to the American College of rheumatology criteria 4 of the following 6
criteria needed and biopsy evidence of vasculitis for diagnosis :
(1) Asthma (a history of wheezing or the finding of diffuse high pitched wheezes
on expiration ) (2)- Eosinophilia >10% present on differential white cell count (3)
Mononeuropathy (including multiplex )or polyneuropathy
(4) Migratory or transient pulmonary opacities detected radiographically
(5) Paranasal sinus abnormality (6) Biopsy containing a blood vessel showing the
accumulation of eosinophils in extravascular areas

213

Lymphadenopathy
Examine pre and post -aurical ,submental ,tonsillar and occipital lymph nodes along with
detection of enlarged supraclavicular lymph nodes in the head and neck area .
In the upper limb, always look for enlargement of epitrochlear lymph nodes .Now
examine both axillae and groins for any lymphadenopathy .
Do not forget to examine the abdomen for any enlargement of paraaortic or iliac lymph
nodes and hepatic or splenomegaly ,
Always look for consistency ,tenderness and matting of the lymph nodes which help in
deciding about the possible cause of lymphadenopathy .
(Q) What are the causes of generalized lymphadenopathy ?
(A) (1) Acute and chronic leukemias .
(2) Lymphoma .
(3) Infections e.g. infectious mononucleosis ,secondary syphilis and miliary tuberculosis
(4) Collagen diseases e.g. systemic lupus erythematosis , Still,s disease and rheumatoid
arthritis .
(5) Other conditions e.g. sarcoidosis .
.

214

Hepatosplenomegaly
The extent of the enlargement of the spleen and the liver usually are helpful in
giving some clue to the cause of hepatosplenomegaly . In myelofibrosis and
chronic myeloid leukemia the spleen is markedly enlarged , whereas in acute
leukemia the enlargement is less than moderate , Locally : Determine the extent
of enlargement in centimeteres or inches , Look for ascites Elsewhere : Look for
lymphadenopathy , spider naevi , leuconychia , raised JVP, jaundice, anaemia ,
sternal tenderness ,gynaecomastia, testicular atrophy, oedema of feet,
Common associated conditions : Lymphoma , leukemia , Cirrhosis ,
Myeloproliferative disorders. (Q) What are the causes of portal hypertension ?
(A) (1) Cirrhosis of the liver (2) Hepatic vein obstruction (Budd Chiari syndrome )
(3) Constrictive pericarditis .
(Q) What are the causes of ascites ? (A) (1) Transudation : Cirrhosis,
Nephrotic syndrome , Congestive cardiac failure .
(2) Exudation : Tuberculous peritonitis , Carcinomatous peritonitis
(3) Chylous ascites : Malignancy , Trauma

215

Splenomegaly
Always ask the patient to turn on his right side ,so that the splenic notch can be easily
palpable in cases where there is only marginal enlargement of the spleen . For a mildly
enlarged spleen : lymphoma, anaemia and polythythemia are important causes and the
presence or absence of co existing features such as pallor, plethoric facies ,liver
enlargement ,lymphadenopathy should help in deciding the most propable cause
.Moderat to grossly enlarged spleens are commonly a result of myelofibrosis , chronic
myeloid leukemia or rarely due to malaria . Cirrhosis is the most likely cause of
splenomegaly associated with portal hypertension .
Locally : Describe how much the spleen is enlarged (in centimetres or inches ) - Look for
hepatomegaly -Look for ascites .
Elswhere :Look for lymphadenopathy ,anaemia , jaundice , plethoric facies, sternal
tenderness ,splinter hemorrhages ,rheumatoid hands.
Common conditions : (A) Mild enlargement of the spleen :
Viral hepatitis , Infectious mononucleosis ,Septicemia ,Typhoid fever , Brucellosis ,
Systemic lupus erythematosus , Felty,s syndrome .
(B) Moderate enlargement of the spleen :
- Chronic lymphatic leukemia - Malignant lymphoma - Acute leukemia
-Anaemia (1) Aplastic (2) Pernicious (3) Hemolytic
- Idiopathic thrombocytopenic purpura - Biliary cirrhosis - Infective endocarditis ,
Sarcoidosis , Amyloidosis .
(C) Massive enlargement of the spleen :
-Chronic myeloid leukemia -Myelofibrosis -polythemia rubra vera
Malaria - Kala azar . (Q) What are the features of enlarged spleen ?
(A) (1) Palpable splenic notch .(2) Inability to get in below the costal margin . (3) Not
bimanually palpable , unlike enlarged kidney
(4) Presence of dullness , unlike a band of resonance over the kidney .
(5) An enlarged spleen moves with respiration .
(6) The direction of enlargement is medially .

216

217

Hepatomegaly
In such a case it is important to know not only the extent of liver enlargement but to look for
important associated features of certain conditions mentioned below . Relevant features for
congestive cardiac failure would be : a raised jugular venous pressure and oedema of the
feet , along with an enlarged, firm , tender liver with a smooth surface and regular sharp
edge . If the liver enlargement is due to cirrhosis , the consistency of the liver and
splenomegaly ,ascites ,spider naevi ,Dupuytren,s contracture and palmar erythema are
commonly seen , a hard knobbly liver with or without tenderness is suggestive of
carcinomatosis . Locally : Describe the extent of enlargement preferably in centimeteres or
inches and not in finger breadth .- Edge of the liver regular or irregular - Surface : smooth or
nodular -Consistency :smooth or hard -Tenderness : present or not -Always percuss the
lower and upper borders (normally dull up to the 4th intercostal space ) - Auscultate for any
bruit or rub - Check for any splenomegaly - Check for ascites or caput medusae .Elswhere :
Look for lymphadenopathy ,spider naevi , Leuconychia , raised jugular venous pressure
,jaundice , gynaecomastia ,testicular atrophy ,pedal oedema ,abscence of secondary sexual
hair
Common associated conditions : Congestive cardiac failure , Cirrhosis of the liver ,
Carcinoma of the liver (secondaries most commonly ) ,Infective hepatitis .(Q) What
investigations would you ask for ?
(A) (1) Liver function tests (2)Liver scan (3) Hepatitis profile (4) Alphafetoprotein (4) Liver
biopsy .

218

Contents
Haha
SSSSSCCSubject
EErythema

marginatum
Erythema infectiosum
Erythema nodosum
Erythema induratum
Erythema multiforme
Erythema migran
Herpes simplex
Chicken Pox
Parotid swelling
Mumps
Rubella (German measles)
Measles
Roseola infantum
Herpes Zoster
Ramsy hunt syndrome
Infectious mononucleaosis
Dengue fever or break bone fever
Bird flu
Swine flu
Pyoderma gangrenosum
Bacterial meningitis
TB meningitis
Cerebral tuberculomas
Encephalitis
Rabies encephalitis
Xanthochromic CSF
Tetanus
Botulism
Opisthotonos
Trichinosis
Filariasis
Malaria
Cachexia
Sites of infection in critically ill patient
Pulmonary tuberculosis
Brucellosis
Leprosy

PPage
4
5
6
7
8
9
10
11
12
13
14
15
16
17
18
19
20
21
22
23
24
25
26
27-28
29
30
31
32
33
34
35
36-37
38
39
40
41-42
43

219

Leptospirosis
Erysipelas
Scarlet fever
Stasis dermatitis and Stasis eczema
Dermatitis herpetiformis
Deep venous thrombosis
The meningococcal rash
Oral leukoplakia
Candida infection or thrush
Hand foot and mouth disease
Impetigo
Staphylococcal scalded skin syndrome
Angular cheilitis
Cold sores (fever blisters)
Aphthous stomatitis
Acute herpetic gingivo stomatitis
Lichen planus
Recurrent superficial abscesses
Intravenous drug misuse
Necrotizing fasciitis
Hydatid disease
Typhoid fever
Rheumatic fever
Arthritis and subcuteanous nodules in rheumatic fever
Infective endocarditis
Roths spots
Splinter hemorrhage
Janeway lesion
Oslers nodes
Familial Mediterranean Fever
Heart failure
Coarctation of aorta
Atrial septal defect
Ventricular septal defect
Aortic incompetence
Aortic stenosis
Acanthosis nigricans
Xanthelasma
Necrobiosis lipoidica diabeticorum
Granuloma annulare
Kaposi Sarcoma

44
45
46
47
48
49
50
51
52
53
54
55
56
57
58
59
60
61-62
63
64
65
66
67
68
69-70
71-72
73
74
75
76
77-78
79
80
81
82
83
84
85
86
87
88

220

Subject
Hodgkins disease or Hodgkins lymphoma
Osteoprosis
Rickets
Vitamin A defeciency
Osteogenesis imperfecta
Turner and pseudo turner (Noonan) Syndrome
Mongolism
Achondroplasia
Marfan Syndrome
Rheumatoid arthritis
Septic arthritis
Reiters disease and reactive arthritis
Psoraitic arthritis
Swelling of the phalanges
Osteoarthritis
Heberdens nodes
Systemic lupus erythematosus
Malar rash
Discoid lupus
Sjogrens Syndrome
Behcets syndrome
Stills disease
Dermatomyositis
Scleroderma
Gout
Hands with joint swelling
Carpal tunnel syndrome
Absent radial pulse (either side)
Kyphoscoliosis
Acute pulmonary edema
Superior vena cava syndrome
Lung collapse
Fibrosing alveolitis
Empyema
Pleural effusion
Consolidation of the lung
Cyanosis
Clubbing of fingers
Lupus pernio
Digital ulcers

Page
89
90
91
92
93
94
95
96
97-98
99
100
101
102
103
104
105
106-107
108
109
110-111
112
113-114
115
116
117
118-119
120
121
122
123-124
125
126
127
128
129
130
131-132
133
134
135

221

Subject
Raynauds syndrome
Peripheral vascular disease
Longitudinal melanotic band of the nail
Nail changes associated with systemic diseases
Leg ulcers
Steven Johnson syndrome
Cuteanous vasculitis
Henoch Schonlein purpura
Tongue abnormalities
Jaundice
Carotenemia
Pyogenic liver abscess
Amoebic liver abscess
Wilsons disease
Primary biliary cirrhosis
Palmar erythema
Gynaecomastia
Spider naevi
Dupuytrens contracture
Leuconychia
Lower limb swelling
Hereditary hemorrhagic telangictasia
Beta thalassemia
Idiopathic thrombocytopenic purpura
Disseminated intravascular coagulation
Polythythemia
Hemophilia
Grey Turners syndrome
Virchows node(Left supraclavicular lymphadenopathy)
Fixed drug eruption
Lupus vulgaris
Ichthyosis vulgaris
Acne vulgaris
Rhinophyma
Rosacea
Angioedema
Eczema
Pemphigus
Pemphigoid
Vitiligo

Page
136
137
138
139
140
141
142
143-144
145
146-147
148
149
150
151
152
153
154
155
156
157
158
159
160
161
162
163
164
165
166
167
168
169
170
171
172
173
174
175
176
177

222

Subject
Ankylosing spondylitis
Livedo reticularis
Cluster headache
Migrain
Pes cavus
Syringomyelia/Syringobubial
Dystrophia myotonica
Facio scapulohumeral muscular dystrophy
Pseudo bulbar palsy
Horners Syndrome
Neurofibromatosis
Papilloedema
Parkinsonism
Subarachnoid hemorrhage
Cerebral edema
Facial (7th nerve) palsy
Abducent (6th nerve) palsy
Oculomotor (3rd nerve ) palsy
Ptosis
Hydrocephalus
Multiple sclerosis
Cushings syndrome
Alopecia
Hirsutism
Acromegaly
Goitre
Addisons disease
Exophthalmos or proptosis
Renal enlargement
Renal mass
Uremic facies
Nephrotic syndrome
Wegeners granulomatosis
Chaurg Strauss Syndrome
Lymphadenopathy
Hepatosplenomegaly
Splenomegaly
Hepatomegaly
Contents

Page
178
179
180
181
182
183
184
185
186
187
188
189
190
191
192-193
194
195
196
197
198
199
200
201
202
203
204
205
206
207
208
209
210
211
212
213
214
215-216
217
218-222

223

Sad

Wo

Dr. Hamdy Mohammed Ibrahim

Consultant of Internal Medicine , Imbaba Fever Hospital , Giza ,Egypt


MBchB (Cairo University) , MSc Internal Medicine (Cairo University, Egypt).
Head of the ICU , Imbaba Fever Hospital .
Member of the Egyptian Society for Fevers
Lecturer and Instructor at the Egyptian Ministry of Health .
i

As per copy regulation any unauthorized use of the material or part there of as commercial
reproduction or multiple distribution by any traditional or electronically based reproduction /
publication method is strictly prohibited

You might also like